PDA

نسخه کامل مشاهده نسخه کامل : اتاق ریاضیات(طرح سؤالات)



صفحه ها : 1 2 3 4 5 6 7 8 9 10 11 12 13 14 15 16 17 [18] 19 20

davy jones
19-02-2012, 21:20
با سلام .

ولی 0^0 اصلا تعریف شده هست ؟

بعد رابطه سری ها با تابع ای که برابر شده فقط برای بازه همگرایی برقرار هست دیگه درسته ؟

سلام.

صفر به توان صفر تعریف نشده است ولی از سمت راست حدش برابر با یک میشه.

در رابطه با سوال دومتون هم باید گفت که بله. فقط در بازه همگرایی برقراره. در حالت کلی به ازای [ برای مشاهده لینک ، لطفا با نام کاربری خود وارد شوید یا ثبت نام کنید ] داریم:


[ برای مشاهده لینک ، لطفا با نام کاربری خود وارد شوید یا ثبت نام کنید ] Bn%7D=%5Cfrac%7B1-x%5E%7Bn+1%7D%7D%7B1-x%7D



================================

سلام
یه سوال داشتم:
می خوام یه خط به معادله (y=f(x رو یه دوران بدم.

[ برای مشاهده لینک ، لطفا با نام کاربری خود وارد شوید یا ثبت نام کنید ]


معادله خط جدید چطور بدست میاد؟


[ برای مشاهده لینک ، لطفا با نام کاربری خود وارد شوید یا ثبت نام کنید ]

سپاس.


سلام. فک کنم منظورتون اینه که کل نمودار رو در جهت مثلثاتی به اندازه آلفا حول مبدا بچرخونیم. راحت ترین راه استفاده از ماتریس دوران هستش:


[ برای مشاهده لینک ، لطفا با نام کاربری خود وارد شوید یا ثبت نام کنید ]


که اگه از رابطه ی زیر استفاده کنید در آن x و y متغیرهای قدیمی و 'x و 'y متغیرهای جدید حاصل از دوران بر حسب x و y خواهند بود:


[ برای مشاهده لینک ، لطفا با نام کاربری خود وارد شوید یا ثبت نام کنید ]

[ برای مشاهده لینک ، لطفا با نام کاربری خود وارد شوید یا ثبت نام کنید ]
[ برای مشاهده لینک ، لطفا با نام کاربری خود وارد شوید یا ثبت نام کنید ]


برای مطالعه ی بیشتر به لینک زیر مراجعه کنین:
[ برای مشاهده لینک ، لطفا با نام کاربری خود وارد شوید یا ثبت نام کنید ]




موفق باشین.
90/11/30

skyzare
19-02-2012, 21:28
سلام.

صفر به توان صفر تعریف نشده است ولی از سمت راست حدش برابر با یک میشه.

در رابطه با سوال دومتون هم باید گفت که بله. فقط در بازه همگرایی برقراره. در حالت کلی به ازای [ برای مشاهده لینک ، لطفا با نام کاربری خود وارد شوید یا ثبت نام کنید ] داریم:


[ برای مشاهده لینک ، لطفا با نام کاربری خود وارد شوید یا ثبت نام کنید ] Bn%7D=%5Cfrac%7B1-x%5E%7Bn+1%7D%7D%7B1-x%7D


موفق باشین.
90/11/30





با سلام .

با تشکر از پاسختون .

ببخشید چه جوری صفر به توان صفر از سمت راست میشه یک ؟

ببخشید اون رابطه سری که نوشتید به ازای همه ی x ها هست ؟ تمام R ؟ ولی فکر کنم اگه x رو خارج از

[ برای مشاهده لینک ، لطفا با نام کاربری خود وارد شوید یا ثبت نام کنید ] بگذاریم سری واگرا میشه . درسته ؟

subuntu
19-02-2012, 21:30
سلام
در تابع [ برای مشاهده لینک ، لطفا با نام کاربری خود وارد شوید یا ثبت نام کنید ] y را در دست داریم و می خواهیم x را بدست آوریم . نحوه بدست آوردن x چگونه هست ؟
(اگه راه حلش طولانی باشه و تایپش وقتگیر باشه ، اگه فقط به مبحثش اشاره کنین میرم از کتاب میخونم ، با تشکر )

davy jones
19-02-2012, 21:36
با سلام .

با تشکر از پاسختون .

ببخشید چه جوری صفر به توان صفر از سمت راست میشه یک ؟

ببخشید اون رابطه سری که نوشتید به ازای همه ی x ها هست ؟ تمام R ؟ ولی فکر کنم اگه x رو خارج از

[ برای مشاهده لینک ، لطفا با نام کاربری خود وارد شوید یا ثبت نام کنید ] بگذاریم سری واگرا میشه . درسته ؟

سلام.

در مورد حد صفر به توان صفر باید از سری مک لورن تابع [ برای مشاهده لینک ، لطفا با نام کاربری خود وارد شوید یا ثبت نام کنید ] در همسایگی x=0 کمک گرفت:


[ برای مشاهده لینک ، لطفا با نام کاربری خود وارد شوید یا ثبت نام کنید ] Cfrac%7B%28x%5Cln&space;%28x%29%29%5E%7B2%7D%7D%7B2%21%7 D+%5Cfrac%7B%28x%5Cln&space;%28x%29%29%5E%7B3%7D%7D%7B3% 21%7D+...+%5Cfrac%7B%28x%5Cln&space;%28x%29%29%5E%7Bn%7D %7D%7Bn%21%7D+...%5CRightarrow&space;%5Clim_%7Bx%5Cto0%5 E%7B+%7D%7Dx%5E%7Bx%7D=1



در مورد واگرایی اون سری در خارج از محدوده ی [ برای مشاهده لینک ، لطفا با نام کاربری خود وارد شوید یا ثبت نام کنید ] هم حق با شماست. خب البته همونطور که میبینین جوابی که بنده هم به دست آوردم به ازای [ برای مشاهده لینک ، لطفا با نام کاربری خود وارد شوید یا ثبت نام کنید ] به سمت بینهایت میره. بنابراین مشکلی از این نظر نداره.



موفق باشین.
90/11/30

subuntu
19-02-2012, 21:51
ببخشید دو سوال دیگر هم دارم .
1-نحوه مشتق گرفتن از [ برای مشاهده لینک ، لطفا با نام کاربری خود وارد شوید یا ثبت نام کنید ] چگونه هست ؟
2- اگر بخواهیم از طرفین معادله Ln بگیریم ، اینطور Ln گرفتن صحیح هست؟ :
[ برای مشاهده لینک ، لطفا با نام کاربری خود وارد شوید یا ثبت نام کنید ] y=x%5Cln&space;x
البته میدونم اشتباهه ها ، ln گرفتن درستش چطور هست ؟
ممنون

davy jones
19-02-2012, 22:06
ببخشید دو سوال دیگر هم دارم .
1-نحوه مشتق گرفتن از [ برای مشاهده لینک ، لطفا با نام کاربری خود وارد شوید یا ثبت نام کنید ] چگونه هست ؟
2- اگر بخواهیم از طرفین معادله Ln بگیریم ، اینطور Ln گرفتن صحیح هست؟ :
[ برای مشاهده لینک ، لطفا با نام کاربری خود وارد شوید یا ثبت نام کنید ] y=x%5Cln&space;x
البته میدونم اشتباهه ها ، ln گرفتن درستش چطور هست ؟
ممنون

سلام.

در مورد سوال پست قبلیتون کمی سرچ کردم ولی نتیجه ی خاصی عاید نشد. بنابراین باید گفت که به نظر میاد تنها راه پیدا کردن ریشه های معادله ی [ برای مشاهده لینک ، لطفا با نام کاربری خود وارد شوید یا ثبت نام کنید ] که در اون k عدد ثابت هستش، استفاده از روشهای تقریب عددی است.

و اما در مورد مشتق این تابع. روشش استفاده از همین کاری هستش که خودتون اشاره کردین. یعنی از دو طرف لگاریتم میگیریم (روش لگاریتم گرفتن تون هم درسته) :


[ برای مشاهده لینک ، لطفا با نام کاربری خود وارد شوید یا ثبت نام کنید ] =%5Cln&space;%28x%5E%7Bx%7D%29=x%5Cln&space;%28x%29%5CRightarr ow&space;%7B[%5Cln&space;%28y%29]%7D%27=%7B[x%5Cln&space;%28x%29]%7D%27%5CRightarrow&space;%5Cfrac%7B%7By%7D%27%7D%7By%7D =%5Cln&space;%28x%29+%5Cfrac%7Bx%7D%7Bx%7D%5Coverset%7Bx %5Cneq&space;0%7D%7B=%7D%5Cln&space;%28x%29+1%5CRightarrow&space;%7B y%7D%27=%28%5Cln&space;%28x%29+1%29.y%5CRightarrow&space;%7B%5 Ccolor%7BGolden%7D&space;%7By%7D%27=%28%5Cln&space;%28x%29+1%2 9.x%5E%7Bx%7D%7D



موفق باشین.
90/11/30

hts1369
20-02-2012, 10:38
با سلام .

با تشکر از پاسختون .

ببخشید چه جوری صفر به توان صفر از سمت راست میشه یک ؟


حمید جان جواب رو با استفاده از سری مکلورن گذاشتن اینطوری هم میشه به این جواب رسید.
[ برای مشاهده لینک ، لطفا با نام کاربری خود وارد شوید یا ثبت نام کنید ]

davy jones
20-02-2012, 10:39
سلام به همگی دوستان.

یه سوال جالب رو بهش برخوردم که دیدم مطرح کردنش در اینجا خالی از لطف نیست:

اگر عدد دو رقمی [ برای مشاهده لینک ، لطفا با نام کاربری خود وارد شوید یا ثبت نام کنید ] b%7D مضرب 19 باشد آنگاه ثابت کنید که [ برای مشاهده لینک ، لطفا با نام کاربری خود وارد شوید یا ثبت نام کنید ] برابر با 19 میشود و بالعکس. یعنی اگر به ازای هر دو عدد طبیعی [ برای مشاهده لینک ، لطفا با نام کاربری خود وارد شوید یا ثبت نام کنید ] که داشته باشیم [ برای مشاهده لینک ، لطفا با نام کاربری خود وارد شوید یا ثبت نام کنید ] آنگاه عدد دورقمی [ برای مشاهده لینک ، لطفا با نام کاربری خود وارد شوید یا ثبت نام کنید ] b%7D حتما مضرب 19 خواهد بود .


موفق باشین.
90/12/1

hts1369
20-02-2012, 12:13
این انتگرال رو درست حل کردم
حل درسته ولی از تغییر ناحیه ها مطمِئن نیستم
[ برای مشاهده لینک ، لطفا با نام کاربری خود وارد شوید یا ثبت نام کنید ]

pinokiu
20-02-2012, 16:02
سلام به همگی دوستان.

یه سوال جالب رو بهش برخوردم که دیدم مطرح کردنش در اینجا خالی از لطف نیست:

اگر عدد دو رقمی [ برای مشاهده لینک ، لطفا با نام کاربری خود وارد شوید یا ثبت نام کنید ] b%7D مضرب 19 باشد آنگاه ثابت کنید که [ برای مشاهده لینک ، لطفا با نام کاربری خود وارد شوید یا ثبت نام کنید ] برابر با 19 میشود و بالعکس. یعنی اگر به ازای هر دو عدد طبیعی [ برای مشاهده لینک ، لطفا با نام کاربری خود وارد شوید یا ثبت نام کنید ] که داشته باشیم [ برای مشاهده لینک ، لطفا با نام کاربری خود وارد شوید یا ثبت نام کنید ] آنگاه عدد دورقمی [ برای مشاهده لینک ، لطفا با نام کاربری خود وارد شوید یا ثبت نام کنید ] b%7D حتما مضرب 19 خواهد بود .


موفق باشین.
90/12/1

ab=10a+b
and
a=19-2b
(ab=b+10*(19-2b
ab=190-19b
(ab=19(10-b

davy jones
20-02-2012, 23:22
ab=10a+b
and
a=19-2b
(ab=b+10*(19-2b
ab=190-19b
(ab=19(10-b


سلام

ممنون از اینکه وقت گذاشتین.
اما شما فقط عکس قضیه رو ثابت کردین. یعنی اگه a+2b=19 شد آنگاه عدد دو رقمی ab بر 19 بخشپذیره. اما صورت خود قضیه رو لزوما نمیشه نتیجه گرفت.

در هر حال ممنون.

============

این انتگرال رو درست حل کردم
حل درسته ولی از تغییر ناحیه ها مطمِئن نیستم

سلام.
انتگرالی مشاهده نمیشه :41:
پیشنهاد میکنم برای فرمول نویسی از سایت معروف زیر که بارها در جاهای مختلف استفاده از اون رو جناب مفیدی تذکر دادند استفاده کنین. یه خوبی ای که داره اینه که هیچگاه فرمولها پاک نمیشن و شما 10 سال بعد هم که بیاین و آرشیو این صفحات رو نگاه کنین، باز هم فرمولهایی که با کدهای استاندارد لاتکس نوشته شده، نشون داده میشه:

[ برای مشاهده لینک ، لطفا با نام کاربری خود وارد شوید یا ثبت نام کنید ]



==============================================



یه سوال مربوط به هندسه هم طرح میکنم تا هر کس علاقه داره جواب بده:

ثابت کنید در هر مثلث قائم الزاویه، مجموع مربعات سه میانه ی مثلث برابر با 1.5 برابر مجذور وتر است.



موفق باشین.
90/12/1

ask_bl
20-02-2012, 23:58
سلام

یه سوال مربوط به هندسه هم طرح میکنم تا هر کس علاقه داره جواب بده:

ثابت کنید در هر مثلث قائم الزاویه، مجموع مربعات سه میانه ی مثلث برابر با 1.5 برابر مجذور وتر است.



موفق باشین.
90/12/1
سلام
r , m & z/2 میانه هان که z/2 هم میانه وارد به وتره


[ برای مشاهده لینک ، لطفا با نام کاربری خود وارد شوید یا ثبت نام کنید ] {\color{Blue} (\frac{z}{2})^{2}+r^{2}+m^{2}=\left ( \frac{x^{2}+y^{2}}{4} \right )+\left (\frac{x^{2}}{4}+y^{2} \right )+\left (x^{2}+\frac{y^{2}}{4} \right )=\frac{3}{2}z^{2}}

nomid
21-02-2012, 07:46
سلام به همگی دوستان.

یه سوال جالب رو بهش برخوردم که دیدم مطرح کردنش در اینجا خالی از لطف نیست:

اگر عدد دو رقمی [ برای مشاهده لینک ، لطفا با نام کاربری خود وارد شوید یا ثبت نام کنید ] b%7D مضرب 19 باشد آنگاه ثابت کنید که [ برای مشاهده لینک ، لطفا با نام کاربری خود وارد شوید یا ثبت نام کنید ] برابر با 19 میشود و بالعکس. یعنی اگر به ازای هر دو عدد طبیعی [ برای مشاهده لینک ، لطفا با نام کاربری خود وارد شوید یا ثبت نام کنید ] که داشته باشیم [ برای مشاهده لینک ، لطفا با نام کاربری خود وارد شوید یا ثبت نام کنید ] آنگاه عدد دورقمی [ برای مشاهده لینک ، لطفا با نام کاربری خود وارد شوید یا ثبت نام کنید ] b%7D حتما مضرب 19 خواهد بود .


موفق باشین.
90/12/1




__
ab = 19x
10a + b = 19x
20a + 2b =19 * 2x
19a + a + 2b = 19 * 2x
a + 2b = 19 * 2x - 19a
(a + 2b = 19 * (2x - a

---------------------

a + 2b = 19
10a + 20b = 190
10a + b = 190 - 19b
__
(ab = 19 * (10 - b

pinokiu
21-02-2012, 09:09
__
ab = 19x
10a + b = 19x
20a + 2b =19 * 2x
19a + a + 2b = 19 * 2x
a + 2b = 19 * 2x - 19a
(a + 2b = 19 * (2x - a





البته اگر نشان می دادید که چطور 2x-a مساوی 1 می شود کامل تر می شد
چون عدد ab دو رقمی است و ضریب x کوچکتر از 10 است بنابراین رقم دهگان مضارب 19 برابر است با:
a=2x-1
در نتیجه:
2x-a=2x-2x+1=1

SuperSt@r
21-02-2012, 11:56
سلام دوستان
الان يه سوال به دستم رسيده خودم زياد روش فكر نكردم چون ميخاستم تا شب به جواب برسم گفتم اينجام بذارم همزمان با هم روش فك كنيم

[ برای مشاهده لینک ، لطفا با نام کاربری خود وارد شوید یا ثبت نام کنید ]

davy jones
21-02-2012, 12:33
سلام دوستان
الان يه سوال به دستم رسيده خودم زياد روش فكر نكردم چون ميخاستم تا شب به جواب برسم گفتم اينجام بذارم همزمان با هم روش فك كنيم

[ برای مشاهده لینک ، لطفا با نام کاربری خود وارد شوید یا ثبت نام کنید ]

سلام.

از تغییر متغیر [ برای مشاهده لینک ، لطفا با نام کاربری خود وارد شوید یا ثبت نام کنید ] frac%7Bx%7D%7B2%7D%29 استفاده کنین.

اینم جواب آخرش با استفاده از سایت والفرام. (گزینه ی show steps رو بزنین تا مراححل حل رو هم نشون بده):

[ برای مشاهده لینک ، لطفا با نام کاربری خود وارد شوید یا ثبت نام کنید ]{dx}{c+plus+\sin%28x%29} ([ برای مشاهده لینک ، لطفا با نام کاربری خود وارد شوید یا ثبت نام کنید ] )



موفق باشین.
90/12/2

SuperSt@r
21-02-2012, 16:00
سلام.

از تغییر متغیر [ برای مشاهده لینک ، لطفا با نام کاربری خود وارد شوید یا ثبت نام کنید ] frac%7Bx%7D%7B2%7D%29 استفاده کنین.

اینم جواب آخرش با استفاده از سایت والفرام. (گزینه ی show steps رو بزنین تا مراححل حل رو هم نشون بده):

[ برای مشاهده لینک ، لطفا با نام کاربری خود وارد شوید یا ثبت نام کنید ]{dx}{c+plus+\sin%28x%29} ([ برای مشاهده لینک ، لطفا با نام کاربری خود وارد شوید یا ثبت نام کنید ] om%2Finput%2F%3Fi%3D%255Cint%255Cfrac%257Bdx%257D% 257Bc%2Bplus%2B%255Csin%2528x%2529%257D)



موفق باشین.
90/12/2


ممنون داداش

خودمم ميخاستم از تانژانت نصف كمان برم چون معمولا اينجور سوالا از اين راه حل ميشه اگه راه حل ديگه اي داره بذاريد اخه استادمون يه جوري صحبت ميكرد كه ادم فك ميكرد يه راه ديگه اي هم داره چون كه بهش گفتن از اين راه گفت اره ميشه ولي يه جوري گفت كه انگار ميخاد از يه راهه ديگه براش حل كنيم
+
مخرج سوال به جاي علامت جمع علامت - هست كه اشتباه تايپي بود ممنون ميشم

skyzare
21-02-2012, 19:10
با سلام .

يه روش ديگه هم هست . (‌البته با چيزي كه اساتيد گفتند يكي هست :31: حالا من حالت كلي اش رو نوشتم :20:)‌ من متن كتاب رو مينويسم . ..... (‌متاسفانه اين سايت codeclge بازه نميشه كه بخوام رياضي بنويسم . )


در محاسبه انتگرال هايي كه تابع زير انتگرال به صورت f 1 / f ( sin ( x ) , cos(x) ) j تعريف ميشود با در نظر گرفتن روابط مثلثاتي :





برای مشاهده محتوا ، لطفا وارد شوید یا ثبت نام کنید

و با تغيير متغير tan ( x ) = t ميتوان انتگرال را حل كرد .



الان توي مخرج انتگرال شما عبارت sin ( x ) k هست كه بايد به صورت اين بنويسيد كه به جاي x‌ ها توي اون رابطه x/2 رو قرار بديد .

skyzare
22-02-2012, 22:54
با سلام .




انتگرالی مشاهده نمیشه [ برای مشاهده لینک ، لطفا با نام کاربری خود وارد شوید یا ثبت نام کنید ]
90/12/1


صورت سوالشون اين هست : (‌ البته الان پست قابل ديدن هست . شايد اون موقع سرور اپلود مشكل پيدا كرده بود ولي كلا بهتر از همون سايت codecoge‌ استفاده كنيم . )


این انتگرال رو درست حل کردم
حل درسته ولی از تغییر ناحیه ها مطمِئن نیستم






[ برای مشاهده لینک ، لطفا با نام کاربری خود وارد شوید یا ثبت نام کنید ] 0%7D%5E%7B%5Cinfty&space;%7D%5Cint_%7B0%7D%5E%7Bx%7Dxe%5 E%7B%5Cfrac%7B-x%5E2%7D%7By%7D%7Ddydx%7D=??%5C%5C%5C%5C%5C%5C&space;%5C int_%7B0%7D%5E%7B%5Cinfty&space;%7D%5Cint_%7By%7D%5E%7B&space; %5Cinfty%7Dxe%5E%7B%5Cfrac%7B-x%5E2%7D%7By%7D%7Ddxdy%5C%5C%5C%5C%5C%5C



با تغيير متغير داريم :

[ برای مشاهده لینک ، لطفا با نام کاربری خود وارد شوید یا ثبت نام کنید ] C:%5Crightarrow&space;%5C:&space;%5C:&space;%5C:&space;%5C:&space;%5C:&space;%5C:&space;%5C: &space;%5C:&space;%5C:&space;xdx=-%5Cfrac%7By%7D%7B2%7Ddu



[ برای مشاهده لینک ، لطفا با نام کاربری خود وارد شوید یا ثبت نام کنید ] fty&space;%7Dxe%5E%5Cfrac%7B-x%5E2%7D%7By%7Ddx=%5Cint_%7B-y%7D%5E%7B-%5Cinfty&space;%7D%5Cfrac%7B-y%7D%7B2%7De%5Eudu=-%5Cfrac%7By%7D%7B2%7D%5Cint_%7B-y%7D%5E%7B-%5Cinfty&space;%7De%5Eudu%5C%5C%5C%5C%5C%5C=-%5Cfrac%7By%7D%7B2%7De%5Eu%5Cmid&space;_%7B-y%7D%5E%7B-%5Cinfty&space;%7D=%5Cfrac%7By%7D%7B2%7De%5E%7B-y%7D





[ برای مشاهده لینک ، لطفا با نام کاربری خود وارد شوید یا ثبت نام کنید ] fty&space;%7D%5Cfrac%7By%7D%7B2%7De%5E%7B-y%7Ddy=%5Cfrac%7B1%7D%7B2%7D%28-ye%5E%7B-y%7D-e%5E%7B-y%7D%29%5Cmid&space;_0%5E%5Cinfty&space;=%5Cfrac%7B1%7D%7B2%7D





================================================

انشاالله كه اشتباه تايپي نداشته باشه :31: (‌ البته بعضي از راه حل هاي ميان رو هم حذف كردم )‌

hts1369
24-02-2012, 15:41
سلام.







برای این منظور ابتدا باید به این نکته دقت کنیم که:

[ برای مشاهده لینک ، لطفا با نام کاربری خود وارد شوید یا ثبت نام کنید ] B2%7D%7D%7D

(چرا؟)




موفق باشین.
90/11/3





داش حمید چرا رو بی جواب نذاشتم:31:
[ برای مشاهده لینک ، لطفا با نام کاربری خود وارد شوید یا ثبت نام کنید ] B1%7D%7B%5Csqrt%7B1+x%5E2%7D%7D%5C%5C&space;arctan&space;%28x% 29=a%5CRightarrow&space;%5Ctan&space;%28a%29=x%5C%5C&space;%5Ccos&space;%5 E2%28a%29=%5Cfrac%7B1%7D%7B1+tan%5E2&space;%28a%29%7D=%5 Cfrac%7B1%7D%7B1+x%5E2%7D%5C%5C&space;cos&space;%28a%29=%5Cfra c%7B1%7D%7B%5Csqrt%7B1+x%5E2%7D%7D%5C%5C&space;cos&space;%28ar ctan&space;%28x%29%29=%5Cfrac%7B1%7D%7B%5Csqrt%7B1+x%5E2 %7D%7D%5C%5C
یه کتاب دستم اومده پر از این جور سوالهاست به مرور میزارم حل کنیم.

hts1369
24-02-2012, 15:52
گفته این اتحاد رو ثابت کنید.
[ برای مشاهده لینک ، لطفا با نام کاربری خود وارد شوید یا ثبت نام کنید ] 7D%29=%5Carccos&space;%28%5Cfrac%7Bb+a%5Ccos&space;%5Ctheta&space;%7 D%7Ba+b%5Ccos&space;%5Ctheta&space;%7D%29
یکی دیگه رو با جوابش میزارم (البته از این بالایی خیلی اسونتر هست).
[ برای مشاهده لینک ، لطفا با نام کاربری خود وارد شوید یا ثبت نام کنید ] ta&space;%29+%5Carcsin&space;%28%5Csqrt%7Bcos2%5Ctheta&space;%7D%29= %5Cfrac%7B%5Cpi&space;%7D%7B2%7D%5C%5C&space;%5Carcsin&space;%28%5Cs qrt%7B2%7D%5Csin&space;%5Ctheta&space;%29=a%5CRightarrow&space;%5Csi n&space;a=%5Csqrt%7B2%7D%5Csin&space;%28%5Ctheta&space;%29%5CRightar row&space;%5Ccos&space;%28a%29=%5Csqrt%7B%5Ccos&space;2%5Ctheta&space;%7D% 5C%5C&space;%5Carcsin&space;%28%5Csqrt%7Bcos2%5Ctheta&space;%7D%29=b %5CRightarrow&space;%5Csin&space;%28b%29=%5Csqrt%7B%5Ccos&space;%282 %5Ctheta&space;%29%7D%5CRightarrow&space;%5Ccos&space;%28b%29=%5Csqr t%7B2%7D%5Csin&space;%5Cthet%28a%29%5C%5C&space;a+b=%5Cfrac%7B %5Cpi&space;%7D%7B2%7D%5C%5C%5CRightarrow&space;%5Csin&space;%28a+b% 29=1%5CRightarrow&space;%5Csin&space;%28a%29%5Ccos&space;%28b%29+%5C sin&space;%28b%29%5Ccos&space;%28a%29=1%5C%5C&space;%5Csqrt%7B2%7D%5 Csin&space;%28%5Ctheta&space;%29%5Csqrt%7B2%7D%5Csin&space;%28%5Cthe ta&space;%29+%5Csqrt%7B%5Ccos&space;2%5Ctheta&space;%7D%5Csqrt%7B%5C cos&space;2%5Ctheta%7D=2%5Csin&space;%5E%7B2%7D%5Ctheta&space;+%5Cco s&space;%282%5Ctheta&space;%29=1%5C%5C&space;2%5Csin&space;%5E%7B2%7D%5Cth eta&space;+%5Ccos&space;%282%5Ctheta&space;%29=1%5CRightarrow%5Ccos&space; %282%5Ctheta&space;%29=1-2%5Csin&space;%5E%7B2%7D%5Ctheta

mjorh
25-02-2012, 01:01
سلام به ریاضی دانان!
بچه ها تو روش نیوتن چجوری باید نقطه ی ابتدایی رو پیدا کنم ؟
مثلا تو این مثال : 2x+lnx=1

pinokiu
25-02-2012, 11:03
سلام به ریاضی دانان!
بچه ها تو روش نیوتن چجوری باید نقطه ی ابتدایی رو پیدا کنم ؟
مثلا تو این مثال : 2x+lnx=1
روش نیوتن ضعفش اینه که باید حدود جواب رو بدونید
نقطه ابتدایی رو باید خودتون حدس بزنید

skyzare
25-02-2012, 13:28
با سلام
ببخشيد اساتيد اگه سه نقطه در فضاي سه بعدي بدن بعد بخوايم معادله صفحه اش رو بنويسيم بايد چي كار كنم ؟ ( اگه يه نقطه و بردار قائم به صفحه رو داشته باشيم ميدونم چه جوري به دست بيارم )
با تشكر . :20:

ashjaee
25-02-2012, 14:53
با سلام
ببخشيد اساتيد اگه سه نقطه در فضاي سه بعدي بدن بعد بخوايم معادله صفحه اش رو بنويسيم بايد چي كار كنم ؟ ( اگه يه نقطه و بردار قائم به صفحه رو داشته باشيم ميدونم چه جوري به دست بيارم )
با تشكر . :20:

اول با دو نقطه بردار AB بعد AC رو تشکیل میدیم ضرب خارجی که کردیم، بردار قائم بدست میاد. با یکی از اون نقطه ها صفحه رو مینویسیم

pinokiu
25-02-2012, 16:24
سلام به ریاضی دانان!
بچه ها تو روش نیوتن چجوری باید نقطه ی ابتدایی رو پیدا کنم ؟
مثلا تو این مثال : 2x+lnx=1
البته شاید این مفید باشه
معمولا تو معادلاتی که جواب متناهی دارند می تونی این کار رو بکنی تا حدود جواب ها دستت بیاد
اول یک عدد مثبت بزرگ بده به الگوریتم
یک جواب بدست میاد مثلا a
بعد یک عدد منفی بزرگ بده به الگوریتم
یک جواب بدست میاد مثلا b
اگر اعداد مثبت و منفی رو به اندازه کافی بزرگ در نظر گزفته باشی
بقیه جوابها بین a و b قرار می گیرند
این بعضی مواقع می تونه کمک کنه
البته وقتی که معادله تعداد صفر هاش متناهی باشه

davy jones
25-02-2012, 19:31
روش نیوتن ضعفش اینه که باید حدود جواب رو بدونید
نقطه ابتدایی رو باید خودتون حدس بزنید
سلام.

ضعف بعدیش هم اینه که ممکنه شما رو از جواب دور کنه به جای اینکه به جواب نزدیک کنه.

مثلا شما نمودار تابع زیر رو در نظر بگیرین:


[ برای مشاهده لینک ، لطفا با نام کاربری خود وارد شوید یا ثبت نام کنید ]


[ برای مشاهده لینک ، لطفا با نام کاربری خود وارد شوید یا ثبت نام کنید ]


همونطور که خیلی هم واضحه و اگه اصلا نمودار رو هم نمیدیدیم، معلوم بود که ریشه ی این تابع x=0 هستش. اما شما هر بازه ی هرچند کوچک در همسایگی x=0 رو هم که در نظر بگیرین، با هربار استفاده از تقریب نیوتون، از ریشه دور تر میشین.

کلا این روش برای توابعی که ریشه ی اونها، ریشه ی ساده ی مشتق دوم همون تابع هم باشه به شرطی که تقعر تابع قبل از ریشه مثبت و بعد از آن منفی باشد، کاربرد ندارد.

به عنوان تمرین هر کسی که علاقه داشت میتونه یه بار روی همین تابع این روش رو امتحان کنه.


موفق باشین.
90/12/6

mjorh
25-02-2012, 21:26
مرسی بچه ها...:11:
آره باو روشش خیلی مضخرفه ...استادمون هم می گف ...ولی دو تا تمرین داد ک حلش کنیم !

pinokiu
25-02-2012, 23:16
مرسی بچه ها...:11:
آره باو روشش خیلی مضخرفه ...استادمون هم می گف ...ولی دو تا تمرین داد ک حلش کنیم !
البته مزیتش هم اینه که خیلی سریع به جواب دقیق می رسه
معمولا با 3 بار تکرار جواب بسیار دقیقی می ده

skyzare
25-02-2012, 23:53
با سلام .

ببخشيد اساتيد ميشه يه راهنمايي كنيد اگه سه نقطه در فضاي سه بعدي داشته باشيم چه جوري ميتونيم تشخيص بديم كه اين سه نقطه سه راس يه مثلث هست ؟ بايد چي كارش كنم ؟ مثلا


a=(2 4 1 )l


b=(-1 3 0 )l


c=( 5 5 2 )l

pinokiu
26-02-2012, 11:02
پاک شود...............................

hts1369
26-02-2012, 13:38
با سلام .

ببخشيد اساتيد ميشه يه راهنمايي كنيد اگه سه نقطه در فضاي سه بعدي داشته باشيم چه جوري ميتونيم تشخيص بديم كه اين سه نقطه سه راس يه مثلث هست ؟ بايد چي كارش كنم ؟ مثلا


a=(2 4 1 )l


b=(-1 3 0 )l


c=( 5 5 2 )l

شما باید نشون بدین که این سه نقطه روی یک صفحه نیستن (تو فضای دوبعدی نشون میدادیم که روی یک خط نیستن)

اطلاع رسان
26-02-2012, 13:39
سلام
((لطفا پاک شود))
با تشکر

pinokiu
26-02-2012, 14:41
شما باید نشون بدین که این سه نقطه روی یک صفحه نیستن (تو فضای دوبعدی نشون میدادیم که روی یک خط نیستن)
من فکر می کنم باز هم باید نشون بدن که این سه نقطه روی یک خط نیستن

skyzare
26-02-2012, 15:04
شما باید نشون بدین که این سه نقطه روی یک صفحه نیستن (تو فضای دوبعدی نشون میدادیم که روی یک خط نیستن)


من فکر می کنم باز هم باید نشون بدن که این سه نقطه روی یک خط نیستن


با سلام .

با تشكر از پاسخ شما .


خوب اگه بخوام نشون بدم كه توي يه صفحه نيستند بايد چي كار كنم ؟ خوب مگه سه نقطه داخل يه صفحه نمي شه باهاش مثلث درست كرد ؟ چرا ؟
شرمنده اگه بخوام نشون بدم كه توي يه خط نيستند بايد چي كار كنم ؟

==============================================

skyzare
26-02-2012, 15:43
سلام
ببخشید اگر میشه جواب این سوالات رو لطف کنید:
1-اگر معادله
p(x,y)dx+q(x,y)dy=0

2-فاکتور انتگرال گیری (انتگرال ساز)یک معادله همگن را بدست آورید.

با تشکر

با سلام .
2 - نميدونم درست هست يا نه ولي فكر كنم اين جور معادلات مرتبه اول رو از راه معادله كامل حل ميكنند . اگه شرطش برقرار باشه كه معادله حل ميشه ولي اگه شرطش برقرار نبود بايد عامل انتكرال براش پيدا كني بعد در طرفين معادله ضرب كنيد تا به معادله كامل تبديل بشه بعد معادله كامل رو حل كني . فرض كن داشته باشيم :



[ برای مشاهده لینک ، لطفا با نام کاربری خود وارد شوید یا ثبت نام کنید ](x,y)dx+q(x,y)dy=0

اگر معادله بالا كامل نباشه ممكنه بتونيم با ضرب تابعي به صورت [ برای مشاهده لینک ، لطفا با نام کاربری خود وارد شوید یا ثبت نام کنید ](x,y) در طرفين معادله ؛ معادله رو تبديل به معادله كامل كرد كه [ برای مشاهده لینک ، لطفا با نام کاربری خود وارد شوید یا ثبت نام کنید ](x,y) در صورت وجود فاكتور انتگرال گير ميگن .


================================================== ===


اول فرض مي كنيم كه [ برای مشاهده لینک ، لطفا با نام کاربری خود وارد شوید یا ثبت نام کنید ] صرفا تابعي از x‌ هست يعني [ برای مشاهده لینک ، لطفا با نام کاربری خود وارد شوید یا ثبت نام کنید ](x)


خوب در طرفين معادله ضرب ميكنيم :


[ برای مشاهده لینک ، لطفا با نام کاربری خود وارد شوید یا ثبت نام کنید ](x)p(x,y)dx+\mu&space;(x)q(x,y)dy=0


اگه معادله كامل باشه بايد داشته باشيم :


[ برای مشاهده لینک ، لطفا با نام کاربری خود وارد شوید یا ثبت نام کنید ]{\color{Red}&space;\frac{d}{dy}}{\color{Red}&space;( }\:&space;\:&space;\mu&space;(x)p(x,y){\color{Red}\:&space;\:&space;)\:&space;}={\colo r{Red}&space;\frac{d}{dx}}{\color{Red}&space;(}\:&space;\:&space;\mu&space;(x)q( x,y)\:&space;\:&space;{\color{Red}&space;)}


خوب حالا بعد از مشتق بالا داريم : ( داخل عبارت زير ديگر متغير وابسته هاي تابع ها رو مشخص نكردم در واقع مو تابعي از x‌ از طرفي p و q‌ هر دو تابعي از x‌ و y هستند . اون انديس y در زير p‌ يعني مشتق P(x ,y) l نسبت به متغير y . براي q هم انديس x‌ يعني مشتق q(x,y) l نسبت به متغير x اش .




[ برای مشاهده لینک ، لطفا با نام کاربری خود وارد شوید یا ثبت نام کنید ]'\:&space;\:&space;q+\mu&space;\:&space;\:&space;q_x



[ برای مشاهده لینک ، لطفا با نام کاربری خود وارد شوید یا ثبت نام کنید ]'\:&space;\:&space;q=\mu&space;\:&space;\:&space;q_x-\mu&space;\:&space;\:&space;p_y&space;\\\\&space;\mu&space;'\:&space;\:&space;q=\mu&space;(\:&space;\:&space;q_x-\:&space;\:&space;p_y&space;)\rightarrow&space;\frac{\mu&space;'}{\mu}=\frac{ q_x-p_y}{q}


پس اگر [ برای مشاهده لینک ، لطفا با نام کاربری خود وارد شوید یا ثبت نام کنید ]{q_x-p_y}{q} تابعي از x‌ باشه [ برای مشاهده لینک ، لطفا با نام کاربری خود وارد شوید یا ثبت نام کنید ] به صورت تابعي از x‌ وجود داره كه داريم :


[ برای مشاهده لینک ، لطفا با نام کاربری خود وارد شوید یا ثبت نام کنید ]{\mu&space;'}{\mu&space;}=p(x)&space;\righta rrow&space;ln(\mu&space;)=\int&space;p(x)dx&space;\\\\&space;{\color{Red}&space;\mu&space;(x )=e^{\int&space;p(x)d(x)}}



============================================


براي حالتي كه مو تابعي از y باشه هم به همين فرم بريد جلو . رابطه ميشه :



[ برای مشاهده لینک ، لطفا با نام کاربری خود وارد شوید یا ثبت نام کنید ]{\mu&space;'}{\mu&space;}=p(y)&space;\righta rrow&space;ln(\mu&space;)=\int&space;p(y)dy&space;\\\\&space;{\color{Red}&space;\mu&space;(y )=e^{\int&space;p(y)d(y)}}


البته p در اين حالت قريته اون p بالايي هست و توي اين حالت بايد صرفا تابعي از y‌ باشه .


===============================================


براي حالتي كه تابعي هم كه مو تابعي از X , Y باشه هم قاعدتا بايد همين جوري باشه ولي جواب اخرش رو نمي دونم چه جوري ميشه . :41:


استادمون هم اقاي عزيزي اون موقع كه معادلات داشتيم اين مورد رو برامون حل نكرد :31:




============================================

hts1369
26-02-2012, 16:32
من فکر می کنم باز هم باید نشون بدن که این سه نقطه روی یک خط نیستن
بله درست میفرمایید

pinokiu
26-02-2012, 18:07
با سلام .

با تشكر از پاسخ شما .


خوب اگه بخوام نشون بدم كه توي يه صفحه نيستند بايد چي كار كنم ؟ خوب مگه سه نقطه داخل يه صفحه نمي شه باهاش مثلث درست كرد ؟ چرا ؟
شرمنده اگه بخوام نشون بدم كه توي يه خط نيستند بايد چي كار كنم ؟

==============================================

شما باید با این سه نقطه سه بردار بسازید
فرقی نمی کنه نقطه شروع و پایان بردار چی باشه
بردار های یکه رو هم برای هر کدوم بدست بیارید
حالا این سه بردار یکه هیچکدوم نباید برابر یا منفی دیگری باشند
در غیر این صورت این سه نقطه روی یک خط هستند

davy jones
26-02-2012, 19:36
با سلام .

ببخشيد اساتيد ميشه يه راهنمايي كنيد اگه سه نقطه در فضاي سه بعدي داشته باشيم چه جوري ميتونيم تشخيص بديم كه اين سه نقطه سه راس يه مثلث هست ؟ بايد چي كارش كنم ؟ مثلا


a=(2 4 1 )l


b=(-1 3 0 )l


c=( 5 5 2 )l

سلام.

علاوه بر راهی که دوستان اشاره کردن، یه راه دیگه هم وجود داره که البته یکم طولانی تره و اونم اینکه طول پاره خطهای ab ، bc و ac رو محاسبه کنین و نشون بدین که در نامساوی مثلث صدق میکنه یا نه.
(یادآوری: در هر مثلث، مجموع طول هر دو ضلع مثلث از ضلع سوم بزرگتر است و بالعکس. یعنی هر سه عددی که مجموع هر 2 تای اونها از سومی بزرگتر باشه میتونه طول سه ضلع سک مثلث باشه.)

===============================




براي حالتي كه تابعي هم كه مو تابعي از X , Y باشه هم قاعدتا بايد همين جوري باشه ولي جواب اخرش رو نمي دونم چه جوري ميشه . [ برای مشاهده لینک ، لطفا با نام کاربری خود وارد شوید یا ثبت نام کنید ]


استادمون هم اقاي عزيزي اون موقع كه معادلات داشتيم اين مورد رو برامون حل نكرد [ برای مشاهده لینک ، لطفا با نام کاربری خود وارد شوید یا ثبت نام کنید ]


برای اون حالت، قاعده ی کلی وجود نداره. معمولا میان و ابتدا فرض میکنن که تابع مو، به صورت حاصل جمع یک تابع منحصرا از x و یک تابع منحصرا از y باشه. یعنی بشه اون رو اینطوری نوشت:


[ برای مشاهده لینک ، لطفا با نام کاربری خود وارد شوید یا ثبت نام کنید ] 7B1%7D%28x%29+%5Cmu&space;_%7B2%7D%28y%29%5Crightarrow&space;% 5Cleft%5C%7B%5Cbegin%7Bmatrix%7D&space;%5Cfrac%7B%5Cpart ial&space;%5Cmu&space;_%7B1%7D%7D%7B%5Cpartial&space;y%7D=0%5C%5C&space;%5 C%5C&space;%5Cfrac%7B%5Cpartial&space;%5Cmu&space;_%7B2%7D%7D%7B%5Cp artial&space;x%7D=0&space;%5Cend%7Bmatrix%7D%5Cright.


اگه به ازای این حالت تونستیم عامل انتگرال ساز رو گیر بیاریم که هیچ. وگرنه مجددا میان و فرض میکنن که تابع مو، به صورت حاصل ضرب یک تابع منحصرا از x و یک تابع منحصرا از y، جداشدنی باشه. یعنی اینطوری:


[ برای مشاهده لینک ، لطفا با نام کاربری خود وارد شوید یا ثبت نام کنید ] 7B1%7D%28x%29&space;%5Cmu&space;_%7B2%7D%28y%29%5Crightarrow&space;% 5Cleft%5C%7B%5Cbegin%7Bmatrix%7D&space;%5Cfrac%7B%5Cpart ial&space;%5Cmu&space;_%7B1%7D%7D%7B%5Cpartial&space;y%7D=0%5C%5C&space;%5 C%5C&space;%5Cfrac%7B%5Cpartial&space;%5Cmu&space;_%7B2%7D%7D%7B%5Cp artial&space;x%7D=0&space;%5Cend%7Bmatrix%7D%5Cright.



ممکنه که این حالت هم لزوما ما رو به جای خاصی نرسونه. اگه به جواب رسیدیم که هیچ وگرنه باید حالتهای جدیدی مثل [ برای مشاهده لینک ، لطفا با نام کاربری خود وارد شوید یا ثبت نام کنید ] 7B1%7D%28x%29e&space;%5E%7B%5Cmu&space;_%7B2%7D%28y%29%7D یا [ برای مشاهده لینک ، لطفا با نام کاربری خود وارد شوید یا ثبت نام کنید ] 7B2%7D%28y%29e&space;%5E%7B%5Cmu&space;_%7B1%7D%28x%29%7D یا [ برای مشاهده لینک ، لطفا با نام کاربری خود وارد شوید یا ثبت نام کنید ] %28%5Cmu&space;_%7B1%7D%28x%29+%5Cmu&space;_%7B2%7D%28y%29%29 و ... رو همینجور مدام امتحان کنین تا یکیش به جواب برسه. :31: ولی معمولش اینه که به یکی از دو حالت جداشدنی حاصل جمع و یا حاصلضرب، حل میشه.


موفق باشین.
90/12/7

skyzare
26-02-2012, 21:07
سلام.
علاوه بر راهی که دوستان اشاره کردن، یه راه دیگه هم وجود داره که البته یکم طولانی تره و اونم اینکه طول پاره خطهای ab ، bc و ac رو محاسبه کنین و نشون بدین که در نامساوی مثلث صدق میکنه یا نه.
(یادآوری: در هر مثلث، مجموع طول هر دو ضلع مثلث از ضلع سوم بزرگتر است و بالعکس. یعنی هر سه عددی که مجموع هر 2 تای اونها از سومی بزرگتر باشه میتونه طول سه ضلع سک مثلث باشه.)


با سلام .

ممنون از پاسخ همه اساتيد .

ببخشيد فرض كنيم كه هر سه تا نقطه داخل يه خط باشند به طوري كه بعد از درست كردن بردار ab‌ ac bc .....اندازه بردار ها طوري بشه كه جمع دو تاش از اون يكي بيشتر بشه . در اين صورت كه نميشه گفت ميشه با اين سه نقطه مثلث درست كرد با وجود اين كه در نا مساوي هم صدق كرده . ممكنه اين جوري اتفاق بيفته ؟


================================

hts1369
26-02-2012, 21:38
با سلام .

ممنون از پاسخ همه اساتيد .

ببخشيد فرض كنيم كه هر سه تا نقطه داخل يه خط باشند به طوري كه بعد از درست كردن بردار ab‌ ac bc .....اندازه بردار ها طوري بشه كه جمع دو تاش از اون يكي بيشتر بشه . در اين صورت كه نميشه گفت ميشه با اين سه نقطه مثلث درست كرد با وجود اين كه در نا مساوي هم صدق كرده . ممكنه اين جوري اتفاق بيفته ؟


================================

بله صحبت شما درسته فکر نمیکنم حرف حمید جان درست باشه.
اتحاد مثلثاتی حل کردم گفتم بزارمش ببینم نمیشه از این کوتاه تر به جواب رسید.(محاسبه ی تانژانت 4x پدر ادمو در میاره)

[ برای مشاهده لینک ، لطفا با نام کاربری خود وارد شوید یا ثبت نام کنید ] %7D%7B4%7D%5C%5C&space;%5Carctan&space;%5Cfrac%7B1%7D%7B5%7D=a %5CRightarrow&space;%5Ctan&space;%28a%29=%5Cfrac%7B1%7D%7B5%7D %5C%5C&space;%5Carctan&space;%28%5Cfrac%7B1%7D%7B239%7D%29=b%5 CRightarrow&space;%5Ctan&space;%28b%29=%5Cfrac%7B1%7D%7B239%7D %5C%5C&space;4a-b=%5Cfrac%7B%5Cpi&space;%7D%7B4%7D%5C%5C&space;%5Ctan&space;%284a-b%29=1%5CRightarrow&space;%5Ctan&space;%284a%29-%5Ctan&space;%28b%29=1+%5Ctan&space;%284a%29%5Ctan&space;%28a%29%5C% 5C

انشالله تا اینجا که مشکلی نیست

[ برای مشاهده لینک ، لطفا با نام کاربری خود وارد شوید یا ثبت نام کنید ] 9%7D%7B1-%5Ctan&space;%5E2%28a%29%7D=%5Cfrac%7B2/5%7D%7B1-1/25%7D=%5Cfrac%7B5%7D%7B12%7D%5C%5C&space;%5Ctan&space;%284a%29 =%5Cfrac%7B2%5Ctan&space;%282a%29%7D%7B1-%5Ctan&space;%5E2%282a%29%7D=%5Cfrac%7B120%7D%7B119%7D%5 C%5C

این هم از این

[ برای مشاهده لینک ، لطفا با نام کاربری خود وارد شوید یا ثبت نام کنید ] 5C&space;%5Cfrac%7B120%7D%7B119%7D-%5Cfrac%7B1%7D%7B239%7D=1+%5Cfrac%7B120%7D%7B119%7 D%5Cfrac%7B1%7D%7B239%7D%5C%5C&space;%5Cfrac%7B28561%7D% 7B28441%7D=%5Cfrac%7B28561%7D%7B28441%7D

برا سوالی که صفحه قبل گذاشتم کسی نتونست جوابی پیدا کنه؟

davy jones
26-02-2012, 22:21
با سلام .

ممنون از پاسخ همه اساتيد .

ببخشيد فرض كنيم كه هر سه تا نقطه داخل يه خط باشند به طوري كه بعد از درست كردن بردار ab‌ ac bc .....اندازه بردار ها طوري بشه كه جمع دو تاش از اون يكي بيشتر بشه . در اين صورت كه نميشه گفت ميشه با اين سه نقطه مثلث درست كرد با وجود اين كه در نا مساوي هم صدق كرده . ممكنه اين جوري اتفاق بيفته ؟


================================




بله صحبت شما درسته فکر نمیکنم حرف حمید جان درست باشه.


در صورتی که سه نقطه روی یک خط باشن اونوقت یکی از نامساویها تبدیل به مساوی میشه. مثلا اگه نقطه ی b نقطه ی وسطی فرض بشه، واضحه که ab+bc>ac دیگه اتفاق نمی افته و درست نیست چون در اونصورت داریم: ab+bc=ac. باز هم نظر شما محترم ... ولی بنده به جواب مطمئنم.


موفق باشین.
90/12/7

skyzare
26-02-2012, 22:46
با سلام .



اتحاد مثلثاتی حل کردم گفتم بزارمش ببینم نمیشه از این کوتاه تر به جواب رسید



نميدونم بشه از اين كوتاه تر رفت يا نه . ولي جالب حلش كرده بوديد .


در صورتی که سه نقطه روی یک خط باشن اونوقت یکی از نامساویها تبدیل به مساوی میشه. مثلا اگه نقطه ی b نقطه ی وسطی فرض بشه، واضحه که ab+bc>ac دیگه اتفاق نمی افته و درست نیست چون در اونصورت داریم: ab+bc=ac. باز هم نظر شما محترم ... ولی بنده به جواب مطمئنم.




بله حق با شماست شرمنده من اشتباه گفتم .



================================================== ====


اساتيد ببخشيد ميشه گفت اكه سه نقطه داخل يه خط باشند در واقع زاويه اي كه از درست كردن دو بردار با اين سه نقطه داريم بايد صفر باشه . درسته ؟ مثلا با سه نقطه دو تا بردار ab‌ و bc‌ رو درست كنيم . بعد اين دو تا بردار رو داخل هم كراس كنيم اگه جوابش صفر شد يعني در واقع زاوبه بينشون صفر بوده . پس سه نقطه روي يه خط هست نميشه مثلث باهاش درست كرد . اين جوري ميشه گفت ؟


================================================== ====

pinokiu
26-02-2012, 23:19
با سلام .




نميدونم بشه از اين كوتاه تر رفت يا نه . ولي جالب حلش كرده بوديد .





بله حق با شماست شرمنده من اشتباه گفتم .



================================================== ====


اساتيد ببخشيد ميشه گفت اكه سه نقطه داخل يه خط باشند در واقع زاويه اي كه از درست كردن دو بردار با اين سه نقطه داريم بايد صفر باشه . درسته ؟ مثلا با سه نقطه دو تا بردار ab‌ و bc‌ رو درست كنيم . بعد اين دو تا بردار رو داخل هم كراس كنيم اگه جوابش صفر شد يعني در واقع زاوبه بينشون صفر بوده . پس سه نقطه روي يه خط هست نميشه مثلث باهاش درست كرد . اين جوري ميشه گفت ؟


================================================== ====

بله کاملا درسته

hts1369
27-02-2012, 16:47
گفته این اتحاد رو ثابت کنید.
[ برای مشاهده لینک ، لطفا با نام کاربری خود وارد شوید یا ثبت نام کنید ] 7D%29=%5Carccos&space;%28%5Cfrac%7Bb+a%5Ccos&space;%5Ctheta&space;%7 D%7Ba+b%5Ccos&space;%5Ctheta&space;%7D%29

اول بگم که من موقع گذاشتن سوال جواب رو بلد نبودم تو این دو سه روز فکر کردم!!! و به این راه حل رسیدم

[ برای مشاهده لینک ، لطفا با نام کاربری خود وارد شوید یا ثبت نام کنید ] B2%7D%29%29=%5Carccos&space;%28%5Cfrac%7Bb+a%5Ccos&space;%28%5 Ctheta&space;%29%7D%7Ba+b%5Ccos&space;%28%5Ctheta&space;%29%7D%29%5C %5C&space;%5Carctan&space;%28%5Csqrt%7B%5Cfrac%7Ba-b%7D%7Ba+b%7D%7D%5Ctan&space;%28%5Cfrac%7B%5Ctheta&space;%7D%7 B2%7D%29%29=z%5CRightarrow&space;%5Ctan&space;%28z%29=%5Csqrt% 7B%5Cfrac%7Ba-b%7D%7Ba+b%7D%7D%5Ctan&space;%28%5Cfrac%7B%5Ctheta&space;%7D%7 B2%7D%29%5C%5C&space;%5Carccos&space;%28%5Cfrac%7Bb+a%5Ccos&space;%2 8%5Ctheta&space;%29%7D%7Ba+b%5Ccos&space;%28%5Ctheta&space;%29%7D%29 =y%5CRightarrow&space;%5Ccos&space;%28y%29=%5Cfrac%7Bb+a%5Ccos &space;%28%5Ctheta&space;%29%7D%7Ba+b%5Ccos&space;%28%5Ctheta&space;%29%7D %5C%5C&space;2z-y=0%5CRightarrow&space;%5Csin&space;%282z-y%29=0%5CRightarrow&space;%5Csin&space;%282z%29%5Ccos&space;%28y%29-%5Csin&space;%28y%29%5Ccos&space;%282z%29=0%5CRightarrow&space;%5Csi n&space;%282z%29%5Ccos&space;%28y%29=%5Csin&space;%28y%29%5Ccos&space;%282 z%29%5CRightarrow&space;%5Ctan&space;%282z%29=%5Ctan&space;%28y%29%5 C%5C&space;%5Ctan&space;%282z%29=%5Cfrac%7B2%5Ctan&space;%28z%29%7D% 7B1-%5Ctan&space;%5E2%28z%29%7D=%5Cfrac%7B2%5Csqrt%7B%5Cfrac %7Ba-b%7D%7Ba+b%7D%5Ctan&space;%28%5Cfrac%7B%5Ctheta&space;%7D%7B2% 7D%29%7D%7D%7B1-%5Cfrac%7Ba-b%7D%7Ba+b%7D%5Ctan&space;%5E2%28%5Cfrac%7B%5Ctheta&space;%7D% 7B2%7D%29%7D=%5Cfrac%7B2%5Csqrt%7Ba%5E2-b%5E2%7D%5Ctan&space;%28%5Cfrac%7B%5Ctheta&space;%7D%7B2%7D%29 %7D%7Ba+b-%28a-b%29%5Ctan&space;%5E2%28%5Cfrac%7B%5Ctheta&space;%7D%7B2%7D%29 %7D%5C%5C
بريم سراغ مخرج کسر(تو خط اول این قسمت یه دونه b اضافه گذاشتم-اشتباه تایپی هست)

[ برای مشاهده لینک ، لطفا با نام کاربری خود وارد شوید یا ثبت نام کنید ] =a+b-a%5Ctan&space;%5E2%28%5Cfrac%7B%5Ctheta&space;%7D%7B2%7D%29+b+ b%5Ctan&space;%5E2%28%5Cfrac%7B%5Ctheta&space;%7D%7B2%7D%29%5C %5C&space;a%281-%5Ctan&space;%5E2%28%5Cfrac%7B%5Ctheta&space;%7D%7B2%7D%29%29+ b%281+%5Ctan&space;%5E2%5Cfrac%7B%5Ctheta&space;%7D%7B2%7D%29% 5C%5C&space;a%28%5Cfrac%7B%5Ccos&space;%28%5Ctheta&space;%29%7D%7B%5 Ccos&space;%5E2%28%5Cfrac%7B%5Ctheta&space;%7D%7B2%7D%29%7D%29 +b%5Cfrac%7B1%7D%7B%5Ccos&space;%5E2%5Cfrac%7B%5Ctheta&space;% 7D%7B2%7D%7D=%5Cfrac%7Bb+a%5Ccos&space;%28%5Ctheta&space;%29%7 D%7B%5Ccos&space;%5E2%5Cfrac%7B%5Ctheta&space;%7D%7B2%7D%7D
با جاگذاری در کسر اصلی
[ برای مشاهده لینک ، لطفا با نام کاربری خود وارد شوید یا ثبت نام کنید ] %5Cfrac%7Bb+a%5Ccos&space;%28%5Ctheta&space;%29%7D%7B%5Ccos&space;%5 E2%5Cfrac%7B%5Ctheta&space;%7D%7B2%7D%7D%7D=%5Cfrac%7B2% 5Csqrt%7Ba%5E2-b%5E2%7D%5Ctan&space;%5Cfrac%7B%5Ctheta&space;%7D%7B2%7D%5Ccos &space;%5E2%5Cfrac%7B%5Ctheta&space;%7D%7B2%7D%7D%7Bb+a%5Ccos&space; %28%5Ctheta&space;%29%7D%5CRightarrow&space;%5Ctan&space;%282z%29=%5 Cfrac%7B%5Csqrt%7Ba%5E2-b%5E2%7D%5Csin&space;%28%5Ctheta&space;%29%7D%7Bb+a%5Ccos&space;%28% 5Ctheta&space;%29%7D
حالا بريم سراغ طرف دوم تساوي
[ برای مشاهده لینک ، لطفا با نام کاربری خود وارد شوید یا ثبت نام کنید ] Ccos&space;%5E2%28y%29%7D%5CRightarrow&space;%5Ctan&space;%5E2%28y%2 9=%5Cfrac%7B1%7D%7B%5Ccos&space;%5E2%28y%29%7D-1%5C%5C&space;%5Ctan&space;%5E2%28y%29=%5Cfrac%7Ba+b%5Ccos&space;%28 %5Ctheta%29%5E2%7D%7Bb+a%5Ccos&space;%28%5Ctheta&space;%29%5E2 %7D-1=%5Cfrac%7B%28a%5E2-b%5E2%29%5Csin&space;%5E2%28%5Ctheta&space;%29%7D%7B%28b+a%5Cc os&space;%28%5Ctheta&space;%29%29%5E2%7D%5C%5C&space;%5Ctan&space;%28y%29= %5Cfrac%7B%5Csqrt%7Ba%5E2-b%5E2%7D%5Csin&space;%28%5Ctheta&space;%29%7D%7Bb+a%5Ccos&space;%28% 5Ctheta&space;%29%7D
که دو طرف با هم برابر هستن.

skyzare
29-02-2012, 19:41
با سلام .

ببخشيد اساتيد اگه بخوام معادله صفحه اي رو بنويسم كه از يه نقطه مثلا A‌ ميگذره و شامل فصل مشترك دو صفحه هم باشه . بايد چه جوري حلش كنم ؟

مثلا : معادله صفحه اي كه از نقطه a(1 2 3 ) k ميگذرد را پيدا كنيد كه شامل فصل مشترك صفحه هاي x+y=2 و y-z=1 باشد .

با تشكر :20:

ashjaee
29-02-2012, 20:58
با سلام .

ببخشيد اساتيد اگه بخوام معادله صفحه اي رو بنويسم كه از يه نقطه مثلا A‌ ميگذره و شامل فصل مشترك دو صفحه هم باشه . بايد چه جوري حلش كنم ؟

مثلا : معادله صفحه اي كه از نقطه a(1 2 3 ) k ميگذرد را پيدا كنيد كه شامل فصل مشترك صفحه هاي x+y=2 و y-z=1 باشد .

با تشكر :20:

سلام دوست عزیز.
ابتدا راحت معادله فصل مشترک حساب میشه:
y = 1 + z = 2 - x
حالا بردار موازی این خط رو که داریم :
(1,1,1-)
یک نقطه هم از این فصل مشترک پیدا میکنیم مثل 2 و 0 و -1 با نقطه a بردارش رو مینویسیم ضرب خارجی در اون بردار دیگه و ...

M . Sayid
29-02-2012, 21:52
چرا مجموعه اعداد حقیقی و طبیعی هم ارز نیستند ؟

davy jones
01-03-2012, 13:33
چرا مجموعه اعداد حقیقی و طبیعی هم ارز نیستند ؟
سلام.

بنده 2 دلیل میارم ولی کاملا مطمئن نیستم که حرفم درست باشه:

1- از روی هر عضو مجموعه ی اعداد طبیعی میشه عضو بعدی رو ساخت که این خاصیت در مجموعه ی اعداد حقیقی وجود نداره.
2- مجموعه ی اعداد طبیعی اصطلاحا خوش ترتیب هستند. یعنی به ازای هر زیر مجموعه ی دلخواه ازاین مجموعه، میتوان برایش یک عضو ابتدا پیدا کرد که از همه ی اعضا کوچکتر هستش. اما این خاصیت هم در اعداد حقیقی وجود نداره و مجموعه ی R در اصل خوش ترتیبی صدق نمیکنه.

موفق باشین.
90/12/11

M . Sayid
01-03-2012, 14:17
سلام.

بنده 2 دلیل میارم ولی کاملا مطمئن نیستم که حرفم درست باشه:

1- از روی هر عضو مجموعه ی اعداد طبیعی میشه عضو بعدی رو ساخت که این خاصیت در مجموعه ی اعداد حقیقی وجود نداره.
2- مجموعه ی اعداد طبیعی اصطلاحا خوش ترتیب هستند. یعنی به ازای هر زیر مجموعه ی دلخواه ازاین مجموعه، میتوان برایش یک عضو ابتدا پیدا کرد که از همه ی اعضا کوچکتر هستش. اما این خاصیت هم در اعداد حقیقی وجود نداره و مجموعه ی R در اصل خوش ترتیبی صدق نمیکنه.

موفق باشین.
90/12/11
خواص این مجموعه ها خوب یادم نیست ولی مطئنم که
اعداد حسابی و حقیقی هم ارزند

به خاطر وجود صفره ؟ چرا ؟

davy jones
01-03-2012, 15:04
خواص این مجموعه ها خوب یادم نیست ولی مطئنم که
اعداد حسابی و حقیقی هم ارزند

به خاطر وجود صفره ؟ چرا ؟

راستش نمیدونم. باید این مباحث رو یه بار دیگه یه نگاه بندازم و از چند نفر از دوستانم سوال کنم.
هر وقت فهمیدم میام و همین پست رو ویرایش میکنم.


موفق باشین.
90/12/11

lebesgue
02-03-2012, 18:12
چرا مجموعه اعداد حقيقي و طبيعي هم ارز نيستند ؟

من نمی دانم منظور شما از هم ارز بودن چيست، اما در مورد اين دو مجموعه ميتوان گفت هيچ تناظر يک به يکي ميان اين دو مجموعه وجود ندارد. (بطور معادل، کارديناليتي آنها يکسان نيست - مجموعه اعداد طبيعي شمارا، اما مجموعه اعداد حقيقي ناشمارا است.) براي اثبات اين قضيه، استدلال ساده اي به نام روش قطري کانتور وجود دارد. در ضمن، مجموعه اعداد حسابي، صحيح و گويا، همگي داراي تناظر يک به يک با مجموعه اعداد طبيعي بوده و شمارا هستند.

skyzare
03-03-2012, 17:18
با سلام .

اساتید میخواستم معادله عمود مشترک این دو خط متنافر رو به دست بیارم . باید چی کار کرد ؟


[ برای مشاهده لینک ، لطفا با نام کاربری خود وارد شوید یا ثبت نام کنید ]{x-1}{2}=\frac{y+2}{2}=\frac{z-1}{1}



[ برای مشاهده لینک ، لطفا با نام کاربری خود وارد شوید یا ثبت نام کنید ]{x}{5}=\frac{y-3&space;}{2}=\frac{z+1}{1}



باتشکر .:20:

pinokiu
03-03-2012, 20:24
با سلام .

اساتید میخواستم معادله عمود مشترک این دو خط متنافر رو به دست بیارم . باید چی کار کرد ؟


[ برای مشاهده لینک ، لطفا با نام کاربری خود وارد شوید یا ثبت نام کنید ]



[ برای مشاهده لینک ، لطفا با نام کاربری خود وارد شوید یا ثبت نام کنید ]



باتشکر .:20:

سلام دوست عزیز
ابتدا معادله دو نقطه فرضی رو روی دو خط بدست می آوریم
1) بر حسب a
{ a,a+1,(a+1)/2}
2) بر جسب b
{b,2b+15,b-6}
حالا بین این دو خط یک بردار تشکیل می دهیم
AB=(b-a,2b+14-a,b-5.5-.5a)
بردار AB باید بر دو خط عمود باشه
از ضرب داخلی بردار AB در 1 و 2 دو معادله و دو مجهول a و b بدست می آیند که در نتیجه AB بدست می آید

skyzare
03-03-2012, 21:14
سلام دوست عزیز
ابتدا معادله دو نقطه فرضی رو روی دو خط بدست می آوریم
1) بر حسب a
{ a,a+1,(a+1)/2}
2) بر جسب b
{b,2b+15,b-6}
حالا بین این دو خط یک بردار تشکیل می دهیم
AB=(b-a,2b+14-a,b-5.5-.5a)
بردار AB باید بر دو خط عمود باشه
از ضرب داخلی بردار AB در 1 و 2 دو معادله و دو مجهول a و b بدست می آیند که در نتیجه AB بدست می آید

با سلام .

با تشکر از پاسخ شما .

ببخشید من متوجه نشدم :41:

1- خوب اول دو تا نقطه فرضی روی دو خط در نظر میگیریم . ولی الان این چیزی که شما نوشتید ( بر حسب a و b ) که روی خط نیست . این ها رو بر چه مبنایی نوشتید ؟

2-خوب حالا با توجه به گفته شما بر حسب این دو تا نقطه به بردار تشکیل میدیم . این بردار برای این که بر دو تا خط عمود باشه یاید با بردار هادی خط ها ضرب داخلی بشه درسته این جوری ؟

کلا غاطی کردم . :31: شرمنده میشه لطف کنید حلش کنید . :41:

pinokiu
03-03-2012, 22:01
با سلام .

با تشکر از پاسخ شما .

ببخشید من متوجه نشدم :41:

1- خوب اول دو تا نقطه فرضی روی دو خط در نظر میگیریم . ولی الان این چیزی که شما نوشتید ( بر حسب a و b ) که روی خط نیست . این ها رو بر چه مبنایی نوشتید ؟

2-خوب حالا با توجه به گفته شما بر حسب این دو تا نقطه به بردار تشکیل میدیم . این بردار برای این که بر دو تا خط عمود باشه یاید با بردار هادی خط ها ضرب داخلی بشه درسته این جوری ؟

کلا غاطی کردم . :31: شرمنده میشه لطف کنید حلش کنید . :41:
سلام دوست عزیز
راستش خودم هم فهمیدم خیلی بد توضیح دادم
ببین اول باید معادلات پارامتری دو خط رو تشکیل بدیم
مثلا پارامتر a رو انتخاب کردم
a رو به جای x قرار دادم و بقیه پارامترها رو بر حسب a بدست آوردم
این معنیش اینه که اگر a رو متغییر فرض کنیم با تغییرش خط ما رو در فضا تعریف می کنه
که تو قسمت 1 نوشتم
این درواقع مهادله خط هست که به این شکل در آمده
اگر a رو یک مقدار مشخص فرض کنیم این معادله پارامتری به یک نقطه تبدیل می شه
برای قسمت 2 هم همینطور
حالا 2 نقطه فرضی هست که در معادلات صدق می کنه
بین این دو خط بردار تعریف می کنیم
خطوط اولیه رو هم که به صورت پارامتریک در آوردیم
حالا باید ضرب داخلی برداری که ساختیم در هریک از خطوط مساوی صفر بشه
دو معادله بدست میاد
دو مجهول هم بیشتر نداریم a و b
چون معادلات اصلی رو برحسب a و b نوشتیم
برای این مثال یک بار AB رو در 1 ضرب داخلی می کنیم و یک بار در 2
که هر دو باید صفر باشند

mjorh
04-03-2012, 13:53
[تو این سئوال باید از عبارت اول نسبت به y مشتق گرفته شه ،عبارت دوم نسبت به x

خودم مشتق می گیرم ولی خیلی شک دارم ...

[ برای مشاهده لینک ، لطفا با نام کاربری خود وارد شوید یا ثبت نام کنید ]

lebesgue
04-03-2012, 19:18
ولی در رابطه با سری که گفتید من دقیقا متن داخل کتاب رو نوشتم حتی اثباتش هم کرده بود . ولی 0^0 اصلا تعریف شده هست ؟


بحث در مورد تعریف صفر به توان صفر و اینکه برابر چه مقداری تعریف شود، بحثی تاریخی میان ریاضیدانان است. گاهاً برای سادگی روابط، صفر به توان صفر برابر یک فرض می شود. مثلاً اتحاد های زیر در x=0 برقرار نیستند، مگر اینکه تعریف کنید [ برای مشاهده لینک ، لطفا با نام کاربری خود وارد شوید یا ثبت نام کنید ]^0=1.


[ برای مشاهده لینک ، لطفا با نام کاربری خود وارد شوید یا ثبت نام کنید ]

[ برای مشاهده لینک ، لطفا با نام کاربری خود وارد شوید یا ثبت نام کنید ]

[ برای مشاهده لینک ، لطفا با نام کاربری خود وارد شوید یا ثبت نام کنید ]


برای اطلاعات بیشتر در این مورد، لینک زیر را ببینید (بخش Zero to the zero power)ا:
[ برای مشاهده لینک ، لطفا با نام کاربری خود وارد شوید یا ثبت نام کنید ]_to_the_zero_power

lebesgue
04-03-2012, 19:58
براي اثبات اين قضيه، استدلال ساده اي به نام روش قطري کانتور وجود دارد.

از آنجا که ظاهراً سوال دوست ما در مورد همین قضیه بوده، به طور مختصر در مورد آن توضیح می دهم.

برای اینکه نشان دهیم هیچ تناظر یک به یکی میان مجموعه اعداد حقیقی [ برای مشاهده لینک ، لطفا با نام کاربری خود وارد شوید یا ثبت نام کنید ]{R} و مجموعه اعداد طبیعی [ برای مشاهده لینک ، لطفا با نام کاربری خود وارد شوید یا ثبت نام کنید ]{N} وجود ندارد، کافی است نشان دهیم هیچ تناظر یک به یکی میان [ برای مشاهده لینک ، لطفا با نام کاربری خود وارد شوید یا ثبت نام کنید ]{N} و بازه (0,1) وجود ندارد.

(وجود تناظر یک به یک میان دو مجموعه، به معنای وجود تابعی یک به یک و پوشا از یکی به روی دیگری است.)

تمام اعداد حقیقی، بسط اعشاری یکتایی دارند، به شرطی که از دو فرم 0 نامتناهی و 9 نامتناهی، تنها یکی را به رسمیت بشناسیم؛ مثلاً از دو نمایش ممکن برای 1/2 یعنی [ برای مشاهده لینک ، لطفا با نام کاربری خود وارد شوید یا ثبت نام کنید ] و [ برای مشاهده لینک ، لطفا با نام کاربری خود وارد شوید یا ثبت نام کنید ] یکی را به عنوان بسط اعشاری 1/2 بپذیریم.

به برهان خلف، فرض کنید تناظری یک به یک میان بازه (0,1) و [ برای مشاهده لینک ، لطفا با نام کاربری خود وارد شوید یا ثبت نام کنید ]{N} وجود دارد. در اين صورت ميتوان این تناظر را مانند نمونه زير نمايش داد:



[ برای مشاهده لینک ، لطفا با نام کاربری خود وارد شوید یا ثبت نام کنید ] 91409327859729401240%5Ccdots%20%5C%5C%202%5Cleftri ghtarrow0.0333333333333333333333333333333%5Ccdots% 20%5C%5C%203%20%5Cleftrightarrow%200.1431008484881 943900214052345721%5Ccdots%20%5C%5C%204%20%5Cleftr ightarrow%200.8300000000000000000000000000000%5Ccd ots%20%5C%5C%20%5Cvdots

حال يک عدد حقيقي را چنين انتخاب مي کنيم: رقم اول پس از اعشارش را رقمي متفاوت با رقم اول پس از اعشار اولين عدد در ليست بالا (يعني متفاوت با 7) انتخاب مي کنيم. رقم دوم را رقمي متفاوت با رقم دوم دومين عدد در ليست بالا (يعني متفاوت با 3) انتخاب مي کنيم و به همين ترتيب، رقم n ام را متفاوت با رقم n ام n امين عدد.

اين عدد جديد، با اولين عدد متفاوت است، چون دستکم يک رقمش فرق دارد، به همين ترتيب با دومين، سومين و کلاً تمام اعداد موجود در ليست متفاوت است. در نتيجه يک عدد حقيقي در بازه (0,1) وجود دارد که با هيچ عدد طبيعي متناظر نشده و اين در تناقض با فرض است. بنابراین فرض وجود تناظر يک به يک ميان بازه (0,1) و [ برای مشاهده لینک ، لطفا با نام کاربری خود وارد شوید یا ثبت نام کنید ]{N} درست نيست و در نهایت بنا به آنچه قبلاً گفته شد، هیچ تناظر یک به یکی میان و [ برای مشاهده لینک ، لطفا با نام کاربری خود وارد شوید یا ثبت نام کنید ]{R} و [ برای مشاهده لینک ، لطفا با نام کاربری خود وارد شوید یا ثبت نام کنید ]{N} وجود ندارد.

به طور حسی میتوان دید با وجود اینکه مجموعه [ برای مشاهده لینک ، لطفا با نام کاربری خود وارد شوید یا ثبت نام کنید ]{R} و مجموعه [ برای مشاهده لینک ، لطفا با نام کاربری خود وارد شوید یا ثبت نام کنید ]{N} هر دو بینهایت عضو دارند، اما اعضای مجموعه [ برای مشاهده لینک ، لطفا با نام کاربری خود وارد شوید یا ثبت نام کنید ]{R} بسیار بیشتر هستند. کاردینالیتی معیاری برای سنجش تعداد اعضای مجموعه ها است که برای مجموعه های دارای بینهایت عضو نیز تعریف می شود. می توان نشان داد که مجموعه اعداد فرد (زوج)، مجموعه اعداد صحیح و مجموعه اعداد گویا همگی دارای تناظر یک به یک با مجموعه اعداد طبیعی هستند. مجموعه هایی (و فقط مجموعه هایی) که دارای تناظر یک به یک با یکدیگر باشند، کاردینالیتی یکسان دارند. از آنجا که به مجموعه اعداد طبیعی، کاردینالیتی [ برای مشاهده لینک ، لطفا با نام کاربری خود وارد شوید یا ثبت نام کنید ] (الف صفر) اختصاص داده می شود، کاردینالیتی مجموعه های نام برده، [ برای مشاهده لینک ، لطفا با نام کاربری خود وارد شوید یا ثبت نام کنید ] است. یک دسته دیگر از مجموعه ها وجود دارند که دارای تناظر یک به یک با [ برای مشاهده لینک ، لطفا با نام کاربری خود وارد شوید یا ثبت نام کنید ]{R} هستند. به این دسته مجموعه ها، کاردینالیتی c اختصاص داده می شود. از آنجا که می توان نشان داد [ برای مشاهده لینک ، لطفا با نام کاربری خود وارد شوید یا ثبت نام کنید ]{R} دارای تناظر یک به یک با مجموعه توانی [ برای مشاهده لینک ، لطفا با نام کاربری خود وارد شوید یا ثبت نام کنید ]{N} است، گاهاً به طور سمبولیک نوشته می شود [ برای مشاهده لینک ، لطفا با نام کاربری خود وارد شوید یا ثبت نام کنید ]^{%5Caleph_0}.

skyzare
04-03-2012, 20:01
[تو این سئوال باید از عبارت اول نسبت به y مشتق گرفته شه ،عبارت دوم نسبت به x

خودم مشتق می گیرم ولی خیلی شک دارم ...

[ برای مشاهده لینک ، لطفا با نام کاربری خود وارد شوید یا ثبت نام کنید ]


با سلام .



[ برای مشاهده لینک ، لطفا با نام کاربری خود وارد شوید یا ثبت نام کنید ]{d(\frac{-1}{y}sin(\frac{x}{y}))}{dy}=\frac{1}{y^2}\:&space;\:&space;sin (\frac{x}{y})+\frac{x}{y^3}\:&space;\:&space;cos(\frac{x}{y})





[ برای مشاهده لینک ، لطفا با نام کاربری خود وارد شوید یا ثبت نام کنید ]{d(\frac{x}{y^2})sin(\frac{x }{y})}{dx}=\frac{1}{y^2}sin(\frac{x}{y})+\frac{x}{ y^3}cos(\frac{x}{y})


=======================================


شرط معادله کامل برقرار هست میتونید از راه معادله کامل حل کنبد :31:

mjorh
04-03-2012, 23:27
با سلام .



[ برای مشاهده لینک ، لطفا با نام کاربری خود وارد شوید یا ثبت نام کنید ]{d(\frac{-1}{y}sin(\frac{x}{y}))}{dy}=\frac{1}{y^2}\:&space;\:&space;sin (\frac{x}{y})+\frac{x}{y^3}\:&space;\:&space;cos(\frac{x}{y})





[ برای مشاهده لینک ، لطفا با نام کاربری خود وارد شوید یا ثبت نام کنید ]{d(\frac{x}{y^2})sin(\frac{x }{y})}{dx}=\frac{1}{y^2}sin(\frac{x}{y})+\frac{x}{ y^3}cos(\frac{x}{y})


=======================================


شرط معادله کامل برقرار هست میتونید از راه معادله کامل حل کنبد :31:



مرسی
الان اینو از راهه ،مشتق اولی ضربدر خود دومی و ....حل کردید ؟
اگه اینجوریه چرا آخری رو ضرب نکردید ؟ضربدر l-1/y
(هواسم نبود شمام بلدید:31:(معادله کامل رو))

skyzare
04-03-2012, 23:33
مرسی
الان اینو از راهه ،مشتق اولی ضربدر خود دومی و ....حل کردید ؟
اگه اینجوریه چرا آخری رو ضرب نکردید ؟ضربدر l-1/y
(هواسم نبود شمام بلدید:31:(معادله کامل رو))

با سلام .

بله همون جوری حل کردم .

منظورتون از اخری کدام هست ؟

اگه منظورتون مورد دومی هست( قسمت دومش ) که ضرب شده البته دیگه منفی نداره چون مشتق نسبت به x داریم میگیریم .

skyzare
04-03-2012, 23:41
با سلام .

اساتید من یه سوال داشتم .

نشان دهید که ماتریس مربع A را میتوان به صورت مجموع دو ماتریس یکی متقارن و دیگری پادمتقارن نوشت .

باتشکر .:20:

mjorh
04-03-2012, 23:52
با سلام .

بله همون جوری حل کردم .

منظورتون از اخری کدام هست ؟

اگه منظورتون مورد دومی هست( قسمت دومش ) که ضرب شده البته دیگه منفی نداره چون مشتق نسبت به x داریم میگیریم .

آها گرفتم مرسی...

---------- Post added at 11:52 PM ---------- Previous post was at 11:47 PM ----------

یه سئوال از معادلات خطی دارم ...خودمو کشتم ولی نتونسم حلش کنم :41:

[ برای مشاهده لینک ، لطفا با نام کاربری خود وارد شوید یا ثبت نام کنید ]

'گفته میتونید به جای x ,y بزارید و برعکس...

davy jones
05-03-2012, 08:37
با سلام .

اساتید من یه سوال داشتم .

نشان دهید که ماتریس مربع A را میتوان به صورت مجموع دو ماتریس یکی متقارن و دیگری پادمتقارن نوشت .

باتشکر .:20:
سلام.

دقت کنید که به ازای هر ماتریس مربعی دلخواه مانند A اگر خود ماتریس A رو با ترانهاده ی خودش جمع کنیم، همواره یک ماتریس متقارن خواهیم داشت. (چرا؟)
همچنین اگه ماتریس A رو منهای ترانهاده ی خودش کنیم، همواره یک ماتریس پاد متقارن خواهیم داشت. (چرا؟)
و باز همچنین اگر همه ی درایه های یک ماتریس متقارن یا یک ماتریس پاد متقارن رو در یک عدد حقیقی مثبت ضرب کنیم، تاثیری در متقارن یا پاد متقارن بودن آن مانریس نمیگذارد. (چرا؟)

با این سه مقدمه میتوان گفت که هر ماتریس مربعی مانند A رو میتوان به صورت زیر نوشت:


[ برای مشاهده لینک ، لطفا با نام کاربری خود وارد شوید یا ثبت نام کنید ] 5Cfrac%7B1%7D%7B2%7D%28A+A%5E%7BT%7D%29%7D+%7B%5Cc olor%7BBlue%7D&space;%5Cfrac%7B1%7D%7B2%7D%28A-A%5E%7BT%7D%29%7D


که قسمت قرمز رنگ طبق گفته های بالا همواره یک ماتریس متقارن است و قسمت آبی رنگ همواره یک مانریس پادمتقارن است.


===========================




یه سئوال از معادلات خطی دارم ...خودمو کشتم ولی نتونسم حلش کنم [ برای مشاهده لینک ، لطفا با نام کاربری خود وارد شوید یا ثبت نام کنید ]

[ برای مشاهده لینک ، لطفا با نام کاربری خود وارد شوید یا ثبت نام کنید ]

'گفته میتونید به جای x ,y بزارید و برعکس...


سلام.

این معادله خطی نیست. هر کی به شما گفته که این معادله خطیه سر کارتون گذاشته.

[ برای مشاهده لینک ، لطفا با نام کاربری خود وارد شوید یا ثبت نام کنید ]^2e^y ([ برای مشاهده لینک ، لطفا با نام کاربری خود وارد شوید یا ثبت نام کنید ])



موفق باشین.
90/12/15

ashjaee
05-03-2012, 09:09
آها گرفتم مرسی...

---------- Post added at 11:52 PM ---------- Previous post was at 11:47 PM ----------

یه سئوال از معادلات خطی دارم ...خودمو کشتم ولی نتونسم حلش کنم :41:

[ برای مشاهده لینک ، لطفا با نام کاربری خود وارد شوید یا ثبت نام کنید ]

'گفته میتونید به جای x ,y بزارید و برعکس...
سلام.
خلاصه شده معادله بالا این میشه:


[ برای مشاهده لینک ، لطفا با نام کاربری خود وارد شوید یا ثبت نام کنید ]

که می تونیم اون رو اینطوری هم بنویسیم:

[ برای مشاهده لینک ، لطفا با نام کاربری خود وارد شوید یا ثبت نام کنید ]

همونطوری که جناب davy jones فرمودند این مهادله بر حسب y خطی نیست اما بر حسب x خطی هست پس داریم:

[ برای مشاهده لینک ، لطفا با نام کاربری خود وارد شوید یا ثبت نام کنید ]

و بالاخره:

[ برای مشاهده لینک ، لطفا با نام کاربری خود وارد شوید یا ثبت نام کنید ]

و این هم جواب عمومی:

[ برای مشاهده لینک ، لطفا با نام کاربری خود وارد شوید یا ثبت نام کنید ]

900190512
05-03-2012, 15:09
:biggrin: این سوال هم برای دوست داران تابع و حساب:
تمامی اعداد حقیقی چون a را بیابید که به ازای آن تابع غیر ثابت f از مجموعه اعداد حقیقی به خودش وجود داشته باشد که: (f(a(x+y ))=f(x)+f(y
حوصله حساب کردن ندارم بهت میگم خودت حساب کن

skyzare
05-03-2012, 15:28
با سلام .

با تشکر از پاسخ شما . :20:

بفرمایید این هم پاسخ چرا ها :



دقت کنید که به ازای هر ماتریس مربعی دلخواه مانند A اگر خود ماتریس A رو با ترانهاده ی خودش جمع کنیم، همواره یک ماتریس متقارن خواهیم داشت. (چرا؟)


فرض کنیم داشته باشیم :


[ برای مشاهده لینک ، لطفا با نام کاربری خود وارد شوید یا ثبت نام کنید ]'&space;\\\\&space;B'=(A+A')'&space;\\\\&space;B'=(A'+(A' )')=(A'+A)=(A+A')=B\:&space;\:&space;\rightarrow&space;B'=B




همچنین اگه ماتریس A رو منهای ترانهاده ی خودش کنیم، همواره یک ماتریس پاد متقارن خواهیم داشت. (چرا؟)



[ برای مشاهده لینک ، لطفا با نام کاربری خود وارد شوید یا ثبت نام کنید ]'&space;\\\\&space;B'=(A-A')'&space;\\\\&space;B'=(A'-(A')')=(A'-A)=-(A-A')=-B\:&space;\:&space;\rightarrow&space;B'=-B\:&space;\:\rightarrow&space;B=-B'





و باز همچنین اگر همه ی درایه های یک ماتریس متقارن یا یک ماتریس پاد متقارن رو در یک عدد حقیقی مثبت ضرب کنیم، تاثیری در متقارن یا پاد متقارن بودن آن مانریس نمیگذارد. (چرا؟)


خوب مشخصه تاثیر نداره .....:31: نمیدوم چه جوری بنویسم :31:

davy jones
05-03-2012, 15:43
با سلام .

با تشکر از پاسخ شما . :20:

بفرمایید این هم پاسخ چرا ها :



فرض کنیم داشته باشیم :


[ برای مشاهده لینک ، لطفا با نام کاربری خود وارد شوید یا ثبت نام کنید ] 27%29%27&space;%5C%5C%5C%5C&space;B%27=%28A%27+%28A%27%29%27%2 9=%28A%27+A%29=%28A+A%27%29=B%5C:&space;%5C:&space;%5Crightarr ow&space;B%27=B





[ برای مشاهده لینک ، لطفا با نام کاربری خود وارد شوید یا ثبت نام کنید ]





خوب مشخصه تاثیر نداره .....:31: نمیدوم چه جوری بنویسم :31:
سلام.

البته بهتر بود به جای علامت پرایم ( ' ) از علامت مخصوص ترانهاده استفاده میکردین. یعنی اینطوری:


[ برای مشاهده لینک ، لطفا با نام کاربری خود وارد شوید یا ثبت نام کنید ] T%7D%29%5E%7BT%7D=A%5E%7BT%7D+%28A%5E%7BT%7D%29%5E %7BT%7D=A%5E%7BT%7D+A=B



چون معمولا از پرایم در مجموعه ها و برای نشون دادن متمم یک زیرمجموعه تا رسیدن به مجموعه ی مرجع استفاده میشه.

هرچند که خیلی هم مهم نیست. مهم همون منظور اصلی بود که محقق شده بود.


موفق باشین.
90/12/15

mjorh
05-03-2012, 16:31
هر کی به شما گفته که این معادله خطیه سر کارتون گذاشته.

پس کتاب سر کارم گزاشته :31: من اشتباه کردم ،هواسم نبود نوشته معادله دیفرانسیل!
با روش که ashjaee گفتن باید رفت ، اما نمیدونم قاعده خاصی دار این عوض کردنه ایکس و وای یا نه ....!؟

skyzare
06-03-2012, 16:53
با سلام .

اساتید این رو ببینید :

الف : مجموعه [ برای مشاهده لینک ، لطفا با نام کاربری خود وارد شوید یا ثبت نام کنید ]{Bmatrix}&space;v_1,v_2,v_3&space;\end{Bmatrix } که در ان داریم :



[ برای مشاهده لینک ، لطفا با نام کاربری خود وارد شوید یا ثبت نام کنید ](1,1,1)\\\\&space;v_2=(-2,1,1)\\\\&space;v_3=(0,1,-1)



مستقل خطی است و یک مولد برای [ برای مشاهده لینک ، لطفا با نام کاربری خود وارد شوید یا ثبت نام کنید ]^3 می باشد .



ب : مجموعه [ برای مشاهده لینک ، لطفا با نام کاربری خود وارد شوید یا ثبت نام کنید ]{Bmatrix}&space;v_1,v_2&space;\end{Bmatrix} که در ان داریم :



[ برای مشاهده لینک ، لطفا با نام کاربری خود وارد شوید یا ثبت نام کنید ](1,1,1)\\\\&space;v_2=(1,2,0)\\\\


مستقل خطی است ولی یک مولد برای [ برای مشاهده لینک ، لطفا با نام کاربری خود وارد شوید یا ثبت نام کنید ]^3نیست



=====================================




حالا سوالم اینه . من نمیدونم چه جوری باید تشخیص بدم که مولد برای [ برای مشاهده لینک ، لطفا با نام کاربری خود وارد شوید یا ثبت نام کنید ]^3 هست .هر دو تاش که مستقل خطی هست . میدونم مستقل خطی بودن رو چه جوری تشخیص


بدم ولی این مولد رو نه . اصلا منظورش از مولد چیه ؟ :31:


با تشکر :20:

lebesgue
06-03-2012, 19:14
در بحث فضاهای برداری، مفهومی هست به نام پایه (Basis). احتمالاً اینجا منظور از مولد همین پایه است. تفاوت مجموعه اول و دوم در این است که در اولی، مجموعه همه ترکیب های خطی ممکن از بردارهای [ برای مشاهده لینک ، لطفا با نام کاربری خود وارد شوید یا ثبت نام کنید ]، برابر کل فضای [ برای مشاهده لینک ، لطفا با نام کاربری خود وارد شوید یا ثبت نام کنید ]{R}^3 است، یا به عبارت دیگر، ترکیب خطی بردارهای مجموعه اول میتوانند کل فضای [ برای مشاهده لینک ، لطفا با نام کاربری خود وارد شوید یا ثبت نام کنید ]{R}^3 را تولید کنند، اما در مورد مجموعه دوم چنین نیست.

در یک فضای برداری، به مجموعه ای از بردارهای مستقل که بتوانند کل فضا را تولید کنند، یک پایه برای آن فضا گفته می شود. بنا به قضایای جبر خطی، برای یک فضای برداری با بعد متناهی، همه پایه ها، تعداد اعضای یکسانی دارند و این تعداد، بعد آن فضا نامیده می شود. بنابراین برای یک فضای n بعدی، هیچ مجموعه ای با کمتر از n بردار، نمی تواند یک پایه برای فضا بوده و کل فضا را بسازد.

فضای برداری [ برای مشاهده لینک ، لطفا با نام کاربری خود وارد شوید یا ثبت نام کنید ]{R}^3 یک فضای سه بعدی است و یک پایه برای آن، مجموعه {(0,0,1),(0,1,0),(1,0,0)} است که پایه استاندارد نامیده می شود. همچنین بنا به آنچه در بالا آمد، هیچ مجموعه ای با کمتر از سه بردار نمی تواند کل فضا را بسازد.

lebesgue
06-03-2012, 19:21
از آنجا که مي توان نشان داد [ برای مشاهده لینک ، لطفا با نام کاربری خود وارد شوید یا ثبت نام کنید ]{R} داراي تناظر يک به يک با مجموعه تواني [ برای مشاهده لینک ، لطفا با نام کاربری خود وارد شوید یا ثبت نام کنید ]{N} است، گاهاً به طور سمبوليک نوشته مي شود [ برای مشاهده لینک ، لطفا با نام کاربری خود وارد شوید یا ثبت نام کنید ]^{%5Caleph_0}.

مسئله 1: نشان دهيد تناظري يک به يک ميان مجموعه اعداد حقيقي و مجموعه تواني اعداد طبيعي (مجموعه زيرمجموعه هاي [ برای مشاهده لینک ، لطفا با نام کاربری خود وارد شوید یا ثبت نام کنید ]{N}) وجود دارد.

مسئله 2: نشان دهيد براي هر m و n صحيح مثبت، تناظري يک به يک ميان مجموعه هاي [ برای مشاهده لینک ، لطفا با نام کاربری خود وارد شوید یا ثبت نام کنید ]{R}^m و [ برای مشاهده لینک ، لطفا با نام کاربری خود وارد شوید یا ثبت نام کنید ]{R}^n وجود دارد.

lovely_killer
06-03-2012, 20:46
سلام دوستان

من 3 تا سئوال جبر خطی دارم که هر کاری کردم نتونستم حلشون کنم.به هیچ کدوم از همکلاسی ها هم دسترسی ندارم :19:. اگه کسی بتونه تو حلشون کمکم کنه واقعا ممنونش میشم.:10:


برای مشاهده محتوا ، لطفا وارد شوید یا ثبت نام کنید

skyzare
06-03-2012, 22:14
در بحث فضاهای برداری، مفهومی هست به نام پایه (Basis). احتمالاً اینجا منظور از مولد همین پایه است. تفاوت مجموعه اول و دوم در این است که در اولی، مجموعه همه ترکیب های خطی ممکن از بردارهای [ برای مشاهده لینک ، لطفا با نام کاربری خود وارد شوید یا ثبت نام کنید ]، برابر کل فضای [ برای مشاهده لینک ، لطفا با نام کاربری خود وارد شوید یا ثبت نام کنید ]{R}^3 است، یا به عبارت دیگر، ترکیب خطی بردارهای مجموعه اول میتوانند کل فضای [ برای مشاهده لینک ، لطفا با نام کاربری خود وارد شوید یا ثبت نام کنید ]{R}^3 را تولید کنند، اما در مورد مجموعه دوم چنین نیست.

در یک فضای برداری، به مجموعه ای از بردارهای مستقل که بتوانند کل فضا را تولید کنند، یک پایه برای آن فضا گفته می شود. بنا به قضایای جبر خطی، برای یک فضای برداری با بعد متناهی، همه پایه ها، تعداد اعضای یکسانی دارند و این تعداد، بعد آن فضا نامیده می شود. بنابراین برای یک فضای n بعدی، هیچ مجموعه ای با کمتر از n بردار، نمی تواند یک پایه برای فضا بوده و کل فضا را بسازد.

فضای برداری [ برای مشاهده لینک ، لطفا با نام کاربری خود وارد شوید یا ثبت نام کنید ]{R}^3 یک فضای سه بعدی است و یک پایه برای آن، مجموعه {(0,0,1),(0,1,0),(1,0,0)} است که پایه استاندارد نامیده می شود. همچنین بنا به آنچه در بالا آمد، هیچ مجموعه ای با کمتر از سه بردار نمی تواند کل فضا را بسازد.

با سلام .

با تشکر از پاسخ شما . :20:

در مورد اون قسمت قرمز رنگ .

یعنی اگه ما یه فضای n بعدی مثلا [ برای مشاهده لینک ، لطفا با نام کاربری خود وارد شوید یا ثبت نام کنید ]^6 داشته یاشیم . طبق مطالب شما هر مجموعه ای که کمتر از 6 بردار

داشته باشه نمیتونه این فضا [ برای مشاهده لینک ، لطفا با نام کاربری خود وارد شوید یا ثبت نام کنید ]^6 را بسازه و در واقع پایه ای برا ی فضا محسوب نمیشه . درسته ؟


حالا میشه بگیم که :

1- اگه مجموعه ما دقیقا 6 بردار داشته باشه حتما یه پایه برای فضای [ برای مشاهده لینک ، لطفا با نام کاربری خود وارد شوید یا ثبت نام کنید ]^6 محسوب میشه ؟

2- یا اگه مجموعه ما بیشتر از 6 بردار باشه باز میشه با قاطیعت گفت که یه فضا برای [ برای مشاهده لینک ، لطفا با نام کاربری خود وارد شوید یا ثبت نام کنید ]^6 محسوب میشه ؟



==================================================


شرمنده من هنوز دقیقا متوجه نشدم از چه روش ریاضی میشه فهمید که مجموعه داده شده مولد ی برای فضای مورد نظر نیست . :41: :41:

skyzare
07-03-2012, 15:19
در بحث فضاهای برداری، مفهومی هست به نام پایه (Basis). احتمالاً اینجا منظور از مولد همین پایه است .

با سلام .

با تشکر از شما . :20:

ببخشید جسارت نشه گویا تعریفی که از مولد و پایه میشه با هم فرق داره .

من این تعریف رو توی کتاب تازه دیدم که این جوری توشته بود :

هر زیر مجموعه مستقل خطی از R^n که R^n را تولید میکند یک پایه برای R^n نامیده میشود .


===============================================

یه جای دیگه هم من این سوال رو پرسیدم یه همچین پاسخی رو در بخشی ار صحبت هاشون دادند که دقیقا گفته ایشون رو نقل قول می کنم . طرف مخاطبشون من هستم یه موقع جسارت نشه . :20:




یه نکته ای هم در ادامه بگم بد نیست....

وقتی اون مجموعه هم مستقل خطی باشه و هم مولد باشه اون وقت پایه میشه

در واقع برای اینکه بگیم مجموغه B پایه برای V هست باید ثابت کنیم که B مستقل خطی و مولد برای V هستش...
و تنها مولد بودن کافی نیست...

(((پایه و مولد را با هم اشتباه نگیر ... پایه باید هم مولد باشه هم مستقل خطی)))



لینک ([ برای مشاهده لینک ، لطفا با نام کاربری خود وارد شوید یا ثبت نام کنید ] forum%2F589%2Ft144237-4.html%23post670316)

lebesgue
07-03-2012, 20:58
خواهش می کنم، من هم مطمئن نبودم که منظور از مولد همان پایه باشد، از همین جهت گفتم احتمالاً. این اصطلاح را ندیده بودم. بیشتر می گویند یک مجموعه از بردارها کل فضا را تولید می کند (Span)، تا اینکه یک مجموعه یک مولد برای فضا است.

بهرحال، توضیحاتی که در پست قبل در مورد پایه دادم، سر جای خود است و اگر تعریف مولد را چنین در نظر بگیریم:


[ برای مشاهده لینک ، لطفا با نام کاربری خود وارد شوید یا ثبت نام کنید ] را یه مولد برای V میگیم اگر هر عنصری از V به صورت ترکیب خطی از عناصر B نوشته شود.

میتوان در مورد یک فضای n بعدی به طور خلاصه گفت:

1. هر مجموعه شامل بیشتر از n بردار، حتماً وابسته خطی خواهد بود (چه مولد باشد، چه نباشد) و در نتیجه نمی تواند یک پایه برای آن فضا باشد.

2. هر مجموعه شامل کمتر از n بردار، نمی تواند یک مولد باشد (چه مستقل خطی باشد، چه نباشد)، در نتیجه نمی تواند یک پایه برای آن فضا باشد.

3. هر مجموعه شامل دقیقاً n بردار، اگر مستقل خطی باشد، حتماً مولد نیز بوده و در نتیجه یک پایه برای آن فضا است.

این نتایج از قضایای جبر خطی حاصل می شوند. توجه کنید استفاده از نکات بالا زمانی میتواند در حل مسائل راهگشا باشد که شما از قبل، بعد فضا را به طریقی بدانید. در مورد فضای [ برای مشاهده لینک ، لطفا با نام کاربری خود وارد شوید یا ثبت نام کنید ]{R}^n، بعد به طور مشخص برابر n است، اما در مواردی ممکن است پیدا کردن بعد به سادگی ممکن نباشد. همچنین یک فضای برداری میتواند بینهایت بعدی باشد.

lovely_killer
08-03-2012, 01:38
سلام دوستان

من 3 تا سئوال جبر خطی دارم که هر کاری کردم نتونستم حلشون کنم.به هیچ کدوم از همکلاسی ها هم دسترسی ندارم :19:. اگه کسی بتونه تو حلشون کمکم کنه واقعا ممنونش میشم.:10:


برای مشاهده محتوا ، لطفا وارد شوید یا ثبت نام کنید

سلام

کسی نبود یکی از اینا رو حل کنه؟ :19:

skyzare
08-03-2012, 16:24
با سلام .

اساتید من یه چند تا سوال دارم هر کدامش رو جواب یدید ممنون میشم . :20: ( سه تا بیشتر نیست . همون جملات ابی رنگ )


========================================


1- یه ماتریس مقدماتی رو چه جوری باید تشخیص بدم ؟ اون سه تا عملیات سطری مقدماتی رو هم میدونم .



نگاه کنید مثلا توی کتاب این ها رو مثال زده :



[ برای مشاهده لینک ، لطفا با نام کاربری خود وارد شوید یا ثبت نام کنید ]{bmatrix}&space;1&space;&&space;3&space;&0&space;\\&space;0&&space;1&space;&0&space;\\&space;0&0&space;&&space;1&space;\end{bmatrix}




[ برای مشاهده لینک ، لطفا با نام کاربری خود وارد شوید یا ثبت نام کنید ]{bmatrix}&space;1&space;&0&space;&0&space;&1&space;\\&space;0&space;&1&space;&0&space;&0&space;\\&space;0&space;&1&space;&1&space;&0&space;\\&space;0&&space;0&space;&&space;0&space;&&space;1&space;\end{bmatrix}



توی کتاب گفته اولی ماتریس مقدماتی هست ولی دومی نیست .

چرا اولی ماتریس مقدماتی هست ولی دومی نه ؟



2 - توی کتاب گفته که :


ماتریس m*n ای چون B را در نظر بگیرید .


B را تحویل شده سطری می نامیم اگر :


الف ) اولین درایه غیر صفر ( در صورت وجود ) هر سطر B برابر با 1 باشد .


ب ) همه درایه های ستونی از B که شامل اولین درایه غیر صفر سطری از B است برابر با صفر باشد .


من این مورد ب رو متوجه نمیشم منظورش چی هست ؟؟؟؟



========================================


در ادامه اومده تعریف ماتریس سطری پلکانی کرده و گفته :


ماتریس B را تحویل شده سطری پلکانی می نامیم اگر تحویل شده سطری باشد و در شرایط زیر هم صدق کند .


پ ) اولین درایه غیر صفر هر سطر از اولین درایه غیر صفر سطر بعدی یه ستون اول ( دست چپ ) نزدیک تر باشد .


ت ) بعد از سطری که همه درایه های آن صفرند ، سطر غیر صفری وجود نداشته باشد .


بعد اومده یه قضیه گفته که :


هر ماتریس m*n چون A با یک ماتریس تحویل شده سطری پلکانی هم ارز است .



========================================


حالا سوالم اینه روش خاصی برای تبدیل ماتریس به سطری پلکانی هست که بشه به همه ماتریس ها اعمال کرد ؟ توی کتاب همه اش با همین عملیات سطری مقدماتی تبدیل کرده ولی روش خاصی رو نگفته . میخوام بدونم روشی هست که بشه یه همه ماتریس ها اعمال کرد ؟؟؟



با تشکر . :20:

davy jones
08-03-2012, 22:28
سلام

کسی نبود یکی از اینا رو حل کنه؟ :19:
سلام دوست گرامی!

شاید سوالات شما رو خیلی از کاربران بیان (از جمله شاید خودم) و بتونن راهنماییتون کنن که خودتون حل کنید. اما بعد از مهلت تحویل دادن اون! اینجا محل حل تمارین و تکالیف دانشگاهی نیست. اینجا محل یادگیری و رفع اشکال درسی است.

بعد از تاریخ 20 ام اگه هنوز هم تمایل داشتین که جوابهاش رو بدونین، برای کسب راهنمایی از جانب سایر دوستان و ریاضیدانان گرامی، به اینجا یه سر بزنین.

امیدوارم از دست بنده ناراحت نشده باشین. متاسفانه خیلی ها در گذشته از کاربران فعال اینجا توقعاتی داشتند که اکثر کاربران فعال اینجا رو به این تصمیم رسوندند. مطمئن باشین که این رویه به نفع خود شما نیز هست.

===========================

با سلام .

اساتید من یه چند تا سوال دارم هر کدامش رو جواب یدید ممنون میشم . [ برای مشاهده لینک ، لطفا با نام کاربری خود وارد شوید یا ثبت نام کنید ] ( سه تا بیشتر نیست . همون جملات ابی رنگ )


========================================


1- یه ماتریس مقدماتی رو چه جوری باید تشخیص بدم ؟ اون سه تا عملیات سطری مقدماتی رو هم میدونم .



نگاه کنید مثلا توی کتاب این ها رو مثال زده :



[ برای مشاهده لینک ، لطفا با نام کاربری خود وارد شوید یا ثبت نام کنید ]




[ برای مشاهده لینک ، لطفا با نام کاربری خود وارد شوید یا ثبت نام کنید ]



توی کتاب گفته اولی ماتریس مقدماتی هست ولی دومی نیست .

چرا اولی ماتریس مقدماتی هست ولی دومی نه ؟



2 - توی کتاب گفته که :


ماتریس m*n ای چون B را در نظر بگیرید .


B را تحویل شده سطری می نامیم اگر :


الف ) اولین درایه غیر صفر ( در صورت وجود ) هر سطر B برابر با 1 باشد .


ب ) همه درایه های ستونی از B که شامل اولین درایه غیر صفر سطری از B است برابر با صفر باشد .


من این مورد ب رو متوجه نمیشم منظورش چی هست ؟؟؟؟



========================================


در ادامه اومده تعریف ماتریس سطری پلکانی کرده و گفته :


ماتریس B را تحویل شده سطری پلکانی می نامیم اگر تحویل شده سطری باشد و در شرایط زیر هم صدق کند .


پ ) اولین درایه غیر صفر هر سطر از اولین درایه غیر صفر سطر بعدی یه ستون اول ( دست چپ ) نزدیک تر باشد .


ت ) بعد از سطری که همه درایه های آن صفرند ، سطر غیر صفری وجود نداشته باشد .


بعد اومده یه قضیه گفته که :


هر ماتریس m*n چون A با یک ماتریس تحویل شده سطری پلکانی هم ارز است .



========================================


حالا سوالم اینه روش خاصی برای تبدیل ماتریس به سطری پلکانی هست که بشه به همه ماتریس ها اعمال کرد ؟ توی کتاب همه اش با همین عملیات سطری مقدماتی تبدیل کرده ولی روش خاصی رو نگفته . میخوام بدونم روشی هست که بشه یه همه ماتریس ها اعمال کرد ؟؟؟



با تشکر . [ برای مشاهده لینک ، لطفا با نام کاربری خود وارد شوید یا ثبت نام کنید ]



سلام.

1- به نظرم ماتریس اولی که گذاشتین هم مقدماتی نیست چون میشه با عملیات سطری مقدماتی روی اون، درایه ی 3 رو هم تبدیل به صفر کرد. ماتریس مقدماتی به نظرم تعریفش این بود که بیشترین تعداد درایه های صفر رو بشه با اعمال سطری توش ایجاد کرد و دیگه بیشتر از اون نشه درایه ی صفر به وجود آورد. به ماتریس تحویل شده با حداکثر تعداد درایه های صفر، ماتریس مقذماتی میگفتن (البته شاید من دارم اشتباه میگم) با این تعریف، هر دو ماتریس سوال یک مقدماتی نیستند.

2- منظورش اینه که مثلا در سطر i ام، اولین درایه ی ناصفر در درایه j ام این سطر اگه وجود داشته باشه، ستون j ام ماتریس، به جز همون درایه ی [ برای مشاهده لینک ، لطفا با نام کاربری خود وارد شوید یا ثبت نام کنید ]، دیگه همه ی درایه های اون ستون، صفر باشن.

3- روش خاص و کلاسیکی به نظرم نداره. ولی باید همیشه دقت کنین که اون سطر یا ستونی که بیشتر از همه توش صفر وجود داره رو اگه با ضریب دلخواه از بقیه ی سطرها و ستونها کم کنید، بهتره. چون صفرهای همون سطر یا ستون، حفظ میشن و هم اینکه دست خودتونه که کجا صفر ایجاد کنین.


موفق باشین.
90/12/18

skyzare
10-03-2012, 22:32
با سلام .

اساتید ببخشید بردار های ویژه این عبارت رو چه جوری میشه ؟

این هم دستگاه هش که بعد از به دست اوردن مقدار ویژها این جوری شده . میدونم باید لاندا رو جایگزین کنم ولی بعدش رو دقیقا نمیدونم چه جوری به دست میاره .




[ برای مشاهده لینک ، لطفا با نام کاربری خود وارد شوید یا ثبت نام کنید ] mbda&space;_3=3&space;\\&space;\\(2&space;-\lambda&space;)x+y=0\\\\&space;x+(&space;2-\lambda&space;)y=0\\\\&space;(&space;1-\lambda&space;)z=0



با تشکر . :20:

davy jones
12-03-2012, 20:10
با سلام .

اساتید ببخشید بردار های ویژه این عبارت رو چه جوری میشه ؟

این هم دستگاه هش که بعد از به دست اوردن مقدار ویژها این جوری شده . میدونم باید لاندا رو جایگزین کنم ولی بعدش رو دقیقا نمیدونم چه جوری به دست میاره .




[ برای مشاهده لینک ، لطفا با نام کاربری خود وارد شوید یا ثبت نام کنید ] 5C:&space;%5C:&space;%5C:%5Clambda&space;_3=3&space;%5C%5C&space;%5C%5C%282&space;-%5Clambda&space;%29x+y=0%5C%5C%5C%5C&space;x+%28&space;2-%5Clambda&space;%29y=0%5C%5C%5C%5C&space;%28&space;1-%5Clambda&space;%29z=0



با تشکر . :20:
سلام.

اصل ماتریس اولیه چی بوده؟

موفق باشین.
90/12/22

skyzare
13-03-2012, 11:39
سلام.

اصل ماتریس اولیه چی بوده؟

موفق باشین.
90/12/22

با سلام .

با تشکر از شما .

ماتریس اصلی این بوده :


[ برای مشاهده لینک ، لطفا با نام کاربری خود وارد شوید یا ثبت نام کنید ]

davy jones
13-03-2012, 19:52
با سلام .

با تشکر از شما .

ماتریس اصلی این بوده :


[ برای مشاهده لینک ، لطفا با نام کاربری خود وارد شوید یا ثبت نام کنید ]


سلام.

طبق تعریف هر ماتریس دلخواه مانند:


[ برای مشاهده لینک ، لطفا با نام کاربری خود وارد شوید یا ثبت نام کنید ] x%5C%5C&space;y%5C%5C&space;z&space;%5Cend%7Bbmatrix%7D


که در اون z=0 باشه مطمئنا میتونه بردار ویژه ی ماتریس A باشه. اگه لاندا رو برابر با 1 فرض بشه، z میتونه مخالف 0 هم باشه و هر عدد دلخواه باشه.

وقتی در معادله ی همگن [ برای مشاهده لینک ، لطفا با نام کاربری خود وارد شوید یا ثبت نام کنید ] خودمون میایم و مقدار دترمینان [ برای مشاهده لینک ، لطفا با نام کاربری خود وارد شوید یا ثبت نام کنید ] رو برابر با صفر میذاریم به این جهته که معادله ی همگن فوق جوابی به جز صفر هم داشته باشه. در اون صورت هر بردار دلخواهی مانند w بردار ویژه ی ماتریس A خواهد بود. دقیقا مشابه معادله ی خطی ax=0 که اگه خودمون ضریب a رو برابر با صفر قرار بدیم، متغیر معادله (یعنی x) میتونه مقادیر ناصفر رو هم اتخاذ کنه و هر مقدار دلخواه متناهی باشه.

برای مطالعه ی بیشتر به لینک زیر مراجعه کنین:
[ برای مشاهده لینک ، لطفا با نام کاربری خود وارد شوید یا ثبت نام کنید ] D9%87_%D9%88_%D8%A8%D8%B1%D8%AF%D8%A7%D8%B1_%D9%88 %DB%8C%DA%98%D9%87


موفق باشین.
90/12/23

Life24
14-03-2012, 18:11
[ برای مشاهده لینک ، لطفا با نام کاربری خود وارد شوید یا ثبت نام کنید ]

لطفا ثابت کنید این چطوری به
[ برای مشاهده لینک ، لطفا با نام کاربری خود وارد شوید یا ثبت نام کنید ]


رسید؟ یک فرمولی داره اگر اشتباه نکنم انتها - ابتدا و از این چیزها ممنون

davy jones
14-03-2012, 19:17
[ برای مشاهده لینک ، لطفا با نام کاربری خود وارد شوید یا ثبت نام کنید ]

لطفا ثابت کنید این چطوری به
[ برای مشاهده لینک ، لطفا با نام کاربری خود وارد شوید یا ثبت نام کنید ]


رسید؟ یک فرمولی داره اگر اشتباه نکنم انتها - ابتدا و از این چیزها ممنون

سلام.

در سری ها اگر c یک عدد ثابت فرض شود (و نه یک متغیر) داریم:


[ برای مشاهده لینک ، لطفا با نام کاربری خود وارد شوید یا ثبت نام کنید ] b-a+1%29%5Ctimes&space;c


پس در مورد [ برای مشاهده لینک ، لطفا با نام کاربری خود وارد شوید یا ثبت نام کنید ] میتوان گفت:


[ برای مشاهده لینک ، لطفا با نام کاربری خود وارد شوید یا ثبت نام کنید ]


پس داریم:


[ برای مشاهده لینک ، لطفا با نام کاربری خود وارد شوید یا ثبت نام کنید ] olden%7D&space;%5Cfrac%7Bn%28n-1%29%7D%7B2%7D%7D



موفق باشین.
90/12/24

Life24
14-03-2012, 22:02
خیلی ممنون از توضیحات شما
راستی میشه این رو هم بگید . میدانم چطوری حساب میشه اما میخوام راه اثباتش بدونم. همین میگن انتها + ابتدا و از این چیزها
کلا از کدام منبع یا سایتی میشه این سری ها رو با اثبات و فرول های کلی یاد گرفت . واقعا لازمشون دارم
[ برای مشاهده لینک ، لطفا با نام کاربری خود وارد شوید یا ثبت نام کنید ]

davy jones
14-03-2012, 23:05
خیلی ممنون از توضیحات شما
راستی میشه این رو هم بگید . میدانم چطوری حساب میشه اما میخوام راه اثباتش بدونم. همین میگن انتها + ابتدا و از این چیزها
کلا از کدام منبع یا سایتی میشه این سری ها رو با اثبات و فرول های کلی یاد گرفت . واقعا لازمشون دارم
[ برای مشاهده لینک ، لطفا با نام کاربری خود وارد شوید یا ثبت نام کنید ]

سلام.

موقعی که عبارت داخل سری از جنس متغیر حدود بالا و پایین سری (به عبارت دیگه متغیر شمارنده یا شماره انداز سری) نیست، در محاسبه حکم یه عدد ثابت رو داره. در محاسبه ی این سریها مثل این میمونه که شما به تعدادی که حدود بالا و پایین سری مشخص کرده، دارین یه مقدار ثابت رو مدام با خودش جمع میکنین. حالا سوال اینه که حدود سری رو چطوری میشه فهمید که چند تاست؟ مثلا در عبارت زیر:


[ برای مشاهده لینک ، لطفا با نام کاربری خود وارد شوید یا ثبت نام کنید ]


متغیر سری برابر با i هستش، نه x. بنابراین در اینجا x مثل یه عدد ثابت میمونه و حاصل سری برابر میشه با مجموع تعدادی x. اما چند تا؟ در واقع باید پرسید که از a تا b چند تا عدد صحیح وجود داره؟ جواب اینه که بین هر دو عدد صحیح دلخواه a و b، با احتساب خود a و b، تعداد b-a+1 عدد صحیح وجود داره.

البته من هیچوقت افراد رو به حفظ کردن چنین قاعده های گفتاری تشویق و ترغیب نمیکنم. همین که شما یکبار راه اصولیش رو ببینین بای همیشه تو ذهنتون خواهد موند. اما اگه واقعا اصرار دارین و فکر میکنین که اونطوری راحت تر تو ذهن شما میمونه، قاعده اش اینه: کران بالا رو منهای کران پایین به علاوه ی یک

البته همونطور که گفتم این فقط مختص زمانیه که عبارت داخل سری از جنس متغیر موجود در کران سری نباشه.

و اما در مورد سریهایی به فرم [ برای مشاهده لینک ، لطفا با نام کاربری خود وارد شوید یا ثبت نام کنید ] که در واقع مجموع اعداد طبیعی متوالی از 1 تا هر عدد دلخواه هستند. بد نیست که یکبار هم راه اثبات فرمولش رو بدونین:

اگه این سری رو باز کنیم خواهیم داشت:


[ برای مشاهده لینک ، لطفا با نام کاربری خود وارد شوید یا ثبت نام کنید ] +3+...+%28n-1%29+n


اما میشه جملات رو دو تا دو تا به صورت یکی از ابتدا و یکی از انتها نوشت:


[ برای مشاهده لینک ، لطفا با نام کاربری خود وارد شوید یا ثبت نام کنید ] Green%7D&space;2%7D+%7B%5Ccolor%7BBlue%7D&space;3%7D+...%7B%5C color%7BBlue%7D&space;%28n-2%29%7D+%7B%5Ccolor%7BGreen%7D&space;%28n-1%29%7D+%7B%5Ccolor%7BRed%7D&space;n%7D=%7B%5Ccolor%7BRe d%7D&space;%28n+1%29%7D+%7B%5Ccolor%7BGreen%7D&space;%28n-1+2%29%7D+%7B%5Ccolor%7BBlue%7D&space;%28n-2+3%29%7D+...=%28n+1%29+%28n+1%29+%28n+1%29+...


میبینیم که حاصل هر پرانتز برابر با n+1 میشه. حالا باید فهمید که چند تا پرانتز داریم؟ چون تعداد کل اعداد موجود n تا بود و ما اونها رو در دسته های 2 تایی قرار دادیم، پس تعداد پرانتزها برابر با [ برای مشاهده لینک ، لطفا با نام کاربری خود وارد شوید یا ثبت نام کنید ] هستش. پس مجموع [ برای مشاهده لینک ، لطفا با نام کاربری خود وارد شوید یا ثبت نام کنید ] تا عبارت n+1 میشه: [ برای مشاهده لینک ، لطفا با نام کاربری خود وارد شوید یا ثبت نام کنید ]

پس:


[ برای مشاهده لینک ، لطفا با نام کاربری خود وارد شوید یا ثبت نام کنید ] 28n+1%29%7D%7B2%7D


اما ممکنه که سوال پیش بیاد که اگه تعداد n زوج نبود چی؟ چون ما فرض کردیم که میشه تعداد n عدد رو به دسته های 2 تایی تقسیم کرد. اگه n فرد هم باشه طبق همین روش و قاعده [ برای مشاهده لینک ، لطفا با نام کاربری خود وارد شوید یا ثبت نام کنید ] تا دسته ی دو تایی تشکیل میشه که حاصل هر کدوم از اون دسته ها برابر با n+1 میشه و مقدار اون عدد وسط در این سری هم خودش برابر با میانگین حسابی کران بالا و پایین سری هستش. پس حاصل سری در این حالت برابر میشه با:


[ برای مشاهده لینک ، لطفا با نام کاربری خود وارد شوید یا ثبت نام کنید ] ac%7B%28n%5E%7B2%7D-1%29+%28n+1%29%7D%7B2%7D=%5Cfrac%7Bn%5E%7B2%7D+n%7 D%7B2%7D=%5Cfrac%7Bn%28n+1%29%7D2%7B%7D


که برابر با همون مقدار قبلی هستش در موقعی که n زوج فرض شده بود. بنابراین در حالت کلی ثابت میشه.



--

برای مطالعه ی بیشتر در مورد سریها به منابع زیر میتونین مراجعه کنین:

منابع لاتین:

[ برای مشاهده لینک ، لطفا با نام کاربری خود وارد شوید یا ثبت نام کنید ]
[ برای مشاهده لینک ، لطفا با نام کاربری خود وارد شوید یا ثبت نام کنید ]


منابع فارسی:






[ برای مشاهده لینک ، لطفا با نام کاربری خود وارد شوید یا ثبت نام کنید ] 8C%D8%A7%D8%AA%29
[ برای مشاهده لینک ، لطفا با نام کاربری خود وارد شوید یا ثبت نام کنید ]


موفق باشین.
90/12/24

*M!L4D*
15-03-2012, 23:48
اساتید این سوال رو درست حل کردم ؟!

[ برای مشاهده لینک ، لطفا با نام کاربری خود وارد شوید یا ثبت نام کنید ]

iranch
19-03-2012, 20:15
دوستان درخت دودویی تقریبا کامل چیه/؟ لطفا دقیق بگوئید که میتونه از سمت چپ هم خالی باشه یا از سمت راست؟
ترتیب پر شدن چطوره/؟

Life24
21-03-2012, 11:53
سلام
یک مساله ای هست که مثلا 6 نفر دور یک میز هستند و تعداد ترتیب های قرار گرفتنشون میشه



(n-1)!



و مساله حل میشه اما برای درکش مشکل دارم میخوام بدون چرا از تعداد نفرات یکی کم میکنیم ؟
این هم زده به دلیل اینکه ترتیب هایی که دروان دیگری بدست میاد یکسان میگیریم.
یعنی چی/؟

davy jones
23-03-2012, 19:09
دوستان درخت دودویی تقریبا کامل چیه/؟ لطفا دقیق بگوئید که میتونه از سمت چپ هم خالی باشه یا از سمت راست؟
ترتیب پر شدن چطوره/؟

سلام.

سوالتون مربوط به کامپیوتر و رشته ی فناوری اطلاعات میشه و خیلی به ریاضیات مربوط نیست. علی ای حالٍ هر چند که خودم در این زمینه سررشته ای نداشتم ولی یه مقداری سرچ کردم براتون و مقالات زیر یافت شد. تقریبا تو همه ی مقالات سوال شما پاسخ داده شده. البته بایستی تسلط نسبی به زبان انگلیسی داشته باشین. امیدوارم مفید باشه براتون:

[ برای مشاهده لینک ، لطفا با نام کاربری خود وارد شوید یا ثبت نام کنید ] ([ برای مشاهده لینک ، لطفا با نام کاربری خود وارد شوید یا ثبت نام کنید ])
[ برای مشاهده لینک ، لطفا با نام کاربری خود وارد شوید یا ثبت نام کنید ]
[ برای مشاهده لینک ، لطفا با نام کاربری خود وارد شوید یا ثبت نام کنید ]

====================



سلام
یک مساله ای هست که مثلا 6 نفر دور یک میز هستند و تعداد ترتیب های قرار گرفتنشون میشه



و مساله حل میشه اما برای درکش مشکل دارم میخوام بدون چرا از تعداد نفرات یکی کم میکنیم ؟
این هم زده به دلیل اینکه ترتیب هایی که دروان دیگری بدست میاد یکسان میگیریم.
یعنی چی/؟

سلام.

برای اینکه وقتی یک زنجیره ی حلقوی از افراد تشکیل شد، دیگه فرقی نمیکنه که نفر اول روی کدوم صندلی بشینه. مهم ترتیب و جایگاه افراد نسبت به همدیگه هستش که ترتیب حلقه حفظ بشه. بنابراین اگه اونها رو ساعتگرد یا پاد ساعتگرد دور میز شیفت بدیم، حالت جدیدی ایجاد نمیشه در حالیکه ما این حالات رو در شمارش اولیه لحاظ کردیم. چون به ازای هر چینش دلخواه از این n نفر دور میز، n حالت برای چرخش این چینش خاص زنجیره دور میز و شیفت خوردن اونها در جهت ساعتگرد یا پاد ساعتگرد وجود داره، پس باید تعداد کل حالات جایگشت n نفر بدست اومده رو بر n تقسیم کنیم. دقت کنید که اگه کلمه ی «به ازای هر» صادق نبود، تقسیم کردن صحیح نبود و باید حالت های استثنا رو با عمل تفریق، کم میکردیم نه تقسیم.



موفق باشین.
91/1/4

Life24
23-03-2012, 20:54
میشه این رو برام یک مثال بزنید؟
[ برای مشاهده لینک ، لطفا با نام کاربری خود وارد شوید یا ثبت نام کنید ] 20D
وقتی اول زیر مجموعه سره هست بعد چطوری زیر مجموعه میشه؟
یک مورد دیگه هم دارم اول این حل بشه شاید اونم حل شد خودکار

davy jones
23-03-2012, 21:47
میشه این رو برام یک مثال بزنید؟
[ برای مشاهده لینک ، لطفا با نام کاربری خود وارد شوید یا ثبت نام کنید ] 20D
وقتی اول زیر مجموعه سره هست بعد چطوری زیر مجموعه میشه؟
یک مورد دیگه هم دارم اول این حل بشه شاید اونم حل شد خودکار

سلام.

تفاوت بین [ برای مشاهده لینک ، لطفا با نام کاربری خود وارد شوید یا ثبت نام کنید ] و [ برای مشاهده لینک ، لطفا با نام کاربری خود وارد شوید یا ثبت نام کنید ] دقیقا مثل تفاوت بین [ برای مشاهده لینک ، لطفا با نام کاربری خود وارد شوید یا ثبت نام کنید ] و [ برای مشاهده لینک ، لطفا با نام کاربری خود وارد شوید یا ثبت نام کنید ] هستش. در کوچکتر و کوچکترمساوی، اگر عبارتی کوچکتر از عبارت دیگری باشد، حتما کوچکتر یا مساوی با آن عبارت هم هست. اما عکس این موضوع بدیهیه که درست نیست.


[ برای مشاهده لینک ، لطفا با نام کاربری خود وارد شوید یا ثبت نام کنید ] 20D

در اینجا وقتی مجموعه ی C زیرمجموعه ی سره ی D باشه، میتونه زیرمجموعه یا مساوی با D هم باشه. چون ما داریم میگیم یــا. و منظورمون اینه که اگه C زیر مجموعه ی D باشه یا مساوی با خود مجموعه ی D باشه هر دو حالت، مطلوب ماست و کافیه که یکی از این دو شرط برقرار باشه.


موفق باشین.
91/1/4

Life24
24-03-2012, 16:33
اگر ساعت 4 بعد از ظهر روز چهارشنبه باشد. [ برای مشاهده لینک ، لطفا با نام کاربری خود وارد شوید یا ثبت نام کنید ] چه روزو ساعتتی خواهد بود؟

davy jones
24-03-2012, 20:59
اگر ساعت 4 بعد از ظهر روز چهارشنبه باشد. [ برای مشاهده لینک ، لطفا با نام کاربری خود وارد شوید یا ثبت نام کنید ] چه روزو ساعتتی خواهد بود؟

سلام. برای اینکه بدونیم [ برای مشاهده لینک ، لطفا با نام کاربری خود وارد شوید یا ثبت نام کنید ] چه ساعتی میشه، باید باقیمانده ی تقسیم این عدد رو بر 24 بدست بیاریم:


[ برای مشاهده لینک ، لطفا با نام کاربری خود وارد شوید یا ثبت نام کنید ] iv&space;%7D%5Cleft&space;%28-1&space;%5Cright&space;%29%5E%7B74%7D%5Coverset%7B24%7D%7B%5Ce quiv&space;%7D1


و این یعنی اگه ساعت 4 بعد از ظهر مبدا زمان باشه، [ برای مشاهده لینک ، لطفا با نام کاربری خود وارد شوید یا ثبت نام کنید ]، ساعت 5 بعد از ظهره.

همچنین برای اینکه بدونیم [ برای مشاهده لینک ، لطفا با نام کاربری خود وارد شوید یا ثبت نام کنید ] چند شنبه میشه، باید باقیمانده ی تقسیم این عدد رو بر 7 بدست بیاریم:


[ برای مشاهده لینک ، لطفا با نام کاربری خود وارد شوید یا ثبت نام کنید ] v&space;%7D%5Cleft&space;%28-2&space;%5Cright&space;%29%5E%7B74%7D%5Coverset%7B7%7D%7B%5Ceq uiv&space;%7D%28-2%29%5E%7B%283%5Ctimes&space;24&space;+2%29%7D%5Coverset%7B7%7 D%7B%5Cequiv&space;%7D%28-8%29%5E%7B24%7D+%28-2%29%5E%7B2%7D%5Coverset%7B7%7D%7B%5Cequiv&space;%7D%28-1%29%5E%7B24%7D+4%5Coverset%7B7%7D%7B%5Cequiv&space;%7D5


و این یعنی اگه چهارشنبه، مبدا روزهای هفته در نظر گرفته شده باشه، [ برای مشاهده لینک ، لطفا با نام کاربری خود وارد شوید یا ثبت نام کنید ] ، 5 روز بعد از چهارشنبه یعنی دوشنبه میشه.

پس [ برای مشاهده لینک ، لطفا با نام کاربری خود وارد شوید یا ثبت نام کنید ]، نشون دهنده ی ساعت 5 بعد از ظهر روز دوشنبه است.






==================
سوال کاربر محترم، Life24 ([ برای مشاهده لینک ، لطفا با نام کاربری خود وارد شوید یا ثبت نام کنید ]) از بنده در پیغام خصوصی که بعد از اجازه ی ایشون برای استفاده ی سایرین در اینجا قرار میدم:
==================


شرمنده اقا یک سوال
راستش میخوام خر فهم بشم
[ برای مشاهده لینک ، لطفا با نام کاربری خود وارد شوید یا ثبت نام کنید ] 20%5C%7D


[ برای مشاهده لینک ، لطفا با نام کاربری خود وارد شوید یا ثبت نام کنید ] left%20%5C%7B%201,2%20%5Cright%20%5C%7D%5Cleft%20% 5C%7B%201,2,3%20%5Cright%20%5C%7D%20%5Cleft%20%5C% 7B%201,2,3,4%20%5Cright%20%5C%7D%5Cright%20%5C%7D

حالا میگیم
[ برای مشاهده لینک ، لطفا با نام کاربری خود وارد شوید یا ثبت نام کنید ]
یعنی هر عضو از مجموعه A درون u وجود دارد
[ برای مشاهده لینک ، لطفا با نام کاربری خود وارد شوید یا ثبت نام کنید ]
یعنی عضوی در u است که در A نیست
[ برای مشاهده لینک ، لطفا با نام کاربری خود وارد شوید یا ثبت نام کنید ]
یعنی مجموعه a به عنوان یک عنصر درون u وجود دارد منظور اینه
[ برای مشاهده لینک ، لطفا با نام کاربری خود وارد شوید یا ثبت نام کنید ] 20%5C%7D

تا اینجا درست؟

اما این ها گیچ میشم

[ برای مشاهده لینک ، لطفا با نام کاربری خود وارد شوید یا ثبت نام کنید ] %20%5Csubseteq%20U
[ برای مشاهده لینک ، لطفا با نام کاربری خود وارد شوید یا ثبت نام کنید ] %20%5Csubset%20U
[ برای مشاهده لینک ، لطفا با نام کاربری خود وارد شوید یا ثبت نام کنید ] %5Cnotin%20U

مخوصا مورد اخر توجیهی براش ندارم




سلام.
دشمنتون شرمنده.

چه خوب بود که این سوال رو هم تو همون تاپیک اتاق ریاضیات میپرسیدین تا اگه سایر عزیزان و کاربران فعال اونجا هم نظری دارند ارائه بدند و هم اینکه این سوال به نوعی اونجا آرشیو بشه تا اگه سوال کاربران دیگه ای هم بعدا همین بود، با یه سرچ میتونستند جواب بگیرن. به هر حال ...

الان تو این مجموعه ی A و U ای که شما نوشتین داریم: [ برای مشاهده لینک ، لطفا با نام کاربری خود وارد شوید یا ثبت نام کنید ] چون: [ برای مشاهده لینک ، لطفا با نام کاربری خود وارد شوید یا ثبت نام کنید ] و در U عضوهای 1 و 2 و 3 و 4 وجود داره (منظورم اعضاییه که قرمز رنگشون کردم):


[ برای مشاهده لینک ، لطفا با نام کاربری خود وارد شوید یا ثبت نام کنید ] Cmathbf%7B1,2,3,4%7D%7D,5,6,x,y,%5Cleft&space;%5C%7B&space;1,2 &space;%5Cright&space;%5C%7D,%5Cleft&space;%5C%7B&space;1,2,3&space;%5Cright&space;%5C %7D,%5Cleft&space;%5C%7B&space;1,2,3,4&space;%5Cright&space;%5C%7D&space;%5Crigh t&space;%5C%7D



خب همونطور که قبلا هم بحث شد، اگر به ازای هر A و U دلخواه داشته باشیم [ برای مشاهده لینک ، لطفا با نام کاربری خود وارد شوید یا ثبت نام کنید ] پس حتما رابطه ی [ برای مشاهده لینک ، لطفا با نام کاربری خود وارد شوید یا ثبت نام کنید ] هم درسته و استدلال جدیدی نداره.

[ برای مشاهده لینک ، لطفا با نام کاربری خود وارد شوید یا ثبت نام کنید ]
یعنی هر عضو از مجموعه A درون u وجود دارد
[ برای مشاهده لینک ، لطفا با نام کاربری خود وارد شوید یا ثبت نام کنید ]
یعنی عضوی در u است که در A نیست این استدلال خیلی درست نیست. چرا که در هر دو حالت ممکنه که عضوی از U وجود داشته باشه که در A نباشه و همچنین در هر دو حالت حتما هر عضو A درون U وجود داره. باز هم تاکید میکنم که مفهوم کوچکتر (>) و کوچکتر مساوی ([ برای مشاهده لینک ، لطفا با نام کاربری خود وارد شوید یا ثبت نام کنید ]) رو تو ذهنتون یادآوری کنین.

------
و اما داریم: [ برای مشاهده لینک ، لطفا با نام کاربری خود وارد شوید یا ثبت نام کنید ] چون همونطور که خودتون هم اشاره کردین مجموعه ی A به عنوان یک عضو یا عنصر، در U موجوده:


[ برای مشاهده لینک ، لطفا با نام کاربری خود وارد شوید یا ثبت نام کنید ] &space;%5C%7B&space;1,2&space;%5Cright&space;%5C%7D,%5Cleft&space;%5C%7B&space;1,2,3&space;% 5Cright&space;%5C%7D,%7B%5Ccolor%7BRed%7D&space;%5Cleft&space;%5C%7B &space;1,2,3,4&space;%5Cright&space;%5C%7D%7D&space;%5Cright&space;%5C%7D


-----




وقتی میخوایم عضو یا اعضایی از یک مجموعه رو نشون بدیم، اونها رو به تنهایی نشون میدیم و دور اون عضو یا اعضا، آکولاد { } نمیذاریم. مثلا در همون U میگیم: [ برای مشاهده لینک ، لطفا با نام کاربری خود وارد شوید یا ثبت نام کنید ]
اما وقتی که بخوایم زیر مجموعه های یک مجموعه رو نشون بدیم، باید حتما اون زیر مجموعه، خودش یک مجموعه باشه و باید دورش آکولاد بذاریم. یعنی باید اینطوری بنویسیم: [ برای مشاهده لینک ، لطفا با نام کاربری خود وارد شوید یا ثبت نام کنید ] et&space;U یا [ برای مشاهده لینک ، لطفا با نام کاربری خود وارد شوید یا ثبت نام کنید ] teq&space;U

بنابراین وقتی که میگیم: [ برای مشاهده لینک ، لطفا با نام کاربری خود وارد شوید یا ثبت نام کنید ] منظورمون اعضای قرمز رنگ زیره:

[ برای مشاهده لینک ، لطفا با نام کاربری خود وارد شوید یا ثبت نام کنید ] Cmathbf%7B1,2,3,4%7D%7D,5,6,x,y,%5Cleft&space;%5C%7B&space;1,2 &space;%5Cright&space;%5C%7D,%5Cleft&space;%5C%7B&space;1,2,3&space;%5Cright&space;%5C %7D,%5Cleft&space;%5C%7B&space;1,2,3,4&space;%5Cright&space;%5C%7D&space;%5Crigh t&space;%5C%7D

اما وقتی که میگیم: [ برای مشاهده لینک ، لطفا با نام کاربری خود وارد شوید یا ثبت نام کنید ] %20%5Csubset%20U منظورمون از [ برای مشاهده لینک ، لطفا با نام کاربری خود وارد شوید یا ثبت نام کنید ] مجموعه ایه که A عضوش باشه یعنی اینطوری: [ برای مشاهده لینک ، لطفا با نام کاربری خود وارد شوید یا ثبت نام کنید ] Cright&space;%5C%7D&space;%5Cright&space;%5C%7D
پس منظورمون از [ برای مشاهده لینک ، لطفا با نام کاربری خود وارد شوید یا ثبت نام کنید ] %20%5Csubset%20U عضو قرمز رنگ زیر هستش:

[ برای مشاهده لینک ، لطفا با نام کاربری خود وارد شوید یا ثبت نام کنید ] &space;%5C%7B&space;1,2&space;%5Cright&space;%5C%7D,%5Cleft&space;%5C%7B&space;1,2,3&space;% 5Cright&space;%5C%7D,%7B%5Ccolor%7BRed%7D&space;%5Cleft&space;%5C%7B &space;1,2,3,4&space;%5Cright&space;%5C%7D%7D&space;%5Cright&space;%5C%7D

به همین حالت و دقیقا با همین استدلال، [ برای مشاهده لینک ، لطفا با نام کاربری خود وارد شوید یا ثبت نام کنید ] %20%5Csubseteq%20U هم نشون داده میشه که درست و صادق هستش.

-----

اما در [ برای مشاهده لینک ، لطفا با نام کاربری خود وارد شوید یا ثبت نام کنید ] %5Cnotin%20U، با توجه به توضیحاتی که داده شد (برای نشون دادن اعضا، آکولاد به کار نمیره و فقط برای نشون دادن زیرمجموعه ها از آکولاد استفاده میکنیم) در اینجا [ برای مشاهده لینک ، لطفا با نام کاربری خود وارد شوید یا ثبت نام کنید ] در حقیقت برابر با [ برای مشاهده لینک ، لطفا با نام کاربری خود وارد شوید یا ثبت نام کنید ] Cright&space;%5C%7D&space;%5Cright&space;%5C%7D هستش و چون U عضوی به صورت [ برای مشاهده لینک ، لطفا با نام کاربری خود وارد شوید یا ثبت نام کنید ] Cright&space;%5C%7D&space;%5Cright&space;%5C%7D نداره، پس [ برای مشاهده لینک ، لطفا با نام کاربری خود وارد شوید یا ثبت نام کنید ] عضو U نیست و داریم: [ برای مشاهده لینک ، لطفا با نام کاربری خود وارد شوید یا ثبت نام کنید ] %5Cnotin%20U

==========

حالا برای اینکه متوجه بشم که متوجه توضیحات من شدید، یه سوال نسبتا سخت هم بنده در این زمینه طرح میکنم:

مجموعه ی M رو به صورت زیر در نظر بگیرین:

[ برای مشاهده لینک ، لطفا با نام کاربری خود وارد شوید یا ثبت نام کنید ] %5C%7B&space;%5Cvarnothing&space;%5Cright&space;%5C%7D&space;,%5Cleft&space;%5C% 7B&space;%5Cvarnothing&space;,%5Cvarnothing&space;%5Cright&space;%5C%7D,%5 Cleft&space;%5C%7B&space;%5Cleft&space;%5C%7B&space;%5Cvarnothing&space;%5Cright &space;%5C%7D&space;%5Cright&space;%5C%7D,%5Cleft&space;%5C%7B&space;%5Cvarnothi ng&space;,%5Cleft&space;%5C%7B&space;%5Cvarnothing&space;%5Cright&space;%5C%7D&space;% 5Cright&space;%5C%7D%5Cright&space;%5C%7D

منظور از [ برای مشاهده لینک ، لطفا با نام کاربری خود وارد شوید یا ثبت نام کنید ] همون مجموعه ی تهی هستش.

الف) مجموعه ی M چند عضو دارد؟
ب) درستی عبارات زیر را بررسی کنید:

1- [ برای مشاهده لینک ، لطفا با نام کاربری خود وارد شوید یا ثبت نام کنید ]
2- [ برای مشاهده لینک ، لطفا با نام کاربری خود وارد شوید یا ثبت نام کنید ] C%7D%5Csubseteq&space;M
3- [ برای مشاهده لینک ، لطفا با نام کاربری خود وارد شوید یا ثبت نام کنید ] ing&space;%5Cright&space;%5C%7D&space;%5Cright&space;%5C%7D%5Csubseteq&space;M
4- [ برای مشاهده لینک ، لطفا با نام کاربری خود وارد شوید یا ثبت نام کنید ] C%7B&space;%5Cvarnothing&space;%5Cright&space;%5C%7D&space;%5Cright&space;%5C%7D &space;%5Cright&space;%5C%7D%5Csubseteq&space;M
5- [ برای مشاهده لینک ، لطفا با نام کاربری خود وارد شوید یا ثبت نام کنید ] C%7B&space;%5Cleft&space;%5C%7B&space;%5Cvarnothing&space;%5Cright&space;%5C%7D&space; %5Cright&space;%5C%7D&space;%5Cright&space;%5C%7D&space;%5Cright&space;%5C%7D%5C subseteq&space;M
6- [ برای مشاهده لینک ، لطفا با نام کاربری خود وارد شوید یا ثبت نام کنید ] ing&space;%5Cright&space;%5C%7D%5Cin&space;M
7- [ برای مشاهده لینک ، لطفا با نام کاربری خود وارد شوید یا ثبت نام کنید ] C%7B&space;%5Cvarnothing&space;%5Cright&space;%5C%7D&space;%5Cright&space;%5C%7D %5Cin&space;M
8- [ برای مشاهده لینک ، لطفا با نام کاربری خود وارد شوید یا ثبت نام کنید ] C%7B&space;%5Cvarnothing&space;%5Cright&space;%5C%7D&space;%5Cright&space;%5C%7D %5Csubseteq&space;M
9- [ برای مشاهده لینک ، لطفا با نام کاربری خود وارد شوید یا ثبت نام کنید ] ing&space;,%5Cleft&space;%5C%7B&space;%5Cvarnothing&space;%5Cright&space;%5C%7D&space; %5Cright&space;%5C%7D&space;%5Cright&space;%5C%7D%5Csubseteq&space;M
10- [ برای مشاهده لینک ، لطفا با نام کاربری خود وارد شوید یا ثبت نام کنید ] C%7B&space;%5Cvarnothing&space;%5Cright&space;%5C%7D,%5Cleft&space;%5C%7B&space; %5Cleft&space;%5C%7B&space;%5Cvarnothing&space;%5Cright&space;%5C%7D&space;%5Cri ght&space;%5C%7D&space;%5Cright&space;%5C%7D%5Csubseteq&space;M

اگه تمایل دارین پاسخ ها رو با نقل قول گرفتن از صورت سوال در اتاق ریاضیات بگذارین تا اونجا سایر دوستان هم بیان و نظرشون رو بگن. همچنین اگه اجازه بدین توضیحات این پیغام خصوصی رو هم تو اتاق ریاضیات قرار بدم.
==================

سلام بر شما
من نمیفهمم تفاوت بین [ برای مشاهده لینک ، لطفا با نام کاربری خود وارد شوید یا ثبت نام کنید ] و [ برای مشاهده لینک ، لطفا با نام کاربری خود وارد شوید یا ثبت نام کنید ] دقیقا مثل تفاوت بین [ برای مشاهده لینک ، لطفا با نام کاربری خود وارد شوید یا ثبت نام کنید ] و [ برای مشاهده لینک ، لطفا با نام کاربری خود وارد شوید یا ثبت نام کنید ] هستش. در کوچکتر و کوچکترمساوی، اگر عبارتی کوچکتر از عبارت دیگری باشد، حتما کوچکتر یا مساوی با آن عبارت هم هست.

چطور ممکنه؟ میشه مثالی بزنید من این رو بد متوجه شدم و یک چیز دیگه ظاهرا رفته تو مغزم [ برای مشاهده لینک ، لطفا با نام کاربری خود وارد شوید یا ثبت نام کنید ]
A={1,2,3}0
B={1,2}0
الان b زیر مجموعه سره هست چطور مساوی این میشه طبق گفته شما>؟
سلام.

ببینین این تعاریف خیلی چیز شاقی نیستن که خودتون رو باهاش اذیت کنین.

ما میدونیم و مطمئنیم که 10>2
مثلا وقتی مینویسیم [ برای مشاهده لینک ، لطفا با نام کاربری خود وارد شوید یا ثبت نام کنید ]، به نظرتون آیا این نامساوی برقرار و صحیح هست یا نه؟
معلومه که برقراره و درسته. چون 2 کوچکتر یا مساوی با 10 هستش. وقتی از علامت [ برای مشاهده لینک ، لطفا با نام کاربری خود وارد شوید یا ثبت نام کنید ] استفاده میکنیم لزوما دوطرف نامساوی نباید با هم برابر باشن که. میتونن برابر هم باشند و میتونه طرف سمت چپ کوچکتر از سمت راست هم باشه.

حالا وقتی مینویسیم [ برای مشاهده لینک ، لطفا با نام کاربری خود وارد شوید یا ثبت نام کنید ] یعنی A یا برابر با خود B هستش و یا زیرمجموعه ی سره ی B. و اگر از قبل میدونستیم که A زیر مجموعه ی سره ی B هست (یعنی: [ برای مشاهده لینک ، لطفا با نام کاربری خود وارد شوید یا ثبت نام کنید ]) بنابراین با خیال راحت میتونیم صحت و درستی عبارت [ برای مشاهده لینک ، لطفا با نام کاربری خود وارد شوید یا ثبت نام کنید ] رو هم اثبات کنیم. چون در این حالت شرط عوض نشده و فقط یک شرط جدید به مساله اضافه شده که میگه یا این یا اون هر کدومشون که درست باشه، شرط برقرار و درسته.

تو منطق، عملگر های And و Or رو دیدی تا حالا؟ اگه دیدین، [ برای مشاهده لینک ، لطفا با نام کاربری خود وارد شوید یا ثبت نام کنید ] مثل Or میمونه.


A={1,2,3}0
B={1,2}0
الان b زیر مجموعه سره هست چطور مساوی این میشه طبق گفته شما>؟ قرار نیست که مساوی باشن حتما که ...
قراره یا مساوی باشند یا B زیرمجموعه ی A باشه که هست.







موفق باشین.
91/1/5

skyzare
24-03-2012, 22:48
مجموعه ی M رو به صورت زیر در نظر بگیرین:

[ برای مشاهده لینک ، لطفا با نام کاربری خود وارد شوید یا ثبت نام کنید ] %5C%7B&space;%5Cvarnothing&space;%5Cright&space;%5C%7D&space;,%5Cleft&space;%5C% 7B&space;%5Cvarnothing&space;,%5Cvarnothing&space;%5Cright&space;%5C%7D,%5 Cleft&space;%5C%7B&space;%5Cleft&space;%5C%7B&space;%5Cvarnothing&space;%5Cright &space;%5C%7D&space;%5Cright&space;%5C%7D,%5Cleft&space;%5C%7B&space;%5Cvarnothi ng&space;,%5Cleft&space;%5C%7B&space;%5Cvarnothing&space;%5Cright&space;%5C%7D&space;% 5Cright&space;%5C%7D%5Cright&space;%5C%7D

منظور از [ برای مشاهده لینک ، لطفا با نام کاربری خود وارد شوید یا ثبت نام کنید ] همون مجموعه ی تهی هستش.

الف) مجموعه ی M چند عضو دارد؟
ب) درستی عبارات زیر را بررسی کنید:

1- [ برای مشاهده لینک ، لطفا با نام کاربری خود وارد شوید یا ثبت نام کنید ]
2- [ برای مشاهده لینک ، لطفا با نام کاربری خود وارد شوید یا ثبت نام کنید ] C%7D%5Csubseteq&space;M
3- [ برای مشاهده لینک ، لطفا با نام کاربری خود وارد شوید یا ثبت نام کنید ] ing&space;%5Cright&space;%5C%7D&space;%5Cright&space;%5C%7D%5Csubseteq&space;M
4- [ برای مشاهده لینک ، لطفا با نام کاربری خود وارد شوید یا ثبت نام کنید ] C%7B&space;%5Cvarnothing&space;%5Cright&space;%5C%7D&space;%5Cright&space;%5C%7D &space;%5Cright&space;%5C%7D%5Csubseteq&space;M
5- [ برای مشاهده لینک ، لطفا با نام کاربری خود وارد شوید یا ثبت نام کنید ] C%7B&space;%5Cleft&space;%5C%7B&space;%5Cvarnothing&space;%5Cright&space;%5C%7D&space; %5Cright&space;%5C%7D&space;%5Cright&space;%5C%7D&space;%5Cright&space;%5C%7D%5C subseteq&space;M
6- [ برای مشاهده لینک ، لطفا با نام کاربری خود وارد شوید یا ثبت نام کنید ] ing&space;%5Cright&space;%5C%7D%5Cin&space;M
7- [ برای مشاهده لینک ، لطفا با نام کاربری خود وارد شوید یا ثبت نام کنید ] C%7B&space;%5Cvarnothing&space;%5Cright&space;%5C%7D&space;%5Cright&space;%5C%7D %5Cin&space;M
8- [ برای مشاهده لینک ، لطفا با نام کاربری خود وارد شوید یا ثبت نام کنید ] C%7B&space;%5Cvarnothing&space;%5Cright&space;%5C%7D&space;%5Cright&space;%5C%7D %5Csubseteq&space;M
9- [ برای مشاهده لینک ، لطفا با نام کاربری خود وارد شوید یا ثبت نام کنید ] ing&space;,%5Cleft&space;%5C%7B&space;%5Cvarnothing&space;%5Cright&space;%5C%7D&space; %5Cright&space;%5C%7D&space;%5Cright&space;%5C%7D%5Csubseteq&space;M
10- [ برای مشاهده لینک ، لطفا با نام کاربری خود وارد شوید یا ثبت نام کنید ] C%7B&space;%5Cvarnothing&space;%5Cright&space;%5C%7D,%5Cleft&space;%5C%7B&space; %5Cleft&space;%5C%7B&space;%5Cvarnothing&space;%5Cright&space;%5C%7D&space;%5Cri ght&space;%5C%7D&space;%5Cright&space;%5C%7D%5Csubseteq&space;M

اگه تمایل دارین پاسخ ها رو با نقل قول گرفتن از صورت سوال در اتاق ریاضیات بگذارین تا اونجا سایر دوستان هم بیان و نظرشون رو بگن. همچنین اگه اجازه بدین توضیحات این پیغام خصوصی رو هم تو اتاق ریاضیات قرار بدم.با سلام .

بفرمایید این نظر من . مطمئن نیستم درست میگم با نه .( :41:‌)

الف : فکر کنم مجموعه M دارای 4 عضو باشه . به نظر میاد این ها یکی باشه .


[ برای مشاهده لینک ، لطفا با نام کاربری خود وارد شوید یا ثبت نام کنید ] Cleft&space;%5C%7B&space;%5Cphi&space;%5C:&space;,%5C:&space;%5Cphi&space;%5Cright&space;%5C %7D


ب :



1- درست

2- درست

3-درست

4-درست

5-غلط

6-درست

7-درست

8-درست

9-درست

10-درست


===================================


اگه همه اش اشتباه بود ..........بهم نخندیدن ها ...:31:


================================================== =====



میشه یه توضیحی هم درباره این بدید .


[ برای مشاهده لینک ، لطفا با نام کاربری خود وارد شوید یا ثبت نام کنید ] iv&space;%7D%5Cleft&space;%28-1&space;%5Cright&space;%29%5E%7B74%7D%5Coverset%7B24%7D%7B%5Ce quiv&space;%7D1

Life24
24-03-2012, 23:38
مجموعه ی M رو به صورت زیر در نظر بگیرین:

[ برای مشاهده لینک ، لطفا با نام کاربری خود وارد شوید یا ثبت نام کنید ] %5C%7B&space;%5Cvarnothing&space;%5Cright&space;%5C%7D&space;,%5Cleft&space;%5C% 7B&space;%5Cvarnothing&space;,%5Cvarnothing&space;%5Cright&space;%5C%7D,%5 Cleft&space;%5C%7B&space;%5Cleft&space;%5C%7B&space;%5Cvarnothing&space;%5Cright &space;%5C%7D&space;%5Cright&space;%5C%7D,%5Cleft&space;%5C%7B&space;%5Cvarnothi ng&space;,%5Cleft&space;%5C%7B&space;%5Cvarnothing&space;%5Cright&space;%5C%7D&space;% 5Cright&space;%5C%7D%5Cright&space;%5C%7D

منظور از [ برای مشاهده لینک ، لطفا با نام کاربری خود وارد شوید یا ثبت نام کنید ] همون مجموعه ی تهی هستش.

الف) مجموعه ی M چند عضو دارد؟
ب) درستی عبارات زیر را بررسی کنید:

1- [ برای مشاهده لینک ، لطفا با نام کاربری خود وارد شوید یا ثبت نام کنید ]
2- [ برای مشاهده لینک ، لطفا با نام کاربری خود وارد شوید یا ثبت نام کنید ] C%7D%5Csubseteq&space;M
3- [ برای مشاهده لینک ، لطفا با نام کاربری خود وارد شوید یا ثبت نام کنید ] ing&space;%5Cright&space;%5C%7D&space;%5Cright&space;%5C%7D%5Csubseteq&space;M
4- [ برای مشاهده لینک ، لطفا با نام کاربری خود وارد شوید یا ثبت نام کنید ] C%7B&space;%5Cvarnothing&space;%5Cright&space;%5C%7D&space;%5Cright&space;%5C%7D &space;%5Cright&space;%5C%7D%5Csubseteq&space;M
5- [ برای مشاهده لینک ، لطفا با نام کاربری خود وارد شوید یا ثبت نام کنید ] C%7B&space;%5Cleft&space;%5C%7B&space;%5Cvarnothing&space;%5Cright&space;%5C%7D&space; %5Cright&space;%5C%7D&space;%5Cright&space;%5C%7D&space;%5Cright&space;%5C%7D%5C subseteq&space;M
6- [ برای مشاهده لینک ، لطفا با نام کاربری خود وارد شوید یا ثبت نام کنید ] ing&space;%5Cright&space;%5C%7D%5Cin&space;M
7- [ برای مشاهده لینک ، لطفا با نام کاربری خود وارد شوید یا ثبت نام کنید ] C%7B&space;%5Cvarnothing&space;%5Cright&space;%5C%7D&space;%5Cright&space;%5C%7D %5Cin&space;M
8- [ برای مشاهده لینک ، لطفا با نام کاربری خود وارد شوید یا ثبت نام کنید ] C%7B&space;%5Cvarnothing&space;%5Cright&space;%5C%7D&space;%5Cright&space;%5C%7D %5Csubseteq&space;M
9- [ برای مشاهده لینک ، لطفا با نام کاربری خود وارد شوید یا ثبت نام کنید ] ing&space;,%5Cleft&space;%5C%7B&space;%5Cvarnothing&space;%5Cright&space;%5C%7D&space; %5Cright&space;%5C%7D&space;%5Cright&space;%5C%7D%5Csubseteq&space;M
10- [ برای مشاهده لینک ، لطفا با نام کاربری خود وارد شوید یا ثبت نام کنید ] C%7B&space;%5Cvarnothing&space;%5Cright&space;%5C%7D,%5Cleft&space;%5C%7B&space; %5Cleft&space;%5C%7B&space;%5Cvarnothing&space;%5Cright&space;%5C%7D&space;%5Cri ght&space;%5C%7D&space;%5Cright&space;%5C%7D%5Csubseteq&space;M5 عضو دارد

ب)
1.غلط
2.درست
3.درست
4.درست
5.اشتباه
6.درست
7.درست
8.درست
9.درست
10.درست

خواستم تو عید خنده تون رو جور کرده باشم :31::31:

hts1369
25-03-2012, 09:06
سلام
سال نو مبارک
یه سوال در مورد به توان رسوندن یک ماتریس
وقتی یه ماتریس رو میخواهیم به توان سه برسونیم
باید کدوم یکی از کارها رو بکنیم
[ برای مشاهده لینک ، لطفا با نام کاربری خود وارد شوید یا ثبت نام کنید ] یا [ برای مشاهده لینک ، لطفا با نام کاربری خود وارد شوید یا ثبت نام کنید ] البته من هرماتریسی رو امتحان میکنم یه جوابی که بدست میاد یه چیز میشه .
در حالت کلی با هم برابر هستن؟

davy jones
25-03-2012, 11:25
با سلام .

بفرمایید این نظر من . مطمئن نیستم درست میگم با نه .( :41:‌)

الف : فکر کنم مجموعه M دارای 4 عضو باشه . به نظر میاد این ها یکی باشه .


[ برای مشاهده لینک ، لطفا با نام کاربری خود وارد شوید یا ثبت نام کنید ] Cleft&space;%5C%7B&space;%5Cphi&space;%5C:&space;,%5C:&space;%5Cphi&space;%5Cright&space;%5C %7D


ب :



1- درست

2- درست

3-درست

4-درست

5-غلط

6-درست

7-درست

8-درست

9-درست

10-درست


===================================


اگه همه اش اشتباه بود ..........بهم نخندیدن ها ...:31:


================================================== =====



میشه یه توضیحی هم درباره این بدید .


[ برای مشاهده لینک ، لطفا با نام کاربری خود وارد شوید یا ثبت نام کنید ] iv&space;%7D%5Cleft&space;%28-1&space;%5Cright&space;%29%5E%7B74%7D%5Coverset%7B24%7D%7B%5Ce quiv&space;%7D1








سلام. جواب هاتون همگی درست اند. اگه دوست داشتین در مورد چرایی جوابهای خودتون در هر قسمت هم یه توضیحی بدید.



میشه یه توضیحی هم درباره این بدید .

[ برای مشاهده لینک ، لطفا با نام کاربری خود وارد شوید یا ثبت نام کنید ] iv&space;%7D%5Cleft&space;%28-1&space;%5Cright&space;%29%5E%7B74%7D%5Coverset%7B24%7D%7B%5Ce quiv&space;%7D1در هم نهشتی، هر مضرب دلخواه از پیمانه ی هم نهشتی (که به هر توان دلخواهی هم رسیده باشه) تاثیری در جواب نهایی نمیذاره. به عبارت دیگه و به بیان ساده تر اگه عددی که میخواهیم باقیمانده ی تقسیم اون عدد رو به 24 حساب کنیم، عامل 24 داشته باشه، میتونیم اون عامل رو تا اونجایی که میتونیم ساده و حذف کنیم و این کار تاثیری در مقدار باقیمانده ی نهایی تقسیم نمیذاره. همچنین اگه بتونیم اون عدد رو به صورت مجموع 2 یا چند عدد که برخی از اونها عامل 24 (منظور عامل عدد پیمانه) رو دارند در بیاریم، چون باقیمانده ی تقسیم قسمتهایی که مضرب 24 هستند، بر 24 برابر با صفر میشه، میتونیم اونها رو حذف کنیم و به همین ترتیب، عدد اولیه رو ساده تر کنیم:


[ برای مشاهده لینک ، لطفا با نام کاربری خود وارد شوید یا ثبت نام کنید ] iv&space;%7D%282%5Ctimes&space;24-1%29%5E%7B74%7D%5Coverset%7B24%7D%7B%5Cequiv&space;%7D%5 Csum_%7Bk=0%7D%5E%7B74%7D&space;%5Cbinom%7B74%7D%7Bk%7D% 282%5Ctimes&space;24%29%5E%7Bk%7D%28-1%29%5E%7B74-k%7D%5Coverset%7B24%7D%7B%5Cequiv&space;%7D%28-1%29%5E%7B74%7D%5Coverset%7B24%7D%7B%5Cequiv&space;%7D1


در اینجا در بسط دو جمله ای، همه ی جملات، به جز جمله ی [ برای مشاهده لینک ، لطفا با نام کاربری خود وارد شوید یا ثبت نام کنید ]، عامل 24 دارند و بنابراین باقیمانده ی تقسیم اونمها بر 24 صفر میشه و تاثیری در جواب کل نمیذارن.

یکی از متداول ترین روشهای حل چنین مسائلی، پیدا کردن ضریبی از عدد پیمانده است که با مقدار پایه ی توان عدد موجود در سمت چپ، یک واحد اختلاف داشته باشد. در اینصورت، مقدار توان عدد سمت چپ، هر چه که باشد مهم نیست چرا که در حقیقت به [ برای مشاهده لینک ، لطفا با نام کاربری خود وارد شوید یا ثبت نام کنید ] یا [ برای مشاهده لینک ، لطفا با نام کاربری خود وارد شوید یا ثبت نام کنید ] میرسیم.


اگه بازم توضیح نیاز داره بگین تا در خدمتتون باشم. اما فکر کنم، این مباحث رو شما در درس ریاضیات گسسته ی پیشدانشگاهی شاخه ی ریاضی فیزیک، قبلا گذروندین.

===============

5 عضو دارد

ب)
1.غلط
2.درست
3.درست
4.درست
5.اشتباه
6.درست
7.درست
8.درست
9.درست
10.درست

خواستم تو عید خنده تون رو جور کرده باشم [ برای مشاهده لینک ، لطفا با نام کاربری خود وارد شوید یا ثبت نام کنید ]

سلام.

به جوابهای کاربر گرامی، جناب skyzare ([ برای مشاهده لینک ، لطفا با نام کاربری خود وارد شوید یا ثبت نام کنید ]) مراجعه کنین.

===================

سلام
سال نو مبارک
یه سوال در مورد به توان رسوندن یک ماتریس
وقتی یه ماتریس رو میخواهیم به توان سه برسونیم
باید کدوم یکی از کارها رو بکنیم
[ برای مشاهده لینک ، لطفا با نام کاربری خود وارد شوید یا ثبت نام کنید ] یا [ برای مشاهده لینک ، لطفا با نام کاربری خود وارد شوید یا ثبت نام کنید ] البته من هرماتریسی رو امتحان میکنم یه جوابی که بدست میاد یه چیز میشه .
در حالت کلی با هم برابر هستن؟

سلام.
در اینجا فرقی نمیکنه:

[ برای مشاهده لینک ، لطفا با نام کاربری خود وارد شوید یا ثبت نام کنید ] mes&space;%28A%5Ctimes&space;A%29=%28A%5Ctimes&space;A%29%5Ctimes&space;A= A%5E%7B2%7D%5Ctimes&space;A

البته فکر کنم تعریف دوم شما، اصولی تر باشه چون همیشه ماتریس جدید، از سمت چپ ضرب شدنش در نظر گرفته میشه




موفق باشین.
91/1/6

iranch
26-03-2012, 11:40
با اعداد 1و2و3و4 چند عدد 4 رقمی میتوان ساخت که تعداد 1 ها فرد باشد؟

davy jones
27-03-2012, 13:46
با اعداد 1و2و3و4 چند عدد 4 رقمی میتوان ساخت که تعداد 1 ها فرد باشد؟
سلام

سوالتون ساده است. یه بار باید فرض کنید که تنها یک رقم 1 وجود داره و مساله رو حل کنین و یه بار هم باید فرض کنین که سه تا رقم 1 در عدد چهاررقمی وجود داره و مساله رو حل کنین. هر دو قسمت مساله هم بیشتر از نیم خط جوابش نمیشه.


اگه راهنمایی خواستین در خدمتتونم ولی خودتون این مساله رو حل کنین بهتره.


موفق باشین.
91/1/8

Life24
27-03-2012, 14:02
سلام

سوالتون ساده است. یه بار باید فرض کنید که تنها یک رقم 1 وجود داره و مساله رو حل کنین و یه بار هم باید فرض کنین که سه تا رقم 1 در عدد چهاررقمی وجود داره و مساله رو حل کنین. هر دو قسمت مساله هم بیشتر از نیم خط جوابش نمیشه.


اگه راهنمایی خواستین در خدمتتونم ولی خودتون این مساله رو حل کنین بهتره.


موفق باشین.
91/1/8
4*3=12
3*3*3*4=108
12+108=120

davy jones
27-03-2012, 14:29
4*3=12
3*3*3*4=108
12+108=120
سلام.
جوابتون درست نیست.
در محاسبه ی قسمت قرمز رنگ اشتباه کردین.


موفق باشین.
91/1/8

Life24
27-03-2012, 15:36
سلام.
جوابتون درست نیست.
در محاسبه ی قسمت قرمز رنگ اشتباه کردین.


موفق باشین.
91/1/8
به نظرم پاسخم صحیح می باشد. :2:

davy jones
27-03-2012, 17:04
به نظرم پاسخم صحیح می باشد. :2:
فرض کنین که تنها یکی از ارقام 1 باشد. بنابراین باید تکلیف سه رقم باقیمانده را از بین ارقام 2 و 3 و 4 تعیین کرد که برای هر کدام 3 حالت وجود دارد. ارقام به دست اومده رو میشه به چهار فاکتوریل هم جایگشت داد. پس جواب کل این حالت میشه:


[ برای مشاهده لینک ، لطفا با نام کاربری خود وارد شوید یا ثبت نام کنید ] 5Ctimes&space;3%5E%7B3%7D


که این مقدار برابر با 648 میشه.


قسمت اول سوال رو درست حل کردین.


موفق باشین.
91/1/8

Life24
27-03-2012, 17:36
فرض کنین که تنها یکی از ارقام 1 باشد. بنابراین باید تکلیف سه رقم باقیمانده را از بین ارقام 2 و 3 و 4 تعیین کرد که برای هر کدام 3 حالت وجود دارد. ارقام به دست اومده رو میشه به چهار فاکتوریل هم جایگشت داد. پس جواب کل این حالت میشه:


[ برای مشاهده لینک ، لطفا با نام کاربری خود وارد شوید یا ثبت نام کنید ] 5Ctimes&space;3%5E%7B3%7D


که این مقدار برابر با 648 میشه.


قسمت اول سوال رو درست حل کردین.


موفق باشین.
91/1/8



نه دوست عزیز این طور نمیشه
فرض میکنیم تعداد یک ها 1 می باشد
3*3*3 عدد تولید میشود که این خودش 4 حالت بجود میاد و میشه 4*3*3*3
فرض میکنیم تعدادیکها 3 تا هست
3 تا میشود و 4 حالت دارد و 4*3
میشه 12+108=120

skyzare
27-03-2012, 17:46
با سلام ...

با تشکر از شماها .

راستش من از آمار و احتمال چیزی بلد نیستم در حد مطالب هنرستان ...

ولی الان با توجه به مطالبی که شما ها گفتید نوشتم مال من هم مثل مال جناب life24 شد ..کجاش اشتباه هست ؟


[ برای مشاهده لینک ، لطفا با نام کاربری خود وارد شوید یا ثبت نام کنید ] 5C%5C&space;1%5Ctimes&space;1%5Ctimes&space;3%5Ctimes1=3%5C%5C&space;1%5Ct imes&space;3%5Ctimes&space;1%5Ctimes1=3%5C:&space;%5C:&space;%5C:&space;%5C:&space;%5C :&space;%5C:&space;%5Crightarrow&space;3%5Ctimes&space;4=12%5C%5C&space;3%5Ctime s&space;1%5Ctimes&space;1%5Ctimes1=3%5C%5C



[ برای مشاهده لینک ، لطفا با نام کاربری خود وارد شوید یا ثبت نام کنید ] %5C%5C&space;3%5Ctimes&space;3%5Ctimes&space;1%5Ctimes3=27%5C%5C&space;3%5 Ctimes&space;1%5Ctimes&space;3%5Ctimes3=27%5C:&space;%5C:&space;%5C:&space;%5C:&space; %5C:&space;%5C:&space;%5Crightarrow&space;27%5Ctimes&space;4=108%5C%5C&space;1%5 Ctimes&space;3%5Ctimes&space;3%5Ctimes3=27%5C%5C

davy jones
27-03-2012, 17:47
نه دوست عزیز این طور نمیشه
فرض میکنیم تعداد یک ها 1 می باشد
3*3*3 عدد تولید میشود که این خودش 4 حالت بوجود میاد و میشه 4*3*3*3
فرض میکنیم تعدادیکها 3 تا هست
3 تا میشود و 4 حالت دارد و 4*3
میشه 12+108=120
درسته. متوجه اشتباه خودم شدم. البته توضیح شما خیلی گویا نبود. بهتره بگیم ابتدا باید برای جایگاهی که رقم 1 میخواد اختیار کنه، یک جایگاه رو انتخاب کنیم که میشه 4 حالت (یکان یا دهگان یا صدگان یا هزارگان) بعد از این که تکلیف رقم 1 معلوم شد، برای سایر جایگاه ها، هر کدوم 3 حالت وجود داره که میشه 3 به توان 3.

اشتباه راه من این بود که حالت هایی که توش رقم تکراری میفته رو استثنا نکرده بودم.


ممنون

Life24
29-03-2012, 16:39
[ برای مشاهده لینک ، لطفا با نام کاربری خود وارد شوید یا ثبت نام کنید ] %5Cleft%20%5C%7B%20x%7Cx%20%5Cin%20_.%7BA%7Di,i%5C in%20A%20For

میشه یک توضیحی بدهید؟
نمیتونم چرا نتونستم درست بنویسیم این مجموعه رو
این for هم یعنی به ازای حداقل یک
i هم اندیس هست یکم اومد بالا تر :13:
نقطه ها هم برای نظم گذاشتم
تو کتاب این نوشته میشه مثالی بزنید برای تفهیم
فرض میکنیم I مجموعه ای ناتهی و U یک عالم باشد.به ازای هر [ برای مشاهده لینک ، لطفا با نام کاربری خود وارد شوید یا ثبت نام کنید ] فرض میکنیم [ برای مشاهده لینک ، لطفا با نام کاربری خود وارد شوید یا ثبت نام کنید ]آنگاه مجموعه I را یک مجموعه اندیس گذار و هر [ برای مشاهده لینک ، لطفا با نام کاربری خود وارد شوید یا ثبت نام کنید ] یک اندیس گویند.

hts1369
02-04-2012, 10:26
سلام
با توجه به z و [ برای مشاهده لینک ، لطفا با نام کاربری خود وارد شوید یا ثبت نام کنید ] al&space;x%5Cpartial&space;y%7D=0&space;%5C%5C مقدار a و b رو طوری بدست بیارید که در این رابطه صدق بکنه.

[ برای مشاهده لینک ، لطفا با نام کاربری خود وارد شوید یا ثبت نام کنید ] ac%7B%5Cpartial&space;%5E2z%7D%7B%5Cpartial&space;x%5Cpartial&space; y%7D-%5Cfrac%7B%5Cpartial&space;z%7D%7B%5Cpartial&space;x%7D-%5Cfrac%7B%5Cpartial&space;z%7D%7B%5Cpartial&space;y%7D+z=0

من تا یه جایی حل کردم ولی بقیه اش رو موندم.

[ برای مشاهده لینک ، لطفا با نام کاربری خود وارد شوید یا ثبت نام کنید ] %7D=%5Cfrac%7B%5Cpartial&space;u%7D%7B%5Cpartial&space;x%7D%5C left%28e%5E%7Bax+by%7D%5Cright%29+u%28x,y%29ae%5E% 7Bax+by%7D&space;%5C%5C&space;%5Cfrac%7B%5Cpartial&space;z%7D%7B%5Cp artial&space;y%7D=%5Cfrac%7B%5Cpartial&space;u%7D%7B%5Cpartial &space;y%7D%5Cleft%28e%5E%7Bax+by%7D%5Cright%29+u%28x,y% 29be%5E%7Bax+by%7D&space;%5C%5C&space;%5Cfrac%7B%5Cpartial&space;%5E 2z%7D%7B%5Cpartial&space;x%5Cpartial&space;y%7D=%5Cfrac%7B%5Cp artial&space;%5E2u%7D%7B%5Cpartial&space;x%5Cpartial&space;y%7D%5Cle ft%28e%5E%7Bax+by%7D%5Cright%29+%5Cfrac%7B%5Cparti al&space;u%7D%7B%5Cpartial&space;y%7D%5Cleft%28ae%5E%7Bax+by%7 D%5Cright%29+%5Cfrac%7B%5Cpartial&space;u%7D%7B%5Cpartia l&space;x%7D%5Cleft%28be%5E%7Bax+by%7D%5Cright%29+u%28x, y%29abe%5E%7Bax+by%7D

و

[ برای مشاهده لینک ، لطفا با نام کاربری خود وارد شوید یا ثبت نام کنید ] al&space;x%5Cpartial&space;y%7D-%5Cfrac%7B%5Cpartial&space;z%7D%7B%5Cpartial&space;x%7D-%5Cfrac%7B%5Cpartial&space;z%7D%7B%5Cpartial&space;y%7D+z=0&space;%5 C%5C&space;%5Cleft%28%5Cfrac%7B%5Cpartial&space;u%7D%7B%5Cpart ial&space;y%7D%5Cleft%28ae%5E%7Bax+by%7D%5Cright%29+%5Cf rac%7B%5Cpartial&space;u%7D%7B%5Cpartial&space;x%7D%5Cleft%28b e%5E%7Bax+by%7D%5Cright%29+u%28x,y%29abe%5E%7Bax+b y%7D%5Cright%29-%5Cleft%28%5Cfrac%7B%5Cpartial&space;u%7D%7B%5Cpartial&space;x %7D%5Cleft%28e%5E%7Bax+by%7D%5Cright%29+u%28x,y%29 ae%5E%7Bax+by%7D%5Cright%29-%5Cleft%28%5Cfrac%7B%5Cpartial&space;u%7D%7B%5Cpartial&space;y %7D%5Cleft%28e%5E%7Bax+by%7D%5Cright%29+u%28x,y%29 be%5E%7Bax+by%7D%5Cright%29+z=0&space;%5C%5C&space;%5Cfrac%7B% 5Cpartial&space;u%7D%7B%5Cpartial&space;y%7D%5Cleft%28ae%5E%7B ax+by%7D%5Cright%29-%5Cfrac%7B%5Cpartial&space;u%7D%7B%5Cpartial&space;y%7D%5Cleft %28e%5E%7Bax+by%7D%5Cright%29+%5Cfrac%7B%5Cpartial &space;u%7D%7B%5Cpartial&space;x%7D%5Cleft%28be%5E%7Bax+by%7D% 5Cright%29-%5Cfrac%7B%5Cpartial&space;u%7D%7B%5Cpartial&space;x%7D%5Cleft %28e%5E%7Bax+by%7D%5Cright%29+abz-az-bz+z=0&space;%5C%5C&space;%5Cfrac%7B%5Cpartial&space;u%7D%7B%5Cparti al&space;y%7D%5Cleft%28e%5E%7Bax+by%7D%5Cright%29%28a-1%29+%5Cfrac%7B%5Cpartial&space;u%7D%7B%5Cpartial&space;x%7D%5 Cleft%28e%5E%7Bax+by%7D%5Cright%29%28b-1%29+z%28a-1%29%28b-1%29=0

من اینجا گیر کردم جواب نهایی رو اخر کتاب نوشته که a=b=1 هست من موندم از کجا تونسته این نتیجه گیری رو بکنه.
یعنی منظورم اینه که از کجا معلوم شاید [ برای مشاهده لینک ، لطفا با نام کاربری خود وارد شوید یا ثبت نام کنید ] %7D یا [ برای مشاهده لینک ، لطفا با نام کاربری خود وارد شوید یا ثبت نام کنید ] %7D برابر با صفر باشه اونموقع با انتخاب هر عددی به جای a و b رابطه برقرار هست.

masoudjanbaz_civil
03-04-2012, 16:39
سلام دوستان
کسی هست که بتونه بهم در مورد دایره بوسان توضیح بده؟
یه مفهوم کلی ازش میخوام و میخوام بدونم که اصلا چی هست.
پیش پیش ممنونم.

sdfssdfs
03-04-2012, 20:51
سلام دوستان
کسی هست که بتونه بهم در مورد دایره بوسان توضیح بده؟
یه مفهوم کلی ازش میخوام و میخوام بدونم که اصلا چی هست.
پیش پیش ممنونم.


دوست عزیز نمیدون تحصیلاتت در چه حد هست ولی دابره بوسان مربوط میشه به ریاضی 2 دانشگاه(ترم دوم سال اول )
مفهومش به طور خلاصه این میشه که در یک نقطه خاص از یک منحنی انحنا داریم و شعاع انحنا هم داریم و دایره بوسان دایره ای هستش که شعاعی برابر با شعاع انحنا داره و در ان نقطه بر منحنی مماسه ( منحنی رو میبوسه )

mojtaba2321
03-04-2012, 21:31
درود به همه دوستان
میخواستم بدونم فرمول بدست آوردن xor در منطق بولی به چه صورتیه؟

hts1369
03-04-2012, 21:38
درود به همه دوستان
میخواستم بدونم فرمول بدست آوردن xor در منطق بولی به چه صورتیه؟
اینجا که تایپیک مدارمنطقی نیست.
اگه تو جزواتم پیداش کردم میزارمش.

mojtaba2321
03-04-2012, 21:44
اینجا که تایپیک مدارمنطقی نیست.
اگه تو جزواتم پیداش کردم میزارمش.
سلام
نمیدونستم مربوط به کدوم بخش میشه.
جوابش 1 نمره فاینال داره:20:
اگه میشه خیلی زود جوابشو بهم بگید.

skyzare
03-04-2012, 21:59
سلام
نمیدونستم مربوط به کدوم بخش میشه.
جوابش 1 نمره فاینال داره:20:
اگه میشه خیلی زود جوابشو بهم بگید.

الان توی این فرمول A با B ؛ xor شده :


[ برای مشاهده لینک ، لطفا با نام کاربری خود وارد شوید یا ثبت نام کنید ]


این هم نمادش :

[ برای مشاهده لینک ، لطفا با نام کاربری خود وارد شوید یا ثبت نام کنید ]

این هم مدارش :

[ برای مشاهده لینک ، لطفا با نام کاربری خود وارد شوید یا ثبت نام کنید ]



[ برای مشاهده لینک ، لطفا با نام کاربری خود وارد شوید یا ثبت نام کنید ] ([ برای مشاهده لینک ، لطفا با نام کاربری خود وارد شوید یا ثبت نام کنید ] Fwiki%2FXOR_gate%23Symbols)




===========================



اینجا که تایپیک مدارمنطقی نیست.


فعلا تا اطلاع ثانوی تالار ریاضی هم نیست ....:31: ( :41:)



تا اطلاع ثانوی، به تالار ریاضیات، سر نخواهم زد.

navidr
03-04-2012, 22:38
سلام
با توجه به z و [ برای مشاهده لینک ، لطفا با نام کاربری خود وارد شوید یا ثبت نام کنید ] al&space;x%5Cpartial&space;y%7D=0&space;%5C%5C مقدار a و b رو طوری بدست بیارید که در این رابطه صدق بکنه.

[ برای مشاهده لینک ، لطفا با نام کاربری خود وارد شوید یا ثبت نام کنید ] ac%7B%5Cpartial&space;%5E2z%7D%7B%5Cpartial&space;x%5Cpartial&space; y%7D-%5Cfrac%7B%5Cpartial&space;z%7D%7B%5Cpartial&space;x%7D-%5Cfrac%7B%5Cpartial&space;z%7D%7B%5Cpartial&space;y%7D+z=0

من تا یه جایی حل کردم ولی بقیه اش رو موندم.

[ برای مشاهده لینک ، لطفا با نام کاربری خود وارد شوید یا ثبت نام کنید ] %7D=%5Cfrac%7B%5Cpartial&space;u%7D%7B%5Cpartial&space;x%7D%5C left%28e%5E%7Bax+by%7D%5Cright%29+u%28x,y%29ae%5E% 7Bax+by%7D&space;%5C%5C&space;%5Cfrac%7B%5Cpartial&space;z%7D%7B%5Cp artial&space;y%7D=%5Cfrac%7B%5Cpartial&space;u%7D%7B%5Cpartial &space;y%7D%5Cleft%28e%5E%7Bax+by%7D%5Cright%29+u%28x,y% 29be%5E%7Bax+by%7D&space;%5C%5C&space;%5Cfrac%7B%5Cpartial&space;%5E 2z%7D%7B%5Cpartial&space;x%5Cpartial&space;y%7D=%5Cfrac%7B%5Cp artial&space;%5E2u%7D%7B%5Cpartial&space;x%5Cpartial&space;y%7D%5Cle ft%28e%5E%7Bax+by%7D%5Cright%29+%5Cfrac%7B%5Cparti al&space;u%7D%7B%5Cpartial&space;y%7D%5Cleft%28ae%5E%7Bax+by%7 D%5Cright%29+%5Cfrac%7B%5Cpartial&space;u%7D%7B%5Cpartia l&space;x%7D%5Cleft%28be%5E%7Bax+by%7D%5Cright%29+u%28x, y%29abe%5E%7Bax+by%7D

و

[ برای مشاهده لینک ، لطفا با نام کاربری خود وارد شوید یا ثبت نام کنید ] al&space;x%5Cpartial&space;y%7D-%5Cfrac%7B%5Cpartial&space;z%7D%7B%5Cpartial&space;x%7D-%5Cfrac%7B%5Cpartial&space;z%7D%7B%5Cpartial&space;y%7D+z=0&space;%5 C%5C&space;%5Cleft%28%5Cfrac%7B%5Cpartial&space;u%7D%7B%5Cpart ial&space;y%7D%5Cleft%28ae%5E%7Bax+by%7D%5Cright%29+%5Cf rac%7B%5Cpartial&space;u%7D%7B%5Cpartial&space;x%7D%5Cleft%28b e%5E%7Bax+by%7D%5Cright%29+u%28x,y%29abe%5E%7Bax+b y%7D%5Cright%29-%5Cleft%28%5Cfrac%7B%5Cpartial&space;u%7D%7B%5Cpartial&space;x %7D%5Cleft%28e%5E%7Bax+by%7D%5Cright%29+u%28x,y%29 ae%5E%7Bax+by%7D%5Cright%29-%5Cleft%28%5Cfrac%7B%5Cpartial&space;u%7D%7B%5Cpartial&space;y %7D%5Cleft%28e%5E%7Bax+by%7D%5Cright%29+u%28x,y%29 be%5E%7Bax+by%7D%5Cright%29+z=0&space;%5C%5C&space;%5Cfrac%7B% 5Cpartial&space;u%7D%7B%5Cpartial&space;y%7D%5Cleft%28ae%5E%7B ax+by%7D%5Cright%29-%5Cfrac%7B%5Cpartial&space;u%7D%7B%5Cpartial&space;y%7D%5Cleft %28e%5E%7Bax+by%7D%5Cright%29+%5Cfrac%7B%5Cpartial &space;u%7D%7B%5Cpartial&space;x%7D%5Cleft%28be%5E%7Bax+by%7D% 5Cright%29-%5Cfrac%7B%5Cpartial&space;u%7D%7B%5Cpartial&space;x%7D%5Cleft %28e%5E%7Bax+by%7D%5Cright%29+abz-az-bz+z=0&space;%5C%5C&space;%5Cfrac%7B%5Cpartial&space;u%7D%7B%5Cparti al&space;y%7D%5Cleft%28e%5E%7Bax+by%7D%5Cright%29%28a-1%29+%5Cfrac%7B%5Cpartial&space;u%7D%7B%5Cpartial&space;x%7D%5 Cleft%28e%5E%7Bax+by%7D%5Cright%29%28b-1%29+z%28a-1%29%28b-1%29=0

من اینجا گیر کردم جواب نهایی رو اخر کتاب نوشته که a=b=1 هست من موندم از کجا تونسته این نتیجه گیری رو بکنه.
یعنی منظورم اینه که از کجا معلوم شاید [ برای مشاهده لینک ، لطفا با نام کاربری خود وارد شوید یا ثبت نام کنید ] %7D یا [ برای مشاهده لینک ، لطفا با نام کاربری خود وارد شوید یا ثبت نام کنید ] %7D برابر با صفر باشه اونموقع با انتخاب هر عددی به جای a و b رابطه برقرار هست.

در رابطه اخر از توان نمایی فاکتور بگیرید و به جای z رابطه رو برحسب x y و u بنویسید.این معادله ای که بدست میاد که به فرم
P(x,y,z) dz/dx+Q(x,y,z)dz/dy=R(x,y,x) هست به معادله لاگرانژ معروفه(دقت کنید قسمت نمایی نمیتونه صفر باشه) حال اگه a=b=1 نباشه که یه جواب بدیهی مسئله هست میایم دستگاه لاگرانژو تشکیل میدیم
dx/P(x,y,x)=dy/Q(x,y,z)=dz/R(x,y,z)
جواب به صورت زیر
بدست میاد
u=(b-1)y
u=-(a-1)x
حال اگه در معادله جایگذاری کنیم مشاهده می کنیم که ضریب های ناشی از du/dx و du/dy حذف میشن و فقط یه عبارت تک جمله ای برحسب x یا y میمونه که نتیجه میده a=1 و b=1
مسئله هنوز یه حل کوچیک داره و اون اینه که به ازای a=1 و b مخالف 1 و برعکس نتیجه بگیریم امکان پذیر نیست که به نظرم حل میشه

mojtaba2321
04-04-2012, 22:03
الان توی این فرمول A با B ؛ xor شده :


[ برای مشاهده لینک ، لطفا با نام کاربری خود وارد شوید یا ثبت نام کنید ]


این هم نمادش :

[ برای مشاهده لینک ، لطفا با نام کاربری خود وارد شوید یا ثبت نام کنید ]

این هم مدارش :

[ برای مشاهده لینک ، لطفا با نام کاربری خود وارد شوید یا ثبت نام کنید ]



[ برای مشاهده لینک ، لطفا با نام کاربری خود وارد شوید یا ثبت نام کنید ] ([ برای مشاهده لینک ، لطفا با نام کاربری خود وارد شوید یا ثبت نام کنید ] Fwiki%2FXOR_gate%23Symbols)














ممنون ازتون
مدارش رو نمیخوام.
روالی که به ما یاد دادن با ورودی 1 و 0 بود.
میشه با 1 و 0 و قوانین or,and و اینا برام اثباتش کنید؟

davy jones
04-04-2012, 22:25
ممنون ازتون
مدارش رو نمیخوام.
روالی که به ما یاد دادن با ورودی 1 و 0 بود.
میشه با 1 و 0 و قوانین or,and و اینا برام اثباتش کنید؟
سلام.

لینک زیر رو مشاهده بفرمایین:

[ برای مشاهده لینک ، لطفا با نام کاربری خود وارد شوید یا ثبت نام کنید ]


موفق باشین.
91/1/16

mojtaba2321
04-04-2012, 22:31
سلام.

لینک زیر رو مشاهده بفرمایین:

[ برای مشاهده لینک ، لطفا با نام کاربری خود وارد شوید یا ثبت نام کنید ] ([ برای مشاهده لینک ، لطفا با نام کاربری خود وارد شوید یا ثبت نام کنید ] Fwiki%2FLogic_gate)


موفق باشین.
91/1/16
ممنون
ولی بازم منطقش رو نفهمیدم.

M E S S I
04-04-2012, 22:31
سلام دوستان
يكي از بچه هاي فاميلمون چند تا سوال به نسبه راحت داشت كه بهش اينجا رو معرفي كردم
ممنون كه چشاتونو خسته ميكنيد و وقت ميزاريد.


[ برای مشاهده لینک ، لطفا با نام کاربری خود وارد شوید یا ثبت نام کنید ]


[ برای مشاهده لینک ، لطفا با نام کاربری خود وارد شوید یا ثبت نام کنید ]


[ برای مشاهده لینک ، لطفا با نام کاربری خود وارد شوید یا ثبت نام کنید ]


[ برای مشاهده لینک ، لطفا با نام کاربری خود وارد شوید یا ثبت نام کنید ]

sdfssdfs
05-04-2012, 10:36
سلام دوستان
يكي از بچه هاي فاميلمون چند تا سوال به نسبه راحت داشت كه بهش اينجا رو معرفي كردم
ممنون كه چشاتونو خسته ميكنيد و وقت ميزاريد.


[ برای مشاهده لینک ، لطفا با نام کاربری خود وارد شوید یا ثبت نام کنید ]


[ برای مشاهده لینک ، لطفا با نام کاربری خود وارد شوید یا ثبت نام کنید ]


[ برای مشاهده لینک ، لطفا با نام کاربری خود وارد شوید یا ثبت نام کنید ]


[ برای مشاهده لینک ، لطفا با نام کاربری خود وارد شوید یا ثبت نام کنید ]






اولی که خیلی راحته میشه حجم متوازی السطوح که از محاسبه دترمینان ماتریس 3*3 حاصل از سه بردار به دست میاد


دومی هم که راحت تره بردار های هم صفحه حجم متوازی السطوح توشون صفره پس باید دترمینان سوال قبل 0 بشه


سومی هم سادس برای تقاطع یا تنافر باید خط هارو قطع بدی ببینی نقطه اشبراک دارن یا نه


اخریم که جوابشو خودت نوشتی؟

masoudjanbaz_civil
05-04-2012, 19:33
سلام
من دانشجوی ترم 2 کارشناسی نا پیوسته عمران البته از رشته فنی هستم.این اسم اولین باری بود که به گوشم خورده.هر جا هم گشتم نتونستم چیزی در موردش پیدا کنم.راستشو بخوای خیلی هم ریاضیاتم خوب نیست.میتونی کتابیرو معرفی کنی که بتونم اطلاعات تکمیلیتر در موردش پیدا کنم.آخه باید به استاد ارائه کنم.
پیش پیش ممنون

---------- Post added at 07:33 PM ---------- Previous post was at 07:30 PM ----------


دوست عزیز نمیدون تحصیلاتت در چه حد هست ولی دابره بوسان مربوط میشه به ریاضی 2 دانشگاه(ترم دوم سال اول )
مفهومش به طور خلاصه این میشه که در یک نقطه خاص از یک منحنی انحنا داریم و شعاع انحنا هم داریم و دایره بوسان دایره ای هستش که شعاعی برابر با شعاع انحنا داره و در ان نقطه بر منحنی مماسه ( منحنی رو میبوسه )
سلام
من دانشجوی ترم 2 کارشناسی نا پیوسته عمران البته از رشته فنی هستم.این اسم اولین باری بود که به گوشم خورده.هر جا هم گشتم نتونستم چیزی در موردش پیدا کنم.راستشو بخوای خیلی هم ریاضیاتم خوب نیست.میتونی کتابیرو معرفی کنی که بتونم اطلاعات تکمیلیتر در موردش پیدا کنم.آخه باید به استاد ارائه کنم.
پیش پیش ممنون

Mohammad Yek110
06-04-2012, 17:27
سلام دوستان
یه سوال داشتم ممنون میشم جواب بدید
پانصد و پنجاه و پنج به توان ششصد و شصت و شش
به علاوه
صد و یازده به توان پانصد و پنجاه و پنج
به علاوه
ششصد و شصت و شش به توان ششصد و شصت و شش
اینا با هم عدد اول میشن یا مرکب؟با دلیل لطفا
ممنون:11:

davy jones
06-04-2012, 20:44
سلام دوستان
یه سوال داشتم ممنون میشم جواب بدید
پانصد و پنجاه و پنج به توان ششصد و شصت و شش
به علاوه
صد و یازده به توان پانصد و پنجاه و پنج
به علاوه
ششصد و شصت و شش به توان ششصد و شصت و شش
اینا با هم عدد اول میشن یا مرکب؟با دلیل لطفا
ممنون:11:
مسلما عدد مرکب هستش و اول نیست. چرا؟
چون که یکان عدد [ برای مشاهده لینک ، لطفا با نام کاربری خود وارد شوید یا ثبت نام کنید ] مطمئنا برابر با 5 میشه و یکان عدد [ برای مشاهده لینک ، لطفا با نام کاربری خود وارد شوید یا ثبت نام کنید ] هم مطمئنا برابر با 1 میشه و این یعنی هر کدوم از این دو عدد، فرد هستند و حاصل جمعشون مطمئنا زوج میشه.

از طرفی عدد [ برای مشاهده لینک ، لطفا با نام کاربری خود وارد شوید یا ثبت نام کنید ] هم مقدارش هر چه که هست، مطمئنا زوج خواهد بود چرا که 666 خودش زوج هستش و به هر توانی هم که برسه، زوج خواهد ماند.

و چون مجموع دو عدد زوج مسلما زوج میشه پس حاصل جمع نهایی عددی زوج است و نمیتونه اول باشه چرا که تنها عددی که هم زوج هست و هم اول هست، خود عدد 2 هستش و این عدد بسیار از 2 بزرگتره.

البته اگه حاصل نهایی طبق این استدلال فرد میشد باز هم نمیشد نتیجه گرفت که اون عدد اول هستش ولی لان دیگه مطمئنیم که اول نیست.



موفق باشین.
91/1/18

davy jones
07-04-2012, 22:19
مسلما عدد مرکب هستش و اول نیست. چرا؟
چون که یکان عدد [ برای مشاهده لینک ، لطفا با نام کاربری خود وارد شوید یا ثبت نام کنید ] مطمئنا برابر با 5 میشه و یکان عدد [ برای مشاهده لینک ، لطفا با نام کاربری خود وارد شوید یا ثبت نام کنید ] هم مطمئنا برابر با 1 میشه و این یعنی هر کدوم از این دو عدد، فرد هستند و حاصل جمعشون مطمئنا زوج میشه.

از طرفی عدد [ برای مشاهده لینک ، لطفا با نام کاربری خود وارد شوید یا ثبت نام کنید ] هم مقدارش هر چه که هست، مطمئنا زوج خواهد بود چرا که 666 خودش زوج هستش و به هر توانی هم که برسه، زوج خواهد ماند.

و چون مجموع دو عدد زوج مسلما زوج میشه پس حاصل جمع نهایی عددی زوج است و نمیتونه اول باشه چرا که تنها عددی که هم زوج هست و هم اول هست، خود عدد 2 هستش و این عدد بسیار از 2 بزرگتره.

البته اگه حاصل نهایی طبق این استدلال فرد میشد باز هم نمیشد نتیجه گرفت که اون عدد اول هستش ولی لان دیگه مطمئنیم که اول نیست.



موفق باشین.
91/1/18
در ادامه علاقه مندان عزیز به عنوان تمرین بیشتر نشان دهید که عدد [ برای مشاهده لینک ، لطفا با نام کاربری خود وارد شوید یا ثبت نام کنید ] اول است یا مرکب، البته با ذکر دلیل!


موفق باشین.
91/1/19

register_e_zoori
07-04-2012, 23:06
سلام
این اولین سوال از اولین مبحث درس حسابان سال سوم دبیرستان هست .
لطفا راهنمایی بفرمایید که چطوری باید حل کنم .
با تشکر فراوان ...

برای یک عدد طبیعی n نشان دهید :

[ برای مشاهده لینک ، لطفا با نام کاربری خود وارد شوید یا ثبت نام کنید ] 20=%20%5Cfrac%7Bn%28n+1%29%7D%7B2%7D

ashjaee
07-04-2012, 23:37
در ادامه علاقه مندان عزیز به عنوان تمرین بیشتر نشان دهید که عدد [ برای مشاهده لینک ، لطفا با نام کاربری خود وارد شوید یا ثبت نام کنید ] اول است یا مرکب، البته با ذکر دلیل!


موفق باشین.
91/1/19

به نظر من مرکب هست. دلیل اول که میتونیم بگیم بر 111 بخش پذیره.
راه دوم هم برای این که یکان رو حساب کنیم؛ [ برای مشاهده لینک ، لطفا با نام کاربری خود وارد شوید یا ثبت نام کنید ] که یکانش 1 هست. [ برای مشاهده لینک ، لطفا با نام کاربری خود وارد شوید یا ثبت نام کنید ] هم که 5 هست. میمونه [ برای مشاهده لینک ، لطفا با نام کاربری خود وارد شوید یا ثبت نام کنید ] یکان 777 میشه 7. [ برای مشاهده لینک ، لطفا با نام کاربری خود وارد شوید یا ثبت نام کنید ] میشه 9. [ برای مشاهده لینک ، لطفا با نام کاربری خود وارد شوید یا ثبت نام کنید ] میشه 3. [ برای مشاهده لینک ، لطفا با نام کاربری خود وارد شوید یا ثبت نام کنید ] میشه 1. دوباره [ برای مشاهده لینک ، لطفا با نام کاربری خود وارد شوید یا ثبت نام کنید ] میشه 7. از این جا به بعد دیگه دور میزنه. باید ببینیم باقیمانده توان 222 بر چهار چند میشه که میشه 2 پس بکانش 9 هست. پس جمع یکان ها میشه [ برای مشاهده لینک ، لطفا با نام کاربری خود وارد شوید یا ثبت نام کنید ] پس بر 5 بخش پذیره.


سلام
این اولین سوال از اولین مبحث درس حسابان سال سوم دبیرستان هست .
لطفا راهنمایی بفرمایید که چطوری باید حل کنم .
با تشکر فراوان ...

برای یک عدد طبیعی n نشان دهید :

[ برای مشاهده لینک ، لطفا با نام کاربری خود وارد شوید یا ثبت نام کنید ] 20=%20%5Cfrac%7Bn%28n+1%29%7D%7B2%7D



من اینو با اصل استقرای ریاضی بلدم ولی با حسابان نمیدونم.

sdfssdfs
07-04-2012, 23:45
سلام
من دانشجوی ترم 2 کارشناسی نا پیوسته عمران البته از رشته فنی هستم.این اسم اولین باری بود که به گوشم خورده.هر جا هم گشتم نتونستم چیزی در موردش پیدا کنم.راستشو بخوای خیلی هم ریاضیاتم خوب نیست.میتونی کتابیرو معرفی کنی که بتونم اطلاعات تکمیلیتر در موردش پیدا کنم.آخه باید به استاد ارائه کنم.
پیش پیش ممنون

---------- Post added at 07:33 PM ---------- Previous post was at 07:30 PM ----------


سلام
من دانشجوی ترم 2 کارشناسی نا پیوسته عمران البته از رشته فنی هستم.این اسم اولین باری بود که به گوشم خورده.هر جا هم گشتم نتونستم چیزی در موردش پیدا کنم.راستشو بخوای خیلی هم ریاضیاتم خوب نیست.میتونی کتابیرو معرفی کنی که بتونم اطلاعات تکمیلیتر در موردش پیدا کنم.آخه باید به استاد ارائه کنم.
پیش پیش ممنون

سلام این خلاصه ریاضی 2 کنکور ارشده مه من یه زمانی میخوندم فصل انحنا رو گذاشتم برات نوشته معتقدی هستش :
[ برای مشاهده لینک ، لطفا با نام کاربری خود وارد شوید یا ثبت نام کنید ]

---------- Post added at 12:45 AM ---------- Previous post was at 12:43 AM ----------


سلام
این اولین سوال از اولین مبحث درس حسابان سال سوم دبیرستان هست .
لطفا راهنمایی بفرمایید که چطوری باید حل کنم .
با تشکر فراوان ...

برای یک عدد طبیعی n نشان دهید :

[ برای مشاهده لینک ، لطفا با نام کاربری خود وارد شوید یا ثبت نام کنید ] 20=%20%5Cfrac%7Bn%28n+1%29%7D%7B2%7D


دوست عزیز این که کاری نداره کافیه یه بار سمت چپو بنویسی و یه بار سمت چپو از راست به چپ زیرش بنویسی و با هم جمع کنی و بعد حاصلو نصف کنی همین

Greedy
14-04-2012, 16:16
سلام
سوالم زیاد ربطی به این تاپیک نداره ولی گفتم بهترین جا واسه جواب گرفتن همینجاست
میخواستم ببینم همچین تاپیکی واسه مسائل مربوط به فیزیک نیست تو اینجا ؟

davy jones
14-04-2012, 17:54
سلام
سوالم زیاد ربطی به این تاپیک نداره ولی گفتم بهترین جا واسه جواب گرفتن همینجاست
میخواستم ببینم همچین تاپیکی واسه مسائل مربوط به فیزیک نیست تو اینجا ؟
سلام.

اگه سوال فیزیکتون مربوط به فیزیک 1 و 2 دانشگاه نباشه، فکر کنم مربوط ترین تاپیک به درخواست شما این باشه:

مهم : رفع اشکال سوالات درسی مباحث مقاطع دبیرستان و پیش دانشگاهی و کنکوری ها ([ برای مشاهده لینک ، لطفا با نام کاربری خود وارد شوید یا ثبت نام کنید ]) ([ برای مشاهده لینک ، لطفا با نام کاربری خود وارد شوید یا ثبت نام کنید ] 1 ([ برای مشاهده لینک ، لطفا با نام کاربری خود وارد شوید یا ثبت نام کنید ]) 2 ([ برای مشاهده لینک ، لطفا با نام کاربری خود وارد شوید یا ثبت نام کنید ]) 3 ([ برای مشاهده لینک ، لطفا با نام کاربری خود وارد شوید یا ثبت نام کنید ]) 4 ([ برای مشاهده لینک ، لطفا با نام کاربری خود وارد شوید یا ثبت نام کنید ]) 5 ([ برای مشاهده لینک ، لطفا با نام کاربری خود وارد شوید یا ثبت نام کنید ]) ... آخرين صفحه ([ برای مشاهده لینک ، لطفا با نام کاربری خود وارد شوید یا ثبت نام کنید ]))


البته همینجا هم بپرسین شاید دوستانی باشن که جواب بدن.


موفق باشین.
91/1/26

lebesgue
14-04-2012, 18:24
در ادامه علاقه مندان عزیز به عنوان تمرین بیشتر نشان دهید که عدد [ برای مشاهده لینک ، لطفا با نام کاربری خود وارد شوید یا ثبت نام کنید ] اول است یا مرکب، البته با ذکر دلیل!


موفق باشین.
91/1/19

به نظر من نه اول است نه مرکب، چون منفی است!

Greedy
14-04-2012, 18:38
سلام.

اگه سوال فیزیکتون مربوط به فیزیک 1 و 2 دانشگاه نباشه، فکر کنم مربوط ترین تاپیک به درخواست شما این باشه:

مهم : رفع اشکال سوالات درسی مباحث مقاطع دبیرستان و پیش دانشگاهی و کنکوری ها ([ برای مشاهده لینک ، لطفا با نام کاربری خود وارد شوید یا ثبت نام کنید ]) ([ برای مشاهده لینک ، لطفا با نام کاربری خود وارد شوید یا ثبت نام کنید ] 1 ([ برای مشاهده لینک ، لطفا با نام کاربری خود وارد شوید یا ثبت نام کنید ]) 2 ([ برای مشاهده لینک ، لطفا با نام کاربری خود وارد شوید یا ثبت نام کنید ]) 3 ([ برای مشاهده لینک ، لطفا با نام کاربری خود وارد شوید یا ثبت نام کنید ]) 4 ([ برای مشاهده لینک ، لطفا با نام کاربری خود وارد شوید یا ثبت نام کنید ]) 5 ([ برای مشاهده لینک ، لطفا با نام کاربری خود وارد شوید یا ثبت نام کنید ]) ... آخرين صفحه ([ برای مشاهده لینک ، لطفا با نام کاربری خود وارد شوید یا ثبت نام کنید ]))


البته همینجا هم بپرسین شاید دوستانی باشن که جواب بدن.


موفق باشین.
91/1/26
اتفاقا عزیز مربوط به فیزیک یک و دو هست :دی شانس نداریم کلا:دی
ممنون از پاسخت

davy jones
14-04-2012, 19:40
[ برای مشاهده لینک ، لطفا با نام کاربری خود وارد شوید یا ثبت نام کنید ] %5Cleft%20%5C%7B%20x%7Cx%20%5Cin%20_.%7BA%7Di,i%5C in%20A%20For

میشه یک توضیحی بدهید؟
نمیتونم چرا نتونستم درست بنویسیم این مجموعه رو
این for هم یعنی به ازای حداقل یک
i هم اندیس هست یکم اومد بالا تر :13:
نقطه ها هم برای نظم گذاشتم
تو کتاب این نوشته میشه مثالی بزنید برای تفهیم
فرض میکنیم I مجموعه ای ناتهی و U یک عالم باشد.به ازای هر [ برای مشاهده لینک ، لطفا با نام کاربری خود وارد شوید یا ثبت نام کنید ] فرض میکنیم [ برای مشاهده لینک ، لطفا با نام کاربری خود وارد شوید یا ثبت نام کنید ]آنگاه مجموعه I را یک مجموعه اندیس گذار و هر [ برای مشاهده لینک ، لطفا با نام کاربری خود وارد شوید یا ثبت نام کنید ] یک اندیس گویند.

سلام.

این در حقیقت یک تعریف هستش و نه یک قضیه.

داره میگه چطوری باید اعضای یک مجموعه رو شماره گذاری و تفکیک کنیم.

اگر U یک عالم مرجع باشه، پس هر چیزی که داخل A باشه، عضو U هم هست چون A هر چه که باشه، زیر مجموعه ی U میشه. پس اگر اعضای مختلف A رو به صورت [ برای مشاهده لینک ، لطفا با نام کاربری خود وارد شوید یا ثبت نام کنید ] نشون بدیم، به ازای هر i داریم: [ برای مشاهده لینک ، لطفا با نام کاربری خود وارد شوید یا ثبت نام کنید ]

حالا اگه بیایم و یک مجموعه ی ناتهی مث I تعریف کنیم که توش فقط اعداد حسابی باشه، I رو میشه یک مجموعه ی اندیس گذار نامید چون در حقیقت اعضای I ، همون اندیسها هستند.


در کل چیز آنچنان مهمی نیست :31:



موفق باشین.
91/1/26

amir_rahmani
14-04-2012, 21:17
[ برای مشاهده لینک ، لطفا با نام کاربری خود وارد شوید یا ثبت نام کنید ] c%7BLn%20x+1%7D%7Bx-e%7D

آقا جواب این چی میشه؟
خیلی ممنون

Samba
14-04-2012, 21:51
[ برای مشاهده لینک ، لطفا با نام کاربری خود وارد شوید یا ثبت نام کنید ] c%7BLn%20x+1%7D%7Bx-e%7D

آقا جواب این چی میشه؟
خیلی ممنون

یه هوپیتال بزنی ساده حل میشه

جوابش میشه

1/e

که e مخرجه

ali_hp
14-04-2012, 22:55
[ برای مشاهده لینک ، لطفا با نام کاربری خود وارد شوید یا ثبت نام کنید ] c%7BLn%20x+1%7D%7Bx-e%7D

آقا جواب این چی میشه؟
خیلی ممنون
سلام ، اين حد كه وجود نداره ، صورت به دو ميل مي كنه و مخرج به صفر .... ولي اگه در صورت كسرجمع به تفريق تبديل بشه ، ههمونطور كه جناب samba عزيز فرمودن حل ميشه.

ramingsm
14-04-2012, 23:03
[ برای مشاهده لینک ، لطفا با نام کاربری خود وارد شوید یا ثبت نام کنید ] c%7BLn%20x+1%7D%7Bx-e%7D

آقا جواب این چی میشه؟
خیلی ممنون
این حد نامتناهی هست
حد چپ منفی بی نهایت و حد راست مثبت بی نهایت میشه
هوپیتال هم نمیشه استفاده کرد چون هوپیتال مال یکی از صور مبهم 0/0 یا بینهایت تقسیم بر بینهایت هست

skyzare
14-04-2012, 23:08
با سلام .


اساتید این سوال زیر رو ببینید . جوابش هم دارم ولی نمیدونم اون قسمت سبز رنگ چه جوری به دست اومده ؟

سوال :

ثابت کنید تمامی بردارهای یک مجموعه متعامد ( Orthogonal ) ، مستقل خطی هستند .


پاسخ :


[ برای مشاهده لینک ، لطفا با نام کاربری خود وارد شوید یا ثبت نام کنید ]


[ برای مشاهده لینک ، لطفا با نام کاربری خود وارد شوید یا ثبت نام کنید ]

ali_hp
14-04-2012, 23:22
با سلام .


اساتید این سوال زیر رو ببینید . جوابش هم دارم ولی نمیدونم اون قسمت سبز رنگ چه جوری به دست اومده ؟

سوال :

ثابت کنید تمامی بردارهای یک مجموعه متعامد ( Orthogonal ) ، مستقل خطی هستند .


پاسخ :


[ برای مشاهده لینک ، لطفا با نام کاربری خود وارد شوید یا ثبت نام کنید ]


[ برای مشاهده لینک ، لطفا با نام کاربری خود وارد شوید یا ثبت نام کنید ]


سلام
خط قبل قسمت سبز رنگ m تا جمله هست ، که بجز جمله i ام ، بقیه جملات صفر میشن!دلیلشم خودش دقیقا بعد قسمت سبز رنگ گفته!

amir_rahmani
14-04-2012, 23:29
دوستان عذر می خواهم. سوال همون Ln x-1 هست (صورتش را می گم) و جواب هم همون 1/e هستش که e مخرجه
اما مشکل اینکه که باید از روشی غیر از hop حل بشه:(
تغییر متغیر فکر کنم جواب بده ولی من تا نصفه رفتم
یعنی x/e -1 را مساوی t قرار دادم (البته قبلش یک سری چیز میزم نوشتم!

ممنونم

ali_hp
14-04-2012, 23:52
دوستان عذر می خواهم. سوال همون Ln x-1 هست (صورتش را می گم) و جواب هم همون 1/e هستش که e مخرجه
اما مشکل اینکه که باید از روشی غیر از hop حل بشه:(
تغییر متغیر فکر کنم جواب بده ولی من تا نصفه رفتم
یعنی x/e -1 را مساوی t قرار دادم (البته قبلش یک سری چیز میزم نوشتم!

ممنونم
این حد تعریف مشتق تابع lnx در نقطه x=e هست!پس مقدار حد میشه مشتق تابع lnx در نقطه x=e.حالا نمی دونم از نظر معلم شما این راهی غیر از هوپیتال حساب میشه یا نه...!

amir_rahmani
15-04-2012, 09:14
آقا من این حد را تا جایی پیش رفتم
بقیش مونده
lim (ln x - ln e) / x-e

lim (ln x/e) / e(x/e -1

حالا تغییر متغیر:
x/e - 1 = t

حالا

Lim (ln t + 1) / et
در اینجا t میل می کند 0
از این به بعدش؟؟؟

skyzare
15-04-2012, 13:28
با سلام .


اساتید یه سوال داشتم .


فرض کنید یه ماتریس داریم و میام پایه مربوط به یکی از فضای ماتریس رو به دست میارم مثلا پایه های مربوط به فضای null (‌ فضای پوچ )‌ رو به دست میارم .

حالا بعد با فرایند گرام اشمیت میام و پایه های متعامد این پایه ها رو به دست میاریم . الان این پایه های متعامد چه خاصیتی دارند ؟ یعنی من اگه چند تا بردار از ترکیب خطی این پایه ها بنویسم با هم عمود هستند ؟

یعنی منظورم این هست که همه ترکیب های خطی که از پایه های متعامد به دست میاد به هم عمود هستن این جوری هست ؟


اصلا حالا یعنی چی این پایه ها رو متعامد کنیم :31:

hts1369
15-04-2012, 14:35
سلام
این ماتریس نشون دهند ه ی یک دستگاه سه معادله و سه مجهول هست و اون j مربوط به اعداد مختلط هست.

[ برای مشاهده لینک ، لطفا با نام کاربری خود وارد شوید یا ثبت نام کنید ][&space;%5Cbegin%7Barray%7D%7Bccc%7D&space;6j+1&space;&&space;-2j&space;&&space;-4j&space;%5C%5C&space;-2j&space;&&space;j+1&space;&&space;j&space;%5C%5C&space;-4j&space;&&space;j&space;&&space;-j+1&space;%5Cend%7Barray%7D&space;%5Cright]%5Cleft[&space;%5Cbegin%7Barray%7D%7Bc%7D&space;e_1&space;%5C%5C&space;e_2&space;%5C%5C&space; e_3&space;%5Cend%7Barray%7D&space;%5Cright]=%5Cleft[&space;%5Cbegin%7Barray%7D%7Bc%7D&space;3%5Cmeasuredangle&space;30&space;% 5C%5C&space;0&space;%5C%5C&space;0&space;%5Cend%7Barray%7D&space;%5Cright]

میخواستم بدونم بهترین و سریعترین روش حل اینجور دستگاهها چی هست؟
با اعمال سطری مقدماتی خیلی سریع میتونم دستگاههایی که اعداد مختلط ندارن رو حساب کنم ولی اینجور ماتریسها رو هر کاری میکنم نمیتونم اونجوری حلشون بکنم.
ممنونم که دوستان راهنمایی میکنن.

ali_hp
15-04-2012, 15:01
با سلام .


اساتید یه سوال داشتم .


فرض کنید یه ماتریس داریم و میام پایه مربوط به یکی از فضای ماتریس رو به دست میارم مثلا پایه های مربوط به فضای null (‌ فضای پوچ )‌ رو به دست میارم .

حالا بعد با فرایند گرام اشمیت میام و پایه های متعامد این پایه ها رو به دست میاریم . الان این پایه های متعامد چه خاصیتی دارند ؟ یعنی من اگه چند تا بردار از ترکیب خطی این پایه ها بنویسم با هم عمود هستند ؟

یعنی منظورم این هست که همه ترکیب های خطی که از پایه های متعامد به دست میاد به هم عمود هستن این جوری هست ؟


اصلا حالا یعنی چی این پایه ها رو متعامد کنیم :31:


سلام
نه لزو ما نمیشه ، مثلا مضرب یک بردار بر خودش عمود نیست!
اما: هر بردار بر هر تر کیب خطی از بقیه بر دار ها عمود میشه.اثباتش سادست ، کافیه ثابت کنید ضرب داخلی این بردار در هر ترکیب خطی از بقیه بردارها صفر میشه.
وقتي كه عناصر يك پايه رو متعامد مي كنيم ، يعني از روي انها يك پايه جديد مي سازيم كه اعضاي اين پايه دو به دو بر هم عمودن ، يا به عبارت ديگه ضرب داخليشون صفر ميشه.
مثال پایه های متعامد یکه ، بردارهای واحد در جهت محور های مختصاتی هستن...مثلا سه بردار i و j  و k د فضای R^3  که از قدیم اشنا هستیم باهاشون...و خواص زیرو دارن:
یک) ترکیبات خطی شون کل فضا رو تولید می کنه.
دو)هر عضو فضا رو فقط به یک شکل میشه به صورت ترکیبات خطی اونا نوشت.(مستقل خطی هستند)
سه)بر هم عمودن!
چهار)یکه هستند!
ویژگی اول باعث میشه که ما بتونیم فضا رو با این بردارها توصیف کنیم.
ویژگی دوم باعث میشه که توصیفمون منحصر به فرد باشه و بی ابهام!یعنی از روی شکل ظاهری دو توصیف متوجه بشیم که هر دوشون یه موجودو توصیف می کنن یا نه!
ویژگی سوم فعلا! از نظر محاسباتی و هندسی جالبه!چون نمایش هر بردارو نسبت به یک پایه متعامد یکه به سادگی میشه بدست میاد... و هر بردار برابر مجموع تصاویرش روی عناصر یک پایه متعامد است...
وقتی ما یک پایه متعامد یکه n  عضوی برای یک فضا بدست بیاریم ، و فضا رو با این پایه توصیف کنیم ، این فضا جبرخطیش ! و هندسش!(البته هنوز تا حدودی!) میشه مثل R^n !  عناصر پایه متعامد ، نقش بردارهای یکه مختصاتی رو دارن...و ما دید و درک مناسبی از این فضا خواهیم داشت!

skyzare
15-04-2012, 15:21
سلام
این ماتریس نشون دهند ه ی یک دستگاه سه معادله و سه مجهول هست و اون j مربوط به اعداد مختلط هست.

[ برای مشاهده لینک ، لطفا با نام کاربری خود وارد شوید یا ثبت نام کنید ][&space;%5Cbegin%7Barray%7D%7Bccc%7D&space;6j+1&space;&&space;-2j&space;&&space;-4j&space;%5C%5C&space;-2j&space;&&space;j+1&space;&&space;j&space;%5C%5C&space;-4j&space;&&space;j&space;&&space;-j+1&space;%5Cend%7Barray%7D&space;%5Cright]%5Cleft[&space;%5Cbegin%7Barray%7D%7Bc%7D&space;e_1&space;%5C%5C&space;e_2&space;%5C%5C&space; e_3&space;%5Cend%7Barray%7D&space;%5Cright]=%5Cleft[&space;%5Cbegin%7Barray%7D%7Bc%7D&space;3%5Cmeasuredangle&space;30&space;% 5C%5C&space;0&space;%5C%5C&space;0&space;%5Cend%7Barray%7D&space;%5Cright]

میخواستم بدونم بهترین و سریعترین روش حل اینجور دستگاهها چی هست؟
با اعمال سطری مقدماتی خیلی سریع میتونم دستگاههایی که اعداد مختلط ندارن رو حساب کنم ولی اینجور ماتریسها رو هر کاری میکنم نمیتونم اونجوری حلشون بکنم.
ممنونم که دوستان راهنمایی میکنن.

با سلام .

من نمیدونم سریع ترین روش چیه . ولی این ماتریس ها رو هم فکر کنم بشه با اعمال سطری مقدماتی به دست اورد . فکر نمیکنم فرقی داشته باشه برای راحتی هم بعد از به دست اورد ماتریس افزوده ......فکر کنم اگه تمام اعدادش رو توی فرم قطبی ببریم بهتر باشه . بعد هم با همون روش حذفی گوس ماتریس افزوده رو به بالا مثلثی تبدیل کنیم و در نهایت حلش کنی . البته نمیدونم چیزی که گفتم درسته یا نه [ برای مشاهده لینک ، لطفا با نام کاربری خود وارد شوید یا ثبت نام کنید ]


========================================


با سلام .

با تشکر از پاسخ شما .

من متوجه نشدم این قسمت چی شد . [ برای مشاهده لینک ، لطفا با نام کاربری خود وارد شوید یا ثبت نام کنید ]





ویژگی دوم باعث میشه که توصیفمون منحصر به فرد باشه و بی ابهام!یعنی از روی شکل ظاهری دو توصیف متوجه بشیم که هر دوشون یه موجودو توصیف می کنن یا نه!
ویژگی سوم فعلا! از نظر محاسباتی و هندسی جالبه!چون نمایش هر بردارو نسبت به یک پایه متعامد یکه به سادگی میشه بدست میاد... و هر بردار برابر مجموع تصاویرش روی عناصر یک پایه متعامد است...
وقتی ما یک پایه متعامد یکه n عضوی برای یک فضا بدست بیاریم ، و فضا رو با این پایه توصیف کنیم ، این فضا جبرخطیش ! و هندسش!(البته هنوز تا حدودی!) میشه مثل R^n ! عناصر پایه متعامد ، نقش بردارهای یکه مختصاتی رو دارن...و ما دید و درک مناسبی از این فضا خواهیم داشت!



یعنی گنگ تر از این جبر خطی ندیدم :31: بخصوص این فضا مضاهاش :31:

mjorh
15-04-2012, 17:17
سلام
بچه ها ممنون میشم یه دستی بدید:20:

خمیدگی دلوار (r=a(1-costرا بیابید

اون t , تتا هستش

hts1369
15-04-2012, 19:22
با سلام .

من نمیدونم سریع ترین روش چیه . ولی این ماتریس ها رو هم فکر کنم بشه با اعمال سطری مقدماتی به دست اورد . فکر نمیکنم فرقی داشته باشه برای راحتی هم بعد از به دست اورد ماتریس افزوده ......فکر کنم اگه تمام اعدادش رو توی فرم قطبی ببریم بهتر باشه . بعد هم با همون روش حذفی گوس ماتریس افزوده رو به بالا مثلثی تبدیل کنیم و در نهایت حلش کنی . البته نمیدونم چیزی که گفتم درسته یا نه [ برای مشاهده لینک ، لطفا با نام کاربری خود وارد شوید یا ثبت نام کنید ]

فکر میکنی نمیدونم لامصب چه سحری داره اصلا نمیشه حلش کرد وقتی میخواهی سطری مقدماتی حلش کنی به جایی میرسی که دو سطر مثله هم هستن.
نمیدونم چرا اینجوری میشه .
راستی بچه ها تو Mathematica چطوری میشه اینجور دستگاهها رو حل کرد.

ashjaee
15-04-2012, 20:27
سلام
بچه ها ممنون میشم یه دستی بدید:20:

خمیدگی دلوار (r=a(1-costرا بیابید

اون t , تتا هستش

اول مختصات قطبی رو تبدیل به دکارتی می کنیم:

[ برای مشاهده لینک ، لطفا با نام کاربری خود وارد شوید یا ثبت نام کنید ]
[ برای مشاهده لینک ، لطفا با نام کاربری خود وارد شوید یا ثبت نام کنید ]

حالا داریم [ برای مشاهده لینک ، لطفا با نام کاربری خود وارد شوید یا ثبت نام کنید ]

[ برای مشاهده لینک ، لطفا با نام کاربری خود وارد شوید یا ثبت نام کنید ]

[ برای مشاهده لینک ، لطفا با نام کاربری خود وارد شوید یا ثبت نام کنید ]

طبق فرمول داریم:

[ برای مشاهده لینک ، لطفا با نام کاربری خود وارد شوید یا ثبت نام کنید ] 5E2%5Ctheta&space;-&space;3sin%5Ctheta&space;sin2%5Ctheta&space;+&space;2sin%5E2&space;2%5Ctheta%29 &space;-&space;a%5E2%28-cos%5E2%5Ctheta&space;+&space;3&space;cos%5Ctheta&space;cos2%5Ctheta&space;-2cos%5E2&space;2%5Ctheta%29%29k%5Cright&space;%7C%7D%7B%5Cleft &space;%7C&space;%7Br%7D%27&space;%5Cright&space;%7C%5E3%7D


ساده که بکنیم داریم:
[ برای مشاهده لینک ، لطفا با نام کاربری خود وارد شوید یا ثبت نام کنید ] theta&space;%29%29%7D%7B%28a%5Csqrt%7B2%281&space;-&space;%28sin%5Ctheta&space;sin2&space;%5Ctheta&space;+&space;cos&space;%5Ctheta&space;cos2% 5Ctheta%29%7D%29%5E%5Cfrac%7B3%7D%7B2%7D%7D

ساده تر:
[ برای مشاهده لینک ، لطفا با نام کاربری خود وارد شوید یا ثبت نام کنید ]

جواب آخر:
[ برای مشاهده لینک ، لطفا با نام کاربری خود وارد شوید یا ثبت نام کنید ] Csqrt%7B1-cos%5Ctheta%7D%7D

البته ممکن هست اشتباه محاسباتی داشته یاشه

nits
15-04-2012, 20:31
سلام
خسته نباشید به همه دوستان
دوستان میشه چندین روش یا متد ریاضی برای تعیین اول بودن یا نبودن یک عدد ارئه بدید ؟ راستش من باید یک متد ریاضی ارائه بدم و بعد الگوریتمش رو بنویسم و و بعد برنا مه ش رو
البته خودم یک روشی یادم مونده
مرسی دوستان

---------- Post added at 09:31 PM ---------- Previous post was at 09:29 PM ----------

منظورم اینه که سریع ترین راه رو میخوام ........ یعنی وقتی عدد بزرگ باشه یک راه حل سریع ارئه بدیم
مرسی

davy jones
15-04-2012, 21:49
سلام
خسته نباشید به همه دوستان
دوستان میشه چندین روش یا متد ریاضی برای تعیین اول بودن یا نبودن یک عدد ارئه بدید ؟ راستش من باید یک متد ریاضی ارائه بدم و بعد الگوریتمش رو بنویسم و و بعد برنا مه ش رو
البته خودم یک روشی یادم مونده
مرسی دوستان

---------- Post added at 09:31 PM ---------- Previous post was at 09:29 PM ----------

منظورم اینه که سریع ترین راه رو میخوام ........ یعنی وقتی عدد بزرگ باشه یک راه حل سریع ارئه بدیم
مرسی

سلام.

قبلا تاپیک در موردش وجود داشته.

خدمت شما:

[ برای مشاهده لینک ، لطفا با نام کاربری خود وارد شوید یا ثبت نام کنید ]

موفق باشین.
91/1/27

skyzare
16-04-2012, 07:22
با سلام .

اساتید این تعریف زیر رو یه لطفی می کنید ، نگاه کنید به نظرتون درست هست ؟



[ برای مشاهده لینک ، لطفا با نام کاربری خود وارد شوید یا ثبت نام کنید ]

ali_hp
17-04-2012, 16:20
با سلام
.

با تشکر از پاسخ شما .

من متوجه نشدم این قسمت چی شد .





یعنی گنگ تر از این جبر خطی ندیدم بخصوص این فضا مضاهاش جبر خطي تا وقتي كه شهودي ازش نداشته باشيم گنگه ، ولي اگه شهود داشته باشيم از مفاهيمش ، گنگ نيست !
اين چهارتا ويژگي ، كه ما بطور اگاهانه و نا اگاهانه در دستگاه مختصات دكارتي ازونا استفاده مي كنيم ، باعث ميشن كه اين دستگاه مختصات دكارتي براي ما مطلوب باشه ، و راحت باهاش كار كنيم و درك خوبي ازش داشته باشيم .
و تلاش ما براي پيد كردن مولد ، پايه ، پايه يكا متعامد.... براي فضاهاي مختلف ديگه هم براي اينه كه بهتر توصيفشون كنيم ، راحت تر باهاشون كار كنيم ، و درك بهتري ازشون داشته باشيم. وقتي يك پايه يك متعامد براي يك فضا پيدا كرديم ديگه اين فضا براي ما كاملا ملموسه.چون فضامون ميشه يه چيزي مثل فضاي دكارتي ، كه عناصر پايه يكا متعامد نقش بردارهاي يكه مختصاتي رو بازي مي كنن !

ويژگي دوم يعني اينكه مثلا اگر دو نقطه 2 i + 3j + k و i + 2j + 4k رو داشته باشيم مي دونيم كه قطعا اينا دو مو جود متفاوت در فضا هستند!


در صورتي كه اگر بردارهاي :


i , j ,k

مستقل خطي نبودند ممكن بود اين دو يك موجود در فضا باشند!
ويژگي سوم :
از نظر محاسباتي كار كردن با پايه متعامد يكه راحت تره : مثلا شما براي بدست اوردن نمايش يك بردار نسبت به يك پايه بايد دستگاه معادلات حل كنيد . در صورتي كه نمايش يك بردار نسبت به يك پايه متعامد يكه با چند تا ضرب داخلي بدست مياد .
از نظر هندسی و دید ما!هم عمود بودن پایه ها خیلی خوبه!شما فرض کن مثلا توی دستگاه مختصات سه بعدی محور هایی که میگرفتیم بر هم عمود نبودن....!خوب نبود دیگه !
یه مثال میزنم :
فرض كنيد A ماتريسي 3*3 باشه ، با رتبه يك . پس فضاي پوچ اين ماتريس يك زير فضاي دو بعدي از فضا ميشه .
يعني يك صفحه گذرنده از مبدا. حالا مثلا فرض كنيد اين ماتريس ماتريسي باشه كه همه درايه هاش يك هستند . به وضوح فضاي پو چ اين ماتريس ميشه صفحه x+y+z=0 .حالا ميايم يك پايه براي اين فضا پيدا مي كنيم ، مثلا:
(1 , 1 , -2) , (1 , -2 , 1 )
پس ما هر عضو فضای پوچو میتونیم به صورت ترکیب خطی از این دو بردار بنویسیم. مثلا نقطه :
(1 , 4 ,5-)
به طور شهودی میتونیم فرض کنیم اعضای پایه گامهای ما هستند!که هر کدوم اندازه و جهت(؟) خاصی دارند .
پس ما اینجا دو نوع گام داریم!حالا برای اینکه درکی از نقطه
(1 , 4 ,5-)
در فضای پوچ داشته باشیم ، باید مشخص کنیم که با چه ترکیبی از این گامها میشه به اون نقطه رسید ...
که منجر به یک دستگاه سه معادله دو مجهول میشه ...
حالا اگر برای فضای پوچ یک پایه یک متعامد ارایه کنیم ، یعنی دو بردار یکه عمود در صفحه x+y+z پیدا کنیم...مثلا فرض کنید a , b این دو تا بردار باشن.


و c نقطه ای دیگر در فضای پوچ باشد. برای بدست اوردن نمایش c بر حسب a , b کافی است c.a , c.b را حساب کنیم(ضرب داخلی)
و نمایش c چنین خواهد بود:

برای مشاهده محتوا ، لطفا وارد شوید یا ثبت نام کنید
و نیازی به حل دستگاه سه معادله دو مجهمول نیست .
از طرفی دیگر اگر راستای a , b را به عنوان محور x ها و y ها در نظر بگیریم ، فضای پوچ ما دقیقا مثل همان صفحه دکارتی خواهد بود!

ali_hp
17-04-2012, 16:55
با سلام .

اساتید این تعریف زیر رو یه لطفی می کنید ، نگاه کنید به نظرتون درست هست ؟



[ برای مشاهده لینک ، لطفا با نام کاربری خود وارد شوید یا ثبت نام کنید ]






سلام ، به نظر من اولی غلطه و دومی و سومی درسته!
مثال نقض اولی:یک دستگاه دو معادله سه مجهول در نظر بگیرید ،(دو معادله مستقل باشند) و b=0 بگیرید!حالا برای ماتریس مربوطه شرایط اولی برقراره ، ولی دستگاه بی نهایت جواب داره!(اشتراک دو صفحه غیر موازی در فضا ، یک خط میشه که بی نهایت نقطه داره!)
فکر می کنم اگه توی اولی و دومی به جای min(m.n)l خود n بزارین ، همه چی درست میشه!

ali_hp
17-04-2012, 17:17
فکر میکنی نمیدونم لامصب چه سحری داره اصلا نمیشه حلش کرد وقتی میخواهی سطری مقدماتی حلش کنی به جایی میرسی که دو سطر مثله هم هستن.
نمیدونم چرا اینجوری میشه .
راستی بچه ها تو Mathematica چطوری میشه اینجور دستگاهها رو حل کرد.
سلام
خوب این که مشکلی نیست، بازم روش سطری مقدماتی رو ادامه بده...!؟یه سطر کامل صفر میشه...بازم ادامه بده!

mojtaba2321
17-04-2012, 19:59
دامنه ln ها رو میشه یکی ساده برای من توضیح بده؟ فردا امتحانشو دارم.:41:

ali_hp
17-04-2012, 21:36
دامنه ln ها رو میشه یکی ساده برای من توضیح بده؟ فردا امتحانشو دارم.:41:
سلام
ln که مبناش ثابته ، فقط کافیه عبارت داخل ln بزرگتر مساوی صفر باشه.
یعنی دامنمون میشه:
دامنه خود عبارت داخل ln اشتراکش با مقادیری که عبارت داخل ln بزرگتر مساوی صفر میشه.
اگر هم log داشتیم که داخل مبناش هم متغیر بود ، مثل بالا ، فقط باید مجموعه بدست اومدرو با دامنه مبنا هم اشتراک بگیریم و همچنین با مقادیری که مبنا هم بزرگتر اکید از صفر میشه هم اشتراک بگیریم و اخرشم مقادیری که مبنا به ازای اونا یک میشه رو از مجموعه بدست اومده کم کنیم.
به طور خلاص:
مبنا مثبت و مخالف یک.
عبارت داخل ln : نامنفی.

skyzare
19-04-2012, 20:50
ويژگي سوم :
از نظر محاسباتي كار كردن با پايه متعامد يكه راحت تره : مثلا شما براي بدست اوردن نمايش يك بردار نسبت به يك پايه بايد دستگاه معادلات حل كنيد . در صورتي كه نمايش يك بردار نسبت به يك پايه متعامد يكه با چند تا ضرب داخلي بدست مياد .
از نظر هندسی و دید ما!هم عمود بودن پایه ها خیلی خوبه!شما فرض کن مثلا توی دستگاه مختصات سه بعدی محور هایی که میگرفتیم بر هم عمود نبودن....!خوب نبود دیگه !
یه مثال میزنم :
فرض كنيد A ماتريسي 3*3 باشه ، با رتبه يك . پس فضاي پوچ اين ماتريس يك زير فضاي دو بعدي از فضا ميشه .
يعني يك صفحه گذرنده از مبدا. حالا مثلا فرض كنيد اين ماتريس ماتريسي باشه كه همه درايه هاش يك هستند . به وضوح فضاي پو چ اين ماتريس ميشه صفحه x+y+z=0 .حالا ميايم يك پايه براي اين فضا پيدا مي كنيم ، مثلا:
(1 , 1 , -2) , (1 , -2 , 1 )
پس ما هر عضو فضای پوچو میتونیم به صورت ترکیب خطی از این دو بردار بنویسیم. مثلا نقطه :
(1 , 4 ,5-)
به طور شهودی میتونیم فرض کنیم اعضای پایه گامهای ما هستند!که هر کدوم اندازه و جهت(؟) خاصی دارند .
پس ما اینجا دو نوع گام داریم!حالا برای اینکه درکی از نقطه
(1 , 4 ,5-)
در فضای پوچ داشته باشیم ، باید مشخص کنیم که با چه ترکیبی از این گامها میشه به اون نقطه رسید ...
که منجر به یک دستگاه سه معادله دو مجهول میشه ...
حالا اگر برای فضای پوچ یک پایه یک متعامد ارایه کنیم ، یعنی دو بردار یکه عمود در صفحه x+y+z پیدا کنیم...مثلا فرض کنید a , b این دو تا بردار باشن.


و c نقطه ای دیگر در فضای پوچ باشد. برای بدست اوردن نمایش c بر حسب a , b کافی است c.a , c.b را حساب کنیم(ضرب داخلی)
و نمایش c چنین خواهد بود:

برای مشاهده محتوا ، لطفا وارد شوید یا ثبت نام کنید
و نیازی به حل دستگاه سه معادله دو مجهمول نیست .
از طرفی دیگر اگر راستای a , b را به عنوان محور x ها و y ها در نظر بگیریم ، فضای پوچ ما دقیقا مثل همان صفحه دکارتی خواهد بود!




با سلام .

بی نهایت سپاس گذار از بابت پاسخ گویی شما .

ببخشید مگه برای فضای برداری پایه هاش منحصر به فرد نیست ؟ اخه این جوری که

شما گفتید مثل این که پایه های یک فضا مثلا فضای پوچ می تونه منحصر به فرد نباشه . اصلا پایه های فضا

منحصر به فرد هست ؟


=========================================


ویرایش :

در واقع منظورم این هست که مثلا توی این مثال پایینی که شما زدید خوب بعد فضای نول 2 هست حالا منهر دو برداری رو که توی این رابطه زیر صدق کنه می تونم به عنوان بردارهای پایه فضای نول در نظر بگیرم ؟ ( البته با این فرض که اون دو بردار شرایط پایه بودن که مستقل خطی باشه و فضای مربوطه رو اسپن کنه داشته باشه )


به وضوح فضاي پو چ اين ماتريس ميشه صفحه x+y+z=0 .حالا ميايم يك پايه براي اين فضا پيدا مي كنيم ، مثلا:
(1 , 1 , -2) , (1 , -2 , 1 )

skyzare
21-04-2012, 08:59
با سلام .

اساتید فرض کنید یه ماتریس A داریم و بردارهای ویژه اش و مقادیر ویژه اش رو به دست اوردیم . مثلا مثل زیر :

الان ستون های Eigenvector در واقع بردارهای ویژه ماتریس A هستند و ستون ها Eigenvalue هم مقادیر ویژه ماتریس A هستند . ( البته فقط همون عدد غیر صفرش )

ولی این بردار های ویژه یه صورت یکامتعامد هست به نظرتون چه جوری میشه فهمید اون بردار

اصلی چی بوده که بعد از یکامتعامد این جوری شده ؟




برای مشاهده محتوا ، لطفا وارد شوید یا ثبت نام کنید

mf79292
21-04-2012, 19:01
سلام لطفا یکی بگه مشتق مرتبه دوم y نسبت به x در معادله پارامتری زیر چی میشه؟ خیر ببینین...

x=t^2
y=t ln t

skyzare
21-04-2012, 19:33
سلام لطفا یکی بگه مشتق مرتبه دوم y نسبت به x در معادله پارامتری زیر چی میشه؟ خیر ببینین...

x=t^2
y=t ln t

با سلام .



[ برای مشاهده لینک ، لطفا با نام کاربری خود وارد شوید یا ثبت نام کنید ] ac%7B%5Cfrac%7Bdy%7D%7Bdt%7D%7D%7B%5Cfrac%7Bdx%7D% 7Bdt%7D%7D=%5Cfrac%7Bln%28t%29+1%7D%7B2t%7D%5C%5C% 5C%5C%5C%5C&space;y%27%27=%5Cfrac%7Bd%5E%7B%5C:&space;2%7Dy%7D %7Bdx%5E2%7D=%5Cfrac%7Bd%28y%5C:&space;%27%29%7D%7Bd%28x %29%7D=%5Cfrac%7Bdy%5C:&space;%27%7D%7Bdt%7D%5Ctimes&space;%5C frac%7Bdt%7D%7Bdx%7D=%5Cfrac%7B%5Cfrac%7Bdy%27%7D% 7Bdt%7D%7D%7B%5Cfrac%7Bdx%7D%7Bdt%7D%7D%5C%5C%5C%5 C%5C%5C&space;%5Cfrac%7Bdy%27%7D%7Bdt%7D=%5Cfrac%7B-ln%28t%29%7D%7B2t%5E2%7D%5C%5C%5C%5C%5C%5C&space;%5Cfrac %7Bdx%7D%7Bdt%7D=2t



[ برای مشاهده لینک ، لطفا با نام کاربری خود وارد شوید یا ثبت نام کنید ]



===========================================

روش کلی اینه حالا محاسبات رو خودتون چک کنید .

پ . ن : لزومی نداره یه سوال رو چند جا بپرسید . باور کنید یه جا بپرسید هم کاربر ها می بینند .:31:

قاهر - Gahir
22-04-2012, 08:49
سلام بر دوستان گرامی؛

کسی اثبات استقرائی برای:
اگر[ برای مشاهده لینک ، لطفا با نام کاربری خود وارد شوید یا ثبت نام کنید ]{i} ها اعداد حقیقی مثبتی از 1 تا n و [ برای مشاهده لینک ، لطفا با نام کاربری خود وارد شوید یا ثبت نام کنید ]{1}\leqslant%20P_{2} باشد، آنگاه :

[ برای مشاهده لینک ، لطفا با نام کاربری خود وارد شوید یا ثبت نام کنید ]{(\sum_{i=1}^{n}x_{i}^{P_ {2}})^{\frac{1}{P_{2}}}\leqslant%20(\sum_{i=1}^{n} x_{i}^{P_{1}})^{\frac{1}{P_{1}}}}
سراغ داره ؟
من خودم از یه روش برای اثبات k+1 امین جمله‌ش پیش رفتم ولی به نتیجه‌ی مورد نظر نرسیدم ... یعنی نتونستم جواب رو ازش بکشم بیرون ...
اگه دوست دارید روشم اینه :
به ترتیب به طرف چپ [ برای مشاهده لینک ، لطفا با نام کاربری خود وارد شوید یا ثبت نام کنید ]{k+1}^{P_{2}} و به طرف راست [ برای مشاهده لینک ، لطفا با نام کاربری خود وارد شوید یا ثبت نام کنید ]{k+1}^{P_{1}} رو کم و زیاد میکنیم.

اثبات غیر استقرائیش آسونه ...

davy jones
22-04-2012, 10:49
سلام بر دوستان گرامی؛

کسی اثبات استقرائی برای:
اگر[ برای مشاهده لینک ، لطفا با نام کاربری خود وارد شوید یا ثبت نام کنید ] ها اعداد حقیقی مثبتی از 1 تا n و [ برای مشاهده لینک ، لطفا با نام کاربری خود وارد شوید یا ثبت نام کنید ] _%7B2%7D باشد، آنگاه :

[ برای مشاهده لینک ، لطفا با نام کاربری خود وارد شوید یا ثبت نام کنید ] D%5E%7Bn%7Dx_%7Bi%7D%5E%7BP_%7B2%7D%7D%29%5E%7B%5C frac%7B1%7D%7BP_%7B2%7D%7D%7D%5Cleqslant%20%28%5Cs um_%7Bi=1%7D%5E%7Bn%7Dx_%7Bi%7D%5E%7BP_%7B1%7D%7D% 29%5E%7B%5Cfrac%7B1%7D%7BP_%7B1%7D%7D%7D%7D
سراغ داره ؟
من خودم از یه روش برای اثبات k+1 امین جمله‌ش پیش رفتم ولی به نتیجه‌ی مورد نظر نرسیدم ... یعنی نتونستم جواب رو ازش بکشم بیرون ...
اگه دوست دارید روشم اینه :
به ترتیب به طرف چپ [ برای مشاهده لینک ، لطفا با نام کاربری خود وارد شوید یا ثبت نام کنید ] D و به طرف راست [ برای مشاهده لینک ، لطفا با نام کاربری خود وارد شوید یا ثبت نام کنید ] D رو کم و زیاد میکنیم.

اثبات غیر استقرائیش آسونه ...


[ برای مشاهده لینک ، لطفا با نام کاربری خود وارد شوید یا ثبت نام کنید ] %7Bk%7D%5E%7Bp_%7B1%7D%7D%29%5E%7B%5Cfrac%7B1%7D%7 Bp_%7B1%7D%7D%7D%5Cgeq&space;%28%5Csum_%7Bk=1%7D%5E%7Bn% 7Dx_%7Bk%7D%5E%7Bp_%7B2%7D%7D%29%5E%7B%5Cfrac%7B1% 7D%7Bp_%7B2%7D%7D%7D%5CRightarrow&space;[%28%5Csum_%7Bk=1%7D%5E%7Bn%7Dx_%7Bk%7D%5E%7Bp_%7B1 %7D%7D%29%5E%7B%5Cfrac%7B1%7D%7Bp_%7B1%7D%7D%7D]%5E%7Bp_%7B1%7Dp_%7B2%7D%7D%5Cgeq&space;[%28%5Csum_%7Bk=1%7D%5E%7Bn%7Dx_%7Bk%7D%5E%7Bp_%7B2 %7D%7D%29%5E%7B%5Cfrac%7B1%7D%7Bp_%7B2%7D%7D%7D]%5E%7Bp_%7B1%7Dp_%7B2%7D%7D%5CRightarrow&space;%28%5Csum _%7Bk=1%7D%5E%7Bn%7Dx_%7Bk%7D%5E%7Bp_%7B1%7D%7D%29 %5E%7Bp_%7B2%7D%7D%5Cgeq&space;%28%5Csum_%7Bk=1%7D%5E%7B n%7Dx_%7Bk%7D%5E%7Bp_%7B2%7D%7D%29%5E%7Bp_%7B1%7D% 7D

[ برای مشاهده لینک ، لطفا با نام کاربری خود وارد شوید یا ثبت نام کنید ] arrow&space;%28%5Csum_%7Bk=1%7D%5E%7Bn+1%7Dx_%7Bk%7D%5E% 7Bp_%7B1%7D%7D%29%5E%7Bp_%7B2%7D%7D%5Cgeq&space;%28%5Csu m_%7Bk=1%7D%5E%7Bn+1%7Dx_%7Bk%7D%5E%7Bp_%7B2%7D%7D %29%5E%7Bp_%7B1%7D%7D%5CRightarrow&space;%28%5Csum_%7Bk= 1%7D%5E%7Bn%7Dx_%7Bk%7D%5E%7Bp_%7B1%7D%7D+x_%7Bn+1 %7D%5E%7Bp_%7B1%7D%7D%29%5E%7Bp_%7B2%7D%7D%5Cgeq&space;% 28%5Csum_%7Bk=1%7D%5E%7Bn%7Dx_%7Bk%7D%5E%7Bp_%7B2% 7D%7D+x_%7Bn+1%7D%5E%7Bp_%7B2%7D%7D%29%5E%7Bp_%7B1 %7D%7D%5CRightarrow&space;%5Csum_%7Bj=0%7D%5E%7Bp_%7B2%7 D%7D%5Cbinom%7Bp_%7B2%7D%7D%7Bj%7D%28%5Csum_%7Bk=1 %7D%5E%7Bn%7Dx_%7Bk%7D%5E%7Bp_%7B1%7D%7D%29%5E%7Bp _%7B2%7D-j%7D%28x_%7Bn+1%7D%5E%7Bp_%7B1%7D%7D%29%5E%7Bj%7D% 5Cgeq&space;%5Csum_%7Bj=0%7D%5E%7Bp_%7B1%7D%7D%5Cbinom%7 Bp_%7B1%7D%7D%7Bj%7D%28%5Csum_%7Bk=1%7D%5E%7Bn%7Dx _%7Bk%7D%5E%7Bp_%7B2%7D%7D%29%5E%7Bp_%7B1%7D-j%7D%28x_%7Bn+1%7D%5E%7Bp_%7B2%7D%7D%29%5E%7Bj%7D



به ازای j=0 به حالت فرض استقرامون یعنی حالتی که n رو به n+1 ارتقا نداده بودیم میرسیم که فرض کردیم همواره برقراره. پس میتونیم سری رو از جمله ی بعدیش شروع کنیم:


[ برای مشاهده لینک ، لطفا با نام کاربری خود وارد شوید یا ثبت نام کنید ] 5E%7Bp_%7B2%7D%7D%5Cbinom%7Bp_%7B2%7D%7D%7Bj%7D%28 %5Csum_%7Bk=1%7D%5E%7Bn%7Dx_%7Bk%7D%5E%7Bp_%7B1%7D %7D%29%5E%7Bp_%7B2%7D-j%7D%28x_%7Bn+1%7D%5E%7Bp_%7B1%7D%7D%29%5E%7Bj%7D% 5Cgeq&space;%5Csum_%7Bj=1%7D%5E%7Bp_%7B1%7D%7D%5Cbinom%7 Bp_%7B1%7D%7D%7Bj%7D%28%5Csum_%7Bk=1%7D%5E%7Bn%7Dx _%7Bk%7D%5E%7Bp_%7B2%7D%7D%29%5E%7Bp_%7B1%7D-j%7D%28x_%7Bn+1%7D%5E%7Bp_%7B2%7D%7D%29%5E%7Bj%7D


که ظاهرا اثبات آخری آسون به نظر نمیاد :31: مغزم دیگه هنگ کرد راستیاتش. :31:

یعنی واقعا اینقدر سخته از راه استقرا بخوایم بریم؟ :41:


موفق باشین.
91/2/3

ali_hp
22-04-2012, 13:15
با سلام .

بی نهایت سپاس گذار از بابت پاسخ گویی شما .

ببخشید مگه برای فضای برداری پایه هاش منحصر به فرد نیست ؟ اخه این جوری که

شما گفتید مثل این که پایه های یک فضا مثلا فضای پوچ می تونه منحصر به فرد نباشه . اصلا پایه های فضا

منحصر به فرد هست ؟


=========================================


ویرایش :

در واقع منظورم این هست که مثلا توی این مثال پایینی که شما زدید خوب بعد فضای نول 2 هست حالا منهر دو برداری رو که توی این رابطه زیر صدق کنه می تونم به عنوان بردارهای پایه فضای نول در نظر بگیرم ؟ ( البته با این فرض که اون دو بردار شرایط پایه بودن که مستقل خطی باشه و فضای مربوطه رو اسپن کنه داشته باشه )
بله ، میتونیم!کلا لزومی بر منحصر به فرد بودن پایه وجود نداره.مثلا در این مساله هر دو بردار غیر هم راستا که در صفحه x+y+z=0
در نظر بگیریم یک پایه برای فضای پوچ میشه.
در یک فضای دلخواه برای ساختن پایه میشه اینجوری عمل کرد:
با یک مجموعه از k بردار مستقل خطی شروع می کنیم.این مجموعه خود زیرفضایی k بعدی از فضای اصلی را تولید می کند.
اگر این زیر فضای تولید شده برابر کل فضا باشد که خودش پایه است و کار تمام است!
اگر هم کل فضا را تولید نکند ، یک بردار خارج از زیر فضای تولید شده در نظر می گیریم و به مجموعه مورد نظر اضافه می کنیم.مجموعه جدید نیز مستقل خطی است(چرا؟) و زیر فضایی k+1 بعدی تولید می کند.
اگر بعد فضای اصلیمون متناهی باشه ، و مثلا n باشه ، دیگه وقتی به n بردار مستقل خطی رسیدیم قطعا زیر فضای تولید شده توسط این n بردار کل فضا میشه و یک پایه بدست اوردیم.
مثلا در فضای دکارتی سه بعدی و حالت k=1 :
با یک تک بردار ناصفر شروع می کنیم.(یک مجموعه تک عضوی مستقل خطی)
زیر فضای تولید شده توسط این تک بردار یعنی یک خط!و برداری خارج از این زیر فضا یعنی هر بردار غیر هم راستا با ان.پس یک بردار غیر هم راستا به ان اضافه میکنیم.و به مجموعهای از دو بردار مستقل خطی می رسیم.زیر فضای تولید شده توسط این دو بردار یعنی یک صفحه(صفحه ای که از این دو بردار می گذرد.)
و در نهایت با در نظر گرفتن یک بردار خارج از این صفحه و اضافه کردن ان ، به یک پایه می رسیم.
با این شهود مشخصه که در فضاهای طبیعی چقدر پایه زیاد داریم!

ali_hp
22-04-2012, 13:54
با سلام .

اساتید فرض کنید یه ماتریس A داریم و بردارهای ویژه اش و مقادیر ویژه اش رو به دست اوردیم . مثلا مثل زیر :

الان ستون های Eigenvector در واقع بردارهای ویژه ماتریس A هستند و ستون ها Eigenvalue هم مقادیر ویژه ماتریس A هستند . ( البته فقط همون عدد غیر صفرش )

ولی این بردار های ویژه یه صورت یکامتعامد هست به نظرتون چه جوری میشه فهمید اون بردار

اصلی چی بوده که بعد از یکامتعامد این جوری شده ؟




برای مشاهده محتوا ، لطفا وارد شوید یا ثبت نام کنید
کدوم بردار اصلی منظورتونه؟
برای هر مقدار ویژه مجموعه همه بردارهای ویژه متناظرش ، یک زیر فضای برداری میسازن.که به این زیر فضا ، فضای ویژه گفته میشه.اینجا فضای ویژه متناظر با مقدار ویژه یک ، دو بعدی هست.و فضای ویژه متناظر با 3 ، یک بعدی.
اومده دو بردار ویژه از فضای ویژه متناظر 1 گرفته که یکه و عمودن . و یک بردار ویژه هم از فضای ویژه 3 گرفته که یکه باشه. که اتفاقا عمود بر دو تای قبلی هم هست....و البته ممکن بود نشه این بردار ویژه اخرو جوری گرفت که بر دو تای قبلی هم عمود باشه...

mojtaba2321
22-04-2012, 19:25
بچه ها یه مشکل خیلی پیش پاافتاده من دارم
اونم اینه که هنوز علامت بزرگتر کوچکتر رو قاطی میکنم:41:
یه روش خوب برای یا خاطر سپردن این علامت وجود داره؟ مث همون دهن ماهی که قبلا میگفتن

abcd20
22-04-2012, 19:51
بچه ها یه مشکل خیلی پیش پاافتاده من دارم
اونم اینه که هنوز علامت بزرگتر کوچکتر رو قاطی میکنم:41:
یه روش خوب برای یا خاطر سپردن این علامت وجود داره؟ مث همون دهن ماهی که قبلا میگفتن

کوچیک یا بزرگی بستگی داره شما از کدام طرف بخونی .

مثلا

3 > 2

شما اگه از سمت راست بخونید میشه :

3 بزرگتر از 2

اگه از سمت چب بخونید میشه :

2 کوچیکتر از 3


در واقع به این بستگی داره که از کدام طرف بخونید

Reza31001
22-04-2012, 20:07
بچه ها یه مشکل خیلی پیش پاافتاده من دارم
اونم اینه که هنوز علامت بزرگتر کوچکتر رو قاطی میکنم:41:
یه روش خوب برای یا خاطر سپردن این علامت وجود داره؟ مث همون دهن ماهی که قبلا میگفتن


کوچیک یا بزرگی بستگی داره شما از کدام طرف بخونی .

مثلا

3 > 2

شما اگه از سمت راست بخونید میشه :

3 بزرگتر از 2

اگه از سمت چب بخونید میشه :

2 کوچیکتر از 3


در واقع به این بستگی داره که از کدام طرف بخونید

و چون ریاضیات از چپ خونده میشه، > علامت کوچکتر از خواهد بود
کافیه همین مثال 3>2 رو حفظ کنی دیگه تا اخر عمر قاطی نمیکنی :46:

abcd20
22-04-2012, 20:19
و چون ریاضیات از چپ خونده میشه، > علامت کوچکتر از خواهد بود
:46:

شرمنده ولی به نظرم من دقیقا بستگی داره شما از کدام طرف بخونی . نمیشه گفت که حالا > علامت کوچیک هست یا بزرگ بستگی داره که سمت چپ و راستش چی باشه . :20:

davy jones
22-04-2012, 20:54
سلام.
به نظرم یه راه راحت برای حفظ کردن جهت علامت کوچکتر و بزرگتر اینه که همیشه دقت کنین که تیزی نوک علامت > یا < باید به سمت عدد کوچکتر باشه. مثل این میمونه که عدد بزرگتر انگار پشت یک سپر قرار گرفته و با یک نیزه که از وسط اون سپر بیرون زده داره عدد کوچکتر رو نابود میکنه :31:

موفق باشین.
91/2/3

sdfssdfs
23-04-2012, 17:03
چقد سطح مطالب بالااست :31:

skyzare
23-04-2012, 17:20
چقد سطح مطالب بالااست :31:
با سلام .
ببنید این جا هر کسی هر سوالی که توی ذهنش باشه می پرسه .( چه سخت ...چه اسون ) بنابراین نمیشه براش سطحی تعیین کرد که حالا سطح مطالب این جا بالا هست یا پایین . حالا ممکنه یه سوال از نظر شما اسون بیاد ولی خوب در هر صورت سوال هست .


============================================


پ .ن : کسی که سوال می کند برای لحظه ای کوتاه در جهل و نادانی هست ولی کسی که سوال نمیکند تا ابد در جهل و نادانی هست .

elahehamini
25-04-2012, 17:34
سلام.میشه یه نفر درون چرخزاد رو کامل برام حل کنه؟؟؟معادلات پارامتری اخرشو نی خوام.حل کاملشو می خوام

davy jones
26-04-2012, 13:24
سلام.میشه یه نفر درون چرخزاد رو کامل برام حل کنه؟؟؟معادلات پارامتری اخرشو نی خوام.حل کاملشو می خوام

سلام.
توضحاتی که میخوام بدم رو در هیچ سایت و فرومی پیدا نمیکنین. منظورم اینه که حتی تو معتبر ترین سایتهای خارجی مث ویکی پدیا هم روش اثبات فرمولهای پارامتری درون چرخزاد ها رو ننوشتن.

اول یه توضیحی برای سایر کاربران بدم که درون چرخزاد چیه؟

درون چرخزاد یا hypotrochoid (یا hypocycloid) عبارت است از اشکالی که از دوران یک دایره درون یک دایره ی دیگه به دست میاد. مانند اینها:


[ برای مشاهده لینک ، لطفا با نام کاربری خود وارد شوید یا ثبت نام کنید ]

[ برای مشاهده لینک ، لطفا با نام کاربری خود وارد شوید یا ثبت نام کنید ]






البته در مقابل مفهوم درون چرخزاد، مفهوم دیگری به نام برون چرخزاد هم وجود داره که تفاوتش با حالت قبل در اینه که دایره و نقطه ی متحرک همراه آن، در بیرون از دایره ی ثابت قرار دارند.

برای دیدن اشکال و توضیحات نسبی در باره ی برون چرخزادها به لینکهای زیر مراجعه کنین:

[ برای مشاهده لینک ، لطفا با نام کاربری خود وارد شوید یا ثبت نام کنید ] ([ برای مشاهده لینک ، لطفا با نام کاربری خود وارد شوید یا ثبت نام کنید ] Fwiki%2FEpitrochoid)
[ برای مشاهده لینک ، لطفا با نام کاربری خود وارد شوید یا ثبت نام کنید ] ([ برای مشاهده لینک ، لطفا با نام کاربری خود وارد شوید یا ثبت نام کنید ] Fwiki%2FEpicycloid)

---------------------

در اینگونه اشکال، 3 پارامتر مهم و تعیین کننده هستند:

1- شعاع دایره ی بزرگتر = R
2- شعاع دایره ی کوچکتر که درون دایره اول قرار گرفته است = r
3- فاصله ی نقطه ی مرجع ما که شکل را میکشد، از مرکز دایره ی کوچکتر = d

اگر در حالتی خاص d=r در نظر گرفته شود، اشکالی مانند شکل دوم بالا تشکیل میشود. در این حالت نسبت شعاع دو دایره را k نامیده و اشکال مختلف بر حسب k های متفاوت، گوناگون خواهند بود. مثالهایی از این حالت خاص را برای k های مختلف در زیر میتوانید ببینید:



[ برای مشاهده لینک ، لطفا با نام کاربری خود وارد شوید یا ثبت نام کنید ]

-----

[ برای مشاهده لینک ، لطفا با نام کاربری خود وارد شوید یا ثبت نام کنید ]

----

[ برای مشاهده لینک ، لطفا با نام کاربری خود وارد شوید یا ثبت نام کنید ]

-----

[ برای مشاهده لینک ، لطفا با نام کاربری خود وارد شوید یا ثبت نام کنید ]

-----

[ برای مشاهده لینک ، لطفا با نام کاربری خود وارد شوید یا ثبت نام کنید ]

-----

[ برای مشاهده لینک ، لطفا با نام کاربری خود وارد شوید یا ثبت نام کنید ]

-----

[ برای مشاهده لینک ، لطفا با نام کاربری خود وارد شوید یا ثبت نام کنید ]

-----

[ برای مشاهده لینک ، لطفا با نام کاربری خود وارد شوید یا ثبت نام کنید ]




اما در اینجا سعی میکنم که برای حالت کلی معادلات پارامتری رو اثبات و حل کنم.
برای راحتی مرکز دایره ی بزرگتر رو در مبدا مختصات قرار میدیم. همچنین باز برای راحتی از مختصات قطبی استفاده میکنیم. و نیز باز هم برای راحتی بیشتر در شروع حرکت، نقطه ی مرجع متحرگ رو روی محور x ها در نظر میگیریم. یعنی در لحظه ی صفر داریم: [ برای مشاهده لینک ، لطفا با نام کاربری خود وارد شوید یا ثبت نام کنید ]

میخواهیم ببینیم در هر لحظه بر حسب [ برای مشاهده لینک ، لطفا با نام کاربری خود وارد شوید یا ثبت نام کنید ] ، فاصله ی متحرک مرجع ما از مبدا مختصات چقدر خواهد بود؟
ذکر این نکته هم ضروریه که [ برای مشاهده لینک ، لطفا با نام کاربری خود وارد شوید یا ثبت نام کنید ] در حقیقت زاویه ی بین خط المرکزین دو دایره با محور x ها در هر لحظه است.


[ برای مشاهده لینک ، لطفا با نام کاربری خود وارد شوید یا ثبت نام کنید ]

[ برای مشاهده لینک ، لطفا با نام کاربری خود وارد شوید یا ثبت نام کنید ] D%27%7D&plus;%5Cvec%7B%7BO%7D%27M%7D


در نتیجه طول نقطه ی M (یعنی تصویر نقطه ی M روی محور x ها) برابر میشه با مجموع طول تصویرهای دو بردار 'OO و O'M.

طول تصویر بردار 'OO که به راحتی محاسبه میشود چرا که زاویه ی بین این بردار با محور x ها را میدانیم که برابر با [ برای مشاهده لینک ، لطفا با نام کاربری خود وارد شوید یا ثبت نام کنید ] است. پس طول تصویر بردار 'OO روی محور x ها برابر میشود با:


[ برای مشاهده لینک ، لطفا با نام کاربری خود وارد شوید یا ثبت نام کنید ] %7D&space;%5Cright&space;%7C%5Ccos&space;%28%5Ctheta&space;%29=%28R-r%29%5Ccos&space;%28%5Ctheta&space;%29


اما برای محاسبه ی طول تصویر بردار O'M لازم است که زاویه ی این بردار را با افق بدانیم. برای اینکار باید محاسبه کرد که دایره ی کوچکتر به ازای هر یک دور که روی محیط دایره ی بزرگ میزند، چند دور روی محیط خود چرخیده است؟ محیط دایره ی بزرگ برابر است با [ برای مشاهده لینک ، لطفا با نام کاربری خود وارد شوید یا ثبت نام کنید ] محیط دایره ی کوچک هم برابر است با [ برای مشاهده لینک ، لطفا با نام کاربری خود وارد شوید یا ثبت نام کنید ] پس وقتی که دایره ی کوچک روی محیط دایره ی بزرگ، یک دور کامل را طی میکند، در حقیقت به اندازه ی:


[ برای مشاهده لینک ، لطفا با نام کاربری خود وارد شوید یا ثبت نام کنید ] r%7D=%5Cfrac%7BR%7D%7Br%7D


روی محیط خودش دور زده است. پس زاویه ی بردار O'M را که آن را [ برای مشاهده لینک ، لطفا با نام کاربری خود وارد شوید یا ثبت نام کنید ] مینامیم این چنین بدست می آید:


[ برای مشاهده لینک ، لطفا با نام کاربری خود وارد شوید یا ثبت نام کنید ]


پس حالا میتونیم مانند بردار 'OO، طول تصویر بردار O'M را بر روی محور x ها را هم حساب کنیم. این طول برابر میشود با:


[ برای مشاهده لینک ، لطفا با نام کاربری خود وارد شوید یا ثبت نام کنید ] &space;%7C%5Ccos&space;%28%5Calpha&space;%29=d.%5Ccos&space;%28%281-%5Cfrac%7BR%7D%7Br%7D%29%5Ctheta&space;%29=d.%5Ccos&space;%28% 5Cfrac%7Br-R%7D%7Br%7D%5Ctheta&space;%29



بنابراین تصویر نقطه ی M روی محور x ها برابر میشه با:


[ برای مشاهده لینک ، لطفا با نام کاربری خود وارد شوید یا ثبت نام کنید ] 8R-r%29%5Ccos&space;%28%5Ctheta&space;%29&plus;d%5Ccos&space;%28%5Cfrac%7BR-r%7D%7Br%7D%5Ctheta&space;%29%7D


با استدلالی مشابه، تصویر نقطه ی M روی محور y ها هم برابر میشه با:


[ برای مشاهده لینک ، لطفا با نام کاربری خود وارد شوید یا ثبت نام کنید ] 8R-r%29%5Csin&space;%28%5Ctheta&space;%29-d%5Csin&space;%28%5Cfrac%7BR-r%7D%7Br%7D%5Ctheta&space;%29%7D



اما این جوابها به خودی خود ارزش ندارند. چرا که باید در هر لحظه محاسبه کنیم که [ برای مشاهده لینک ، لطفا با نام کاربری خود وارد شوید یا ثبت نام کنید ] برابر با چند است و متغیر [ برای مشاهده لینک ، لطفا با نام کاربری خود وارد شوید یا ثبت نام کنید ] نیز متغیری وابسته به زمان است. اگرحرکت دایره ی کوچک را در درون دایره ی بزرگ، حرکتی یکنواخت و بدون شتاب زاویه ای فرض کنیم، فرکانس حرکت دایره ی کوچک را در درون دایره ی بزرگ مقدار ثابتی مانند f خواهد بود و [ برای مشاهده لینک ، لطفا با نام کاربری خود وارد شوید یا ثبت نام کنید ] در هر لحظه برابر میشود با:


[ برای مشاهده لینک ، لطفا با نام کاربری خود وارد شوید یا ثبت نام کنید ] theta&space;_%7B0%7D


اما خودمان برای راحتی، قرارداد کردیم که : [ برای مشاهده لینک ، لطفا با نام کاربری خود وارد شوید یا ثبت نام کنید ] پس داریم:


[ برای مشاهده لینک ، لطفا با نام کاربری خود وارد شوید یا ثبت نام کنید ]


با این حساب و در نهایت مختصات نقطه ی مرجع متحرک در هر لحظه ی دلخواه t برابر است با:


[ برای مشاهده لینک ، لطفا با نام کاربری خود وارد شوید یا ثبت نام کنید ] %5C%7B%5Cbegin%7Bmatrix%7D&space;x%28t%29=%28R-r%29%5Ccos&space;%282%5Cpi&space;ft%29&plus;d%5Ccos&space;%28%5Cfrac%7B2% 28R-r%29%5Cpi&space;f%7D%7Br%7D%5C:&space;t%29%5C%5C&space;%5C%5C&space;y%28t% 29=%28R-r%29%5Csin&space;%282%5Cpi&space;ft%29-d%5Csin&space;%28%5Cfrac%7B2%28R-r%29%5Cpi&space;f%7D%7Br%7D%5C:&space;t%29&space;%5Cend%7Bmatrix%7D% 5Cright.%7D


موفق باشین.
91/2/7

skyzare
26-04-2012, 15:45
با سلام .

با تشکر از پاسخ شما .

این قسمت :


ما برای محاسبه ی طول تصویر بردار O'M لازم است که زاویه ی این بردار را با افق بدانیم. برای اینکار باید محاسبه کرد که دایره ی کوچکتر به ازای هر یک دور که روی محیط دایره ی بزرگ میزند، چند دور روی محیط خود چرخیده است؟ محیط دایره ی بزرگ برابر است با [ برای مشاهده لینک ، لطفا با نام کاربری خود وارد شوید یا ثبت نام کنید ] محیط دایره ی کوچک هم برابر است با [ برای مشاهده لینک ، لطفا با نام کاربری خود وارد شوید یا ثبت نام کنید ] پس وقتی که دایره ی کوچک روی محیط دایره ی بزرگ، یک دور کامل را طی میکند، در حقیقت به اندازه ی:

[ برای مشاهده لینک ، لطفا با نام کاربری خود وارد شوید یا ثبت نام کنید ] r%7D=%5Cfrac%7BR%7D%7Br%7D

روی محیط خودش دور زده است. پس زاویه ی بردار O'M را که آن را [ برای مشاهده لینک ، لطفا با نام کاربری خود وارد شوید یا ثبت نام کنید ] مینامیم این چنین بدست می آید:

[ برای مشاهده لینک ، لطفا با نام کاربری خود وارد شوید یا ثبت نام کنید ]
می فهمم چی کار کردید ولی خوب اون زاویه الفا که در واقع زاویه برادر O'M با افق هست چه ارتباطی با محیط دایره بزرگ و تعداد دفعات چرخش دایره کوچیک روی محیط خودش داره ؟

قاهر - Gahir
26-04-2012, 22:13
[ برای مشاهده لینک ، لطفا با نام کاربری خود وارد شوید یا ثبت نام کنید ] %7Bk%7D%5E%7Bp_%7B1%7D%7D%29%5E%7B%5Cfrac%7B1%7D%7 Bp_%7B1%7D%7D%7D%5Cgeq&space;%28%5Csum_%7Bk=1%7D%5E%7Bn% 7Dx_%7Bk%7D%5E%7Bp_%7B2%7D%7D%29%5E%7B%5Cfrac%7B1% 7D%7Bp_%7B2%7D%7D%7D%5CRightarrow&space;[%28%5Csum_%7Bk=1%7D%5E%7Bn%7Dx_%7Bk%7D%5E%7Bp_%7B1 %7D%7D%29%5E%7B%5Cfrac%7B1%7D%7Bp_%7B1%7D%7D%7D]%5E%7Bp_%7B1%7Dp_%7B2%7D%7D%5Cgeq&space;[%28%5Csum_%7Bk=1%7D%5E%7Bn%7Dx_%7Bk%7D%5E%7Bp_%7B2 %7D%7D%29%5E%7B%5Cfrac%7B1%7D%7Bp_%7B2%7D%7D%7D]%5E%7Bp_%7B1%7Dp_%7B2%7D%7D%5CRightarrow&space;%28%5Csum _%7Bk=1%7D%5E%7Bn%7Dx_%7Bk%7D%5E%7Bp_%7B1%7D%7D%29 %5E%7Bp_%7B2%7D%7D%5Cgeq&space;%28%5Csum_%7Bk=1%7D%5E%7B n%7Dx_%7Bk%7D%5E%7Bp_%7B2%7D%7D%29%5E%7Bp_%7B1%7D% 7D




[ برای مشاهده لینک ، لطفا با نام کاربری خود وارد شوید یا ثبت نام کنید ] arrow&space;%28%5Csum_%7Bk=1%7D%5E%7Bn&plus;1%7Dx_%7Bk%7D%5E% 7Bp_%7B1%7D%7D%29%5E%7Bp_%7B2%7D%7D%5Cgeq&space;%28%5Csu m_%7Bk=1%7D%5E%7Bn&plus;1%7Dx_%7Bk%7D%5E%7Bp_%7B2%7D%7D %29%5E%7Bp_%7B1%7D%7D%5CRightarrow&space;%28%5Csum_%7Bk= 1%7D%5E%7Bn%7Dx_%7Bk%7D%5E%7Bp_%7B1%7D%7D&plus;x_%7Bn&plus;1 %7D%5E%7Bp_%7B1%7D%7D%29%5E%7Bp_%7B2%7D%7D%5Cgeq&space;% 28%5Csum_%7Bk=1%7D%5E%7Bn%7Dx_%7Bk%7D%5E%7Bp_%7B2% 7D%7D&plus;x_%7Bn&plus;1%7D%5E%7Bp_%7B2%7D%7D%29%5E%7Bp_%7B1 %7D%7D%5CRightarrow&space;%5Csum_%7Bj=0%7D%5E%7Bp_%7B2%7 D%7D%5Cbinom%7Bp_%7B2%7D%7D%7Bj%7D%28%5Csum_%7Bk=1 %7D%5E%7Bn%7Dx_%7Bk%7D%5E%7Bp_%7B1%7D%7D%29%5E%7Bp _%7B2%7D-j%7D%28x_%7Bn&plus;1%7D%5E%7Bp_%7B1%7D%7D%29%5E%7Bj%7D% 5Cgeq&space;%5Csum_%7Bj=0%7D%5E%7Bp_%7B1%7D%7D%5Cbinom%7 Bp_%7B1%7D%7D%7Bj%7D%28%5Csum_%7Bk=1%7D%5E%7Bn%7Dx _%7Bk%7D%5E%7Bp_%7B2%7D%7D%29%5E%7Bp_%7B1%7D-j%7D%28x_%7Bn&plus;1%7D%5E%7Bp_%7B2%7D%7D%29%5E%7Bj%7D




به ازای j=0 به حالت فرض استقرامون یعنی حالتی که n رو به n+1 ارتقا نداده بودیم میرسیم که فرض کردیم همواره برقراره. پس میتونیم سری رو از جمله ی بعدیش شروع کنیم:





[ برای مشاهده لینک ، لطفا با نام کاربری خود وارد شوید یا ثبت نام کنید ] 5E%7Bp_%7B2%7D%7D%5Cbinom%7Bp_%7B2%7D%7D%7Bj%7D%28 %5Csum_%7Bk=1%7D%5E%7Bn%7Dx_%7Bk%7D%5E%7Bp_%7B1%7D %7D%29%5E%7Bp_%7B2%7D-j%7D%28x_%7Bn&plus;1%7D%5E%7Bp_%7B1%7D%7D%29%5E%7Bj%7D% 5Cgeq&space;%5Csum_%7Bj=1%7D%5E%7Bp_%7B1%7D%7D%5Cbinom%7 Bp_%7B1%7D%7D%7Bj%7D%28%5Csum_%7Bk=1%7D%5E%7Bn%7Dx _%7Bk%7D%5E%7Bp_%7B2%7D%7D%29%5E%7Bp_%7B1%7D-j%7D%28x_%7Bn&plus;1%7D%5E%7Bp_%7B2%7D%7D%29%5E%7Bj%7D








که ظاهرا اثبات آخری آسون به نظر نمیاد :31: مغزم دیگه هنگ کرد راستیاتش. :31:






یعنی واقعا اینقدر سخته از راه استقرا بخوایم بریم؟ :41:







موفق باشین.



91/2/3













سلام دوست گرامی ... ممنون بابت جوابتون ولی یه مشکلی هست که عجیب میدونم واسه چی به اون دقت نکردید.؟!

اوندش در مرحله‌ی اول واسه سادگی کار، توان [ برای مشاهده لینک ، لطفا با نام کاربری خود وارد شوید یا ثبت نام کنید ] \frac{1}{P_{1}} و [ برای مشاهده لینک ، لطفا با نام کاربری خود وارد شوید یا ثبت نام کنید ] \frac{1}{P_{2}}رو در دو طرف نامساوی با ضرب [ برای مشاهده لینک ، لطفا با نام کاربری خود وارد شوید یا ثبت نام کنید ] P_{1}P_{2}(مثبت) به صورت ساده‌تر نوشتید ... در واقع تا اینجا، کاری انجام نداده‌اید.
اگه فرض کنیم برای n=k تا جمله، این عبارت نامساوی برقرار باشه، باید بتونیم تا n=k+1 امین جمله هم برقرار کنیم تا حکم اصل استقرای ریاضی برای هر n ای صادق بشه.(اینا مراحل حل اثباتی به روش اصل استقرای ریاضی است)

ولی شما با چه مجوزی همون اول کاری در اثبات، n رو به قول خودتون به n+1 ارتقا دادید و بعد با دونستن برقراری این نامساوی مابقی کارا رو انجام داده‌اید ؟؟؟ ما که تا اینجای کار نمیدونیم نامساوی :

[ برای مشاهده لینک ، لطفا با نام کاربری خود وارد شوید یا ثبت نام کنید ] %28%5Csum_%7Bk=1%7D%5E%7Bn&plus;1%7Dx_%7Bk%7D%5E%7Bp_%7 B1%7D%7D%29%5E%7Bp_%7B2%7D%7D%5Cgeq&space;%28%5Csum_%7Bk =1%7D%5E%7Bn&plus;1%7Dx_%7Bk%7D%5E%7Bp_%7B2%7D%7D%29%5E %7Bp_%7B1%7D%7D
برقرار هستش!:18: بعد بیایم ازش دیگر انتسابات یک طرفه رو استنتاج کنیم !:13: این اصلا در فرآیند اثبات استقرایی نیست!


مثلا با توجه به فرض روی سؤال که [ برای مشاهده لینک ، لطفا با نام کاربری خود وارد شوید یا ثبت نام کنید ] \inline 0< P_{1} \leqslant P_{2} فقط کافیه [ برای مشاهده لینک ، لطفا با نام کاربری خود وارد شوید یا ثبت نام کنید ] \inline x_{n+1}> 1 باشه تا نتونیم با اطمینان بگیم که طرفین نامساوی بالا درست هستش!


حالا فرض کنید که این [ برای مشاهده لینک ، لطفا با نام کاربری خود وارد شوید یا ثبت نام کنید ] \inline _{(i=1,...,n+1)}x_{i}های شما که در دو طرف نامساوی گنجونده میشن، حتی پریشی از این آرایش n+1تایی از این اعداد حقیقی مثبت نباشند.



قسمت آخری که فرمودید خیلی ساده به نظر نمیاد ، ظاهراً بسط نیتونی اون عبارت رو نوشته‌اید.


من که قانع نشدم ... اگه اینجوری مدنظرتون بوده باشه یه اشتباه فاحش رو مرتکب شدید ...
از خیر اثبات استقرائیش گذشتم ... البته این مسئله حل شدست و کاملا برقراره ...
ایشالا اگه تونستم اثبات به روش غیر استقرایی رو همینجا قرار میدم.
بازم بابت توجهتون به سوالم خیلی ممنونم.


با تشکر : قاهر.

elahehamini
27-04-2012, 08:53
سلام میشه این مسئله از مبحث چرخزاد رو برام حل کنید لطفا؟
خط L به معادله ی L:x=2r و دایره ی c به مرکز (c(r,0 و شعاع r داده شده اند.هر خط m که از o بگذرد دایره را در نقطه ای مانند A و خط L را در نقطه ای چون B قطع می کند.نقطه ی M بر خط m را چنان انتخاب کنید که M=o یا M بین A و o باشد.همچنین AB=oM .وقتی خط m تغییر کند نقطه ی M یک خم به نام پیچکوار یا سیسوئید پدید می اورد.با توجه به شکل معادلات پارامتری پیچکوار را بر حسب تتا بنویسید.


---------- Post added at 09:53 AM ---------- Previous post was at 09:52 AM ----------

ببخشید شکلشو نمیدونم چطوری بکشم.اگه بدون شکل نمی تونید ,بگید یه کاری می کنم

elahehamini
27-04-2012, 11:18
سلام.
توضحاتی که میخوام بدم رو در هیچ سایت و فرومی پیدا نمیکنین. منظورم اینه که حتی تو معتبر ترین سایتهای خارجی مث ویکی پدیا هم روش اثبات فرمولهای پارامتری درون چرخزاد ها رو ننوشتن.

اول یه توضیحی برای سایر کاربران بدم که درون چرخزاد چیه؟

درون چرخزاد یا hypotrochoid (یا hypocycloid) عبارت است از اشکالی که از دوران یک دایره درون یک دایره ی دیگه به دست میاد. مانند اینها:


[ برای مشاهده لینک ، لطفا با نام کاربری خود وارد شوید یا ثبت نام کنید ]




[ برای مشاهده لینک ، لطفا با نام کاربری خود وارد شوید یا ثبت نام کنید ]






البته در مقابل مفهوم درون چرخزاد، مفهوم دیگری به نام برون چرخزاد هم وجود داره که تفاوتش با حالت قبل در اینه که دایره و نقطه ی متحرک همراه آن، در بیرون از دایره ی ثابت قرار دارند.






برای دیدن اشکال و توضیحات نسبی در باره ی برون چرخزادها به لینکهای زیر مراجعه کنین:




[ برای مشاهده لینک ، لطفا با نام کاربری خود وارد شوید یا ثبت نام کنید ] ([ برای مشاهده لینک ، لطفا با نام کاربری خود وارد شوید یا ثبت نام کنید ] Fwiki%2FEpitrochoid)


[ برای مشاهده لینک ، لطفا با نام کاربری خود وارد شوید یا ثبت نام کنید ] ([ برای مشاهده لینک ، لطفا با نام کاربری خود وارد شوید یا ثبت نام کنید ] Fwiki%2FEpicycloid)






---------------------





در اینگونه اشکال، 3 پارامتر مهم و تعیین کننده هستند:




1- شعاع دایره ی بزرگتر = R


2- شعاع دایره ی کوچکتر که درون دایره اول قرار گرفته است = r


3- فاصله ی نقطه ی مرجع ما که شکل را میکشد، از مرکز دایره ی کوچکتر = d




اگر در حالتی خاص d=r در نظر گرفته شود، اشکالی مانند شکل دوم بالا تشکیل میشود. در این حالت نسبت شعاع دو دایره را k نامیده و اشکال مختلف بر حسب k های متفاوت، گوناگون خواهند بود. مثالهایی از این حالت خاص را برای k های مختلف در زیر میتوانید ببینید:






[ برای مشاهده لینک ، لطفا با نام کاربری خود وارد شوید یا ثبت نام کنید ]





-----




[ برای مشاهده لینک ، لطفا با نام کاربری خود وارد شوید یا ثبت نام کنید ]




----




[ برای مشاهده لینک ، لطفا با نام کاربری خود وارد شوید یا ثبت نام کنید ]




-----




[ برای مشاهده لینک ، لطفا با نام کاربری خود وارد شوید یا ثبت نام کنید ]




-----




[ برای مشاهده لینک ، لطفا با نام کاربری خود وارد شوید یا ثبت نام کنید ]




-----




[ برای مشاهده لینک ، لطفا با نام کاربری خود وارد شوید یا ثبت نام کنید ]




-----




[ برای مشاهده لینک ، لطفا با نام کاربری خود وارد شوید یا ثبت نام کنید ]




-----




[ برای مشاهده لینک ، لطفا با نام کاربری خود وارد شوید یا ثبت نام کنید ]









اما در اینجا سعی میکنم که برای حالت کلی معادلات پارامتری رو اثبات و حل کنم.


برای راحتی مرکز دایره ی بزرگتر رو در مبدا مختصات قرار میدیم. همچنین باز برای راحتی از مختصات قطبی استفاده میکنیم. و نیز باز هم برای راحتی بیشتر در شروع حرکت، نقطه ی مرجع متحرگ رو روی محور x ها در نظر میگیریم. یعنی در لحظه ی صفر داریم: [ برای مشاهده لینک ، لطفا با نام کاربری خود وارد شوید یا ثبت نام کنید ]




میخواهیم ببینیم در هر لحظه بر حسب [ برای مشاهده لینک ، لطفا با نام کاربری خود وارد شوید یا ثبت نام کنید ] ، فاصله ی متحرک مرجع ما از مبدا مختصات چقدر خواهد بود؟


ذکر این نکته هم ضروریه که [ برای مشاهده لینک ، لطفا با نام کاربری خود وارد شوید یا ثبت نام کنید ] در حقیقت زاویه ی بین خط المرکزین دو دایره با محور x ها در هر لحظه است.





[ برای مشاهده لینک ، لطفا با نام کاربری خود وارد شوید یا ثبت نام کنید ]





[ برای مشاهده لینک ، لطفا با نام کاربری خود وارد شوید یا ثبت نام کنید ] D%27%7D&plus;%5Cvec%7B%7BO%7D%27M%7D





در نتیجه طول نقطه ی M (یعنی تصویر نقطه ی M روی محور x ها) برابر میشه با مجموع طول تصویرهای دو بردار 'OO و O'M.




طول تصویر بردار 'OO که به راحتی محاسبه میشود چرا که زاویه ی بین این بردار با محور x ها را میدانیم که برابر با [ برای مشاهده لینک ، لطفا با نام کاربری خود وارد شوید یا ثبت نام کنید ] است. پس طول تصویر بردار 'OO روی محور x ها برابر میشود با:





[ برای مشاهده لینک ، لطفا با نام کاربری خود وارد شوید یا ثبت نام کنید ] %7D&space;%5Cright&space;%7C%5Ccos&space;%28%5Ctheta&space;%29=%28R-r%29%5Ccos&space;%28%5Ctheta&space;%29






اما برای محاسبه ی طول تصویر بردار O'M لازم است که زاویه ی این بردار را با افق بدانیم. برای اینکار باید محاسبه کرد که دایره ی کوچکتر به ازای هر یک دور که روی محیط دایره ی بزرگ میزند، چند دور روی محیط خود چرخیده است؟ محیط دایره ی بزرگ برابر است با [ برای مشاهده لینک ، لطفا با نام کاربری خود وارد شوید یا ثبت نام کنید ] محیط دایره ی کوچک هم برابر است با [ برای مشاهده لینک ، لطفا با نام کاربری خود وارد شوید یا ثبت نام کنید ] پس وقتی که دایره ی کوچک روی محیط دایره ی بزرگ، یک دور کامل را طی میکند، در حقیقت به اندازه ی:





[ برای مشاهده لینک ، لطفا با نام کاربری خود وارد شوید یا ثبت نام کنید ] r%7D=%5Cfrac%7BR%7D%7Br%7D






روی محیط خودش دور زده است. پس زاویه ی بردار O'M را که آن را [ برای مشاهده لینک ، لطفا با نام کاربری خود وارد شوید یا ثبت نام کنید ] مینامیم این چنین بدست می آید:





[ برای مشاهده لینک ، لطفا با نام کاربری خود وارد شوید یا ثبت نام کنید ]






پس حالا میتونیم مانند بردار 'OO، طول تصویر بردار O'M را بر روی محور x ها را هم حساب کنیم. این طول برابر میشود با:





[ برای مشاهده لینک ، لطفا با نام کاربری خود وارد شوید یا ثبت نام کنید ] &space;%7C%5Ccos&space;%28%5Calpha&space;%29=d.%5Ccos&space;%28%281-%5Cfrac%7BR%7D%7Br%7D%29%5Ctheta&space;%29=d.%5Ccos&space;%28% 5Cfrac%7Br-R%7D%7Br%7D%5Ctheta&space;%29







بنابراین تصویر نقطه ی M روی محور x ها برابر میشه با:





[ برای مشاهده لینک ، لطفا با نام کاربری خود وارد شوید یا ثبت نام کنید ] 8R-r%29%5Ccos&space;%28%5Ctheta&space;%29&plus;d%5Ccos&space;%28%5Cfrac%7BR-r%7D%7Br%7D%5Ctheta&space;%29%7D






با استدلالی مشابه، تصویر نقطه ی M روی محور y ها هم برابر میشه با:





[ برای مشاهده لینک ، لطفا با نام کاربری خود وارد شوید یا ثبت نام کنید ] 8R-r%29%5Csin&space;%28%5Ctheta&space;%29-d%5Csin&space;%28%5Cfrac%7BR-r%7D%7Br%7D%5Ctheta&space;%29%7D







اما این جوابها به خودی خود ارزش ندارند. چرا که باید در هر لحظه محاسبه کنیم که [ برای مشاهده لینک ، لطفا با نام کاربری خود وارد شوید یا ثبت نام کنید ] برابر با چند است و متغیر [ برای مشاهده لینک ، لطفا با نام کاربری خود وارد شوید یا ثبت نام کنید ] نیز متغیری وابسته به زمان است. اگرحرکت دایره ی کوچک را در درون دایره ی بزرگ، حرکتی یکنواخت و بدون شتاب زاویه ای فرض کنیم، فرکانس حرکت دایره ی کوچک را در درون دایره ی بزرگ مقدار ثابتی مانند f خواهد بود و [ برای مشاهده لینک ، لطفا با نام کاربری خود وارد شوید یا ثبت نام کنید ] در هر لحظه برابر میشود با:





[ برای مشاهده لینک ، لطفا با نام کاربری خود وارد شوید یا ثبت نام کنید ] theta&space;_%7B0%7D






اما خودمان برای راحتی، قرارداد کردیم که : [ برای مشاهده لینک ، لطفا با نام کاربری خود وارد شوید یا ثبت نام کنید ] پس داریم:





[ برای مشاهده لینک ، لطفا با نام کاربری خود وارد شوید یا ثبت نام کنید ]






با این حساب و در نهایت مختصات نقطه ی مرجع متحرک در هر لحظه ی دلخواه t برابر است با:





[ برای مشاهده لینک ، لطفا با نام کاربری خود وارد شوید یا ثبت نام کنید ] %5C%7B%5Cbegin%7Bmatrix%7D&space;x%28t%29=%28R-r%29%5Ccos&space;%282%5Cpi&space;ft%29&plus;d%5Ccos&space;%28%5Cfrac%7B2% 28R-r%29%5Cpi&space;f%7D%7Br%7D%5C:&space;t%29%5C%5C&space;%5C%5C&space;y%28t% 29=%28R-r%29%5Csin&space;%282%5Cpi&space;ft%29-d%5Csin&space;%28%5Cfrac%7B2%28R-r%29%5Cpi&space;f%7D%7Br%7D%5C:&space;t%29&space;%5Cend%7Bmatrix%7D% 5Cright.%7D






موفق باشین.



91/2/7































































ممنونممممممم

davy jones
28-04-2012, 18:36
با سلام .

با تشکر از پاسخ شما .

این قسمت :

می فهمم چی کار کردید ولی خوب اون زاویه الفا که در واقع زاویه برادر O'M با افق هست چه ارتباطی با محیط دایره بزرگ و تعداد دفعات چرخش دایره کوچیک روی محیط خودش داره ؟



سلام

من یه نکته ای رو همون اول یادم رفت بگم و اونم اینکه وقتی دایره ی متحرک در درون دایره ی ثابت حرکت میکنه، جهت چرخش نقطه ی مرجع (نقطه ی M)، در خلاف جهت چرخش مرکز دایره ی متحرک خواهد بود. یعنی اگه پاره خط 'OO حول نقطه ی O داره ساعتگرد میچرخه، مطمئنا پاره خط O'M حول نقطه ی 'O پاد ساعتگرد خواهد چرخید (و بالعکس) بنابراین وقتی که میخوایم زاویه ی آلفا رو حساب کنیم باید مقدار زاویه ای که پاره خط O'M حول نقطه ی 'O چرخیده رو از مقدار زاویه ی تتا کم کنیم.(که در حقیقت تتا برابر با زاویه ی دوران پاره خط 'OO حول نقطه ی O هستش) حالا مقدار دوران پاره خط O'M حول نقطه ی 'O با تتا متناسب است اما با یک ضریب. که آن ضریب نسبت محیط دو دایره است. چرا که قبلا هم گفتیم که هنگامی که دایره ی کوچکتر یک دور روی محیط دایره ی بزرگ (دایره ی ثابت) دور میزند، به اندازه ی نسبت محیط دایره ی بزرگ به محیط خودش، دور خودش چرخیده است و چون زاویه ی دوران هر دایره با کمانی از محیط خودش که معادل آن زاویه است، مستقیما متناسب است، پس نسبت زاویه ی دوران 2 دایره هم به همان نسبت دو محیط است.
(میدونم که تصورش یکمی سخته ولی اگه حدود 10 دقیقه به اشکالی که گذاشتم دقت کنین، کم کم این چیزی که گفتم میاد دستتون:31:)


سلام میشه این مسئله از مبحث چرخزاد رو برام حل کنید لطفا؟
خط L به معادله ی L:x=2r و دایره ی c به مرکز (c(r,0 و شعاع r داده شده اند.هر خط m که از o بگذرد دایره را در نقطه ای مانند A و خط L را در نقطه ای چون B قطع می کند.نقطه ی M بر خط m را چنان انتخاب کنید که M=o یا M بین A و o باشد.همچنین AB=oM .وقتی خط m تغییر کند نقطه ی M یک خم به نام پیچکوار یا سیسوئید پدید می اورد.با توجه به شکل معادلات پارامتری پیچکوار را بر حسب تتا بنویسید.


---------- Post added at 09:53 AM ---------- Previous post was at 09:52 AM ----------

ببخشید شکلشو نمیدونم چطوری بکشم.اگه بدون شکل نمی تونید ,بگید یه کاری می کنم

سلام.

معادله ی هر خط دلخواه m که از مبدا میگذره برابر است با:


[ برای مشاهده لینک ، لطفا با نام کاربری خود وارد شوید یا ثبت نام کنید ]


همچنین معادله ی دایره ی c هم برابره با:


[ برای مشاهده لینک ، لطفا با نام کاربری خود وارد شوید یا ثبت نام کنید ]


معادله ی خط L هم که همونطور که در صورت سوال گفته برابره با : x=2r

خط m ، خط L را در نقطه ی [ برای مشاهده لینک ، لطفا با نام کاربری خود وارد شوید یا ثبت نام کنید ] قطع میکند که در حقیقت مختصات پارامتری نقطه ی B است.
برای پیدا کردن مختصات پارامتری نقطه ی A هم کافی است که خط m را با دایره ی c برخورد دهیم:


[ برای مشاهده لینک ، لطفا با نام کاربری خود وارد شوید یا ثبت نام کنید ] Coverset%7By=mx&space;%7D%7B%5CRightarrow&space;%7D%5C;&space;%5C;&space;% 28x-r%29%5E%7B2%7D&plus;%28mx%29%5E%7B2%7D=r%5E%7B2%7D



[ برای مشاهده لینک ، لطفا با نام کاربری خود وارد شوید یا ثبت نام کنید ] 29x%5E%7B2%7D-2rx=0%5CRightarrow&space;%5Cleft%5C%7B%5Cbegin%7Bmatrix% 7D&space;x_%7B1%7D=0%5CRightarrow&space;y_%7B1%7D=0%5Crightarr ow&space;A=o%5C%5C&space;%5C%5C&space;x_%7B2%7D=%5Cfrac%7B2r%7D%7B%2 8m%5E%7B2%7D&plus;1%29%7D%5CRightarrow&space;y_%7B2%7D=%5Cfra c%7B2mr%7D%7B%28m%5E%7B2%7D&plus;1%29%7D%5Crightarrow&space;% 7B%5Ccolor%7BRed%7D&space;A=%28%5Cfrac%7B2r%7D%7B%28m%5E %7B2%7D&plus;1%29%7D,%5Cfrac%7B2mr%7D%7B%28m%5E%7B2%7D&plus; 1%29%7D%29%7D&space;%5Cend%7Bmatrix%7D%5Cright.



حالا با این اوصاف باید فاصله ی بین دو نقطه ی A و B رو محاسبه کنیم:


[ برای مشاهده لینک ، لطفا با نام کاربری خود وارد شوید یا ثبت نام کنید ] C&space;AB&space;%5Cright&space;%7C%7D=%5Csqrt%7B%28%5CDelta&space;x%29%5E %7B2%7D&plus;%28%5CDelta&space;y%29%5E%7B2%7D%7D=%5Csqrt%7B%7 Cx_%7BA%7D-x_%7BB%7D%7C%5E%7B2%7D&plus;%7Cy_%7BA%7D-y_%7BB%7D%7C%5E%7B2%7D%7D=%5Csqrt%7B%28%5Cfrac%7B2 r%7D%7B%28m%5E%7B2%7D&plus;1%29%7D-2r%29%5E%7B2%7D&plus;%28%5Cfrac%7B2mr%7D%7B%28m%5E%7B2% 7D&plus;1%29%7D-2mr%29%5E%7B2%7D%7D=2r%5Csqrt%7B%281&plus;m%5E%7B2%7D%2 9%28%5Cfrac%7B1%7D%7B1&plus;m%5E%7B2%7D%7D-1%29%5E%7B2%7D%7D=2r%5Csqrt%7B-m%5E%7B2%7D%28%5Cfrac%7B1%7D%7B1&plus;m%5E%7B2%7D%7D-1%29%7D=%7B%5Ccolor%7BRed%7D&space;2rm%5E%7B2%7D%5Csqrt% 7B%5Cfrac%7B1%7D%7B1&plus;m%5E%7B2%7D%7D%7D%7D


که این مقدار طبق صورت سوال باید برابر با فاصله ی مبدا تا نقطه ی M روی خط m که برابر است با [ برای مشاهده لینک ، لطفا با نام کاربری خود وارد شوید یا ثبت نام کنید ] 5E%7B2%7D%7D=x%5Csqrt%7B1&plus;m%5E%7B2%7D%7D باشد. با مساوی قرار دادن این دو مقدار، رابطه ی x و y بر حسب m به دست میآید که در حقیقت همان معادله ی پارامتری پیچکوار است که به عنوان تمرین، این قسمت آخرش رو به عهده ی خودتون میذارم. :31:

===============


ممنونممممممم




خواهش میکنم. برای تشکر کردن صرفا از دکمه ی [ برای مشاهده لینک ، لطفا با نام کاربری خود وارد شوید یا ثبت نام کنید ] استفاده کنین.




موفق باشین.
91/2/9

skyzare
28-04-2012, 23:49
با سلام .

ببخشید اساتید می خواستم بدون این تصاویر چه ربطی به گرادیان داره ؟ گرادیان حالا همون تعریف ریاضی اش رو فقط در حد همون فرمولش میدونم . ولی این تصاویر چه ربطی بهش داره ؟ اون فلش ها چی هست ؟

[ برای مشاهده لینک ، لطفا با نام کاربری خود وارد شوید یا ثبت نام کنید ]


[ برای مشاهده لینک ، لطفا با نام کاربری خود وارد شوید یا ثبت نام کنید ]


[ برای مشاهده لینک ، لطفا با نام کاربری خود وارد شوید یا ثبت نام کنید ]

Kesel
30-04-2012, 13:21
سلام
معادله ی زیر رو ملاحظه بفرمایید
[ برای مشاهده لینک ، لطفا با نام کاربری خود وارد شوید یا ثبت نام کنید ]
چون توانش زوج و عدد 4 هست می تونیم بگیم چهار تا ریشه ی مختلط داره که دو به دو مزدوجن.
اما چطوری می شه این ریشه ها رو پیدا کرد؟مگه می تونیم به اتحادی تبدیلش کنیم که اونور تساوی بشه صفر ؟ مگه برای معادلات درجه ی 4 دستور خاصی داریم؟مگه با عدد گذاری نتیجه می ده ؟

ممنونم می شم راهنماییم کنین !
مرسی

hts1369
30-04-2012, 17:24
سلام
معادله ی زیر رو ملاحظه بفرمایید
[ برای مشاهده لینک ، لطفا با نام کاربری خود وارد شوید یا ثبت نام کنید ]
چون توانش زوج و عدد 4 هست می تونیم بگیم چهار تا ریشه ی مختلط داره که دو به دو مزدوجن.
اما چطوری می شه این ریشه ها رو پیدا کرد؟مگه می تونیم به اتحادی تبدیلش کنیم که اونور تساوی بشه صفر ؟ مگه برای معادلات درجه ی 4 دستور خاصی داریم؟مگه با عدد گذاری نتیجه می ده ؟

ممنونم می شم راهنماییم کنین !
مرسی
برو اینجا ([ برای مشاهده لینک ، لطفا با نام کاربری خود وارد شوید یا ثبت نام کنید ]) قبلا در موردش بحث شده.

Kesel
30-04-2012, 18:52
برو اینجا ([ برای مشاهده لینک ، لطفا با نام کاربری خود وارد شوید یا ثبت نام کنید ]) قبلا در موردش بحث شده.

ممنون از جوابتون ولی این چیزی نیست که من می خوام
این سوال امتحان ریاضی 1 دانشکده ی مهندسی دانشگاه فردوسیه. مال علوم ریاضی که نیس که بخوایم به روشای عجیب غریبی شبیه کاردانو حلش کنیم.
من می خوام این مساله که در حد کتاب ریاضی 1 عمومی هست حل بشه . یعنی از همون روش cis و روش های معمول نه این قدر پیچیده.
مطمئن باشید این مساله خیلی راحت حل می شه چون می گم این سوال در حد ریاضی 1 هست
+
روش شما فقط برای اعداد حقیقی جواب می ده ! دوست من ! این معادله مختلطه ! دقت کنید . اگه نه با یه نگاه ساده می شه فهمید که این معادله ریشه ی حقیقی نداره.تمام چهار تا ریشش مختلطن.

اگر کسی روشی داره اگه لطف کنه بگه خوشحال می شم

باز هم تشکر

Ship Storm
30-04-2012, 19:59
سلام
یکسری نمونه سوال هست در رابطه با درس ریاضی مهندسی خواستم ببینم اینجا میشه مطرح کرد که بتونم جواب کاملش رو بگیرم
من خودم سر در نمیارم از این درس و میخوام روی این چندین سوال وقت بزارم و کار کنم اگر کسی از دوستان هست که میتونه در این زمینه کمک بده ممنون میشم
بازم ممنون

ali_hp
30-04-2012, 22:44
ممنون از جوابتون ولی این چیزی نیست که من می خوام
این سوال امتحان ریاضی 1 دانشکده ی مهندسی دانشگاه فردوسیه. مال علوم ریاضی که نیس که بخوایم به روشای عجیب غریبی شبیه کاردانو حلش کنیم.
من می خوام این مساله که در حد کتاب ریاضی 1 عمومی هست حل بشه . یعنی از همون روش cis و روش های معمول نه این قدر پیچیده.
مطمئن باشید این مساله خیلی راحت حل می شه چون می گم این سوال در حد ریاضی 1 هست
+
روش شما فقط برای اعداد حقیقی جواب می ده ! دوست من ! این معادله مختلطه ! دقت کنید . اگه نه با یه نگاه ساده می شه فهمید که این معادله ریشه ی حقیقی نداره.تمام چهار تا ریشش مختلطن.

اگر کسی روشی داره اگه لطف کنه بگه خوشحال می شم

باز هم تشکر

سلام
سه جور ميشه اين معادله رو حل كرد كه البته هر سه تاش يكيه!و البته هرسه تاش بر پايه نظم خاصيه كه اين عبارت داره ، توالي توانهاي z!
اول:معادله رو در z-1 ضرب كنيد! و بعد سادش كنيد ، به معادله z^5=1 ميرسيد كه احتمالا حلشو بلدين ، فقط بايد حواسمون باشه كه z=1 ريشه معادله اصلي نيست.
دوم:اين عبارت مجموع جملات يك تصاعد هندسي هست ، پس ما يك فرم ساده تر براش بلديم !
سوم: اين عبارت جمله چاق در تعميم اتحاد چاق و لاغر هست!

ali_hp
30-04-2012, 22:52
با سلام .

ببخشید اساتید می خواستم بدون این تصاویر چه ربطی به گرادیان داره ؟ گرادیان حالا همون تعریف ریاضی اش رو فقط در حد همون فرمولش میدونم . ولی این تصاویر چه ربطی بهش داره ؟ اون فلش ها چی هست ؟

[ برای مشاهده لینک ، لطفا با نام کاربری خود وارد شوید یا ثبت نام کنید ]


[ برای مشاهده لینک ، لطفا با نام کاربری خود وارد شوید یا ثبت نام کنید ]


[ برای مشاهده لینک ، لطفا با نام کاربری خود وارد شوید یا ثبت نام کنید ]





سلام
والا گراديان يك ميدان برداريه ، و ميدان بر داري هم شهودا يعني اينكه ما بر نقطه از فضا يك بردار نسبت بديم ، حالا تو اين تصوير ها هم تعدادي از اين بردارها براي اينكه ما تجسمي از ميدانمون در نقاط مختلف فضا داشته باشيم رسم شده .اين بردارها بردارهاي گراديان در نقاط مختلف هستن...

Kesel
30-04-2012, 23:28
سلام
سه جور ميشه اين معادله رو حل كرد كه البته هر سه تاش يكيه!و البته هرسه تاش بر پايه نظم خاصيه كه اين عبارت داره ، توالي توانهاي z!
اول:معادله رو در z-1 ضرب كنيد! و بعد سادش كنيد ، به معادله z^5=1 ميرسيد كه احتمالا حلشو بلدين ، فقط بايد حواسمون باشه كه z=1 ريشه معادله اصلي نيست.
دوم:اين عبارت مجموع جملات يك تصاعد هندسي هست ، پس ما يك فرم ساده تر براش بلديم !
سوم: اين عبارت جمله چاق در تعميم اتحاد چاق و لاغر هست!
جواب شما کاملا درسته . همونطور که گفتین راه اول و سوم یکی بود که هر دو تا قابل ارائه هستن و قبلا دیده بودم.چون توان چهار بود من خودم به ذهنم تعمیم چاق و لاغر نرسیده بود. بازم ممنون

MasterGeek
30-04-2012, 23:44
سلام
معادله ی زیر رو ملاحظه بفرمایید
[ برای مشاهده لینک ، لطفا با نام کاربری خود وارد شوید یا ثبت نام کنید ]
چون توانش زوج و عدد 4 هست می تونیم بگیم چهار تا ریشه ی مختلط داره که دو به دو مزدوجن.
اما چطوری می شه این ریشه ها رو پیدا کرد؟مگه می تونیم به اتحادی تبدیلش کنیم که اونور تساوی بشه صفر ؟ مگه برای معادلات درجه ی 4 دستور خاصی داریم؟مگه با عدد گذاری نتیجه می ده ؟

ممنونم می شم راهنماییم کنین !
مرسی


ممنون از جوابتون ولی این چیزی نیست که من می خوام
این سوال امتحان ریاضی 1 دانشکده ی مهندسی دانشگاه فردوسیه. مال علوم ریاضی که نیس که بخوایم به روشای عجیب غریبی شبیه کاردانو حلش کنیم.
من می خوام این مساله که در حد کتاب ریاضی 1 عمومی هست حل بشه . یعنی از همون روش cis و روش های معمول نه این قدر پیچیده.
مطمئن باشید این مساله خیلی راحت حل می شه چون می گم این سوال در حد ریاضی 1 هست
+
روش شما فقط برای اعداد حقیقی جواب می ده ! دوست من ! این معادله مختلطه ! دقت کنید . اگه نه با یه نگاه ساده می شه فهمید که این معادله ریشه ی حقیقی نداره.تمام چهار تا ریشش مختلطن.

اگر کسی روشی داره اگه لطف کنه بگه خوشحال می شم

باز هم تشکر

شاید با این بتونین حلش کنین:
* ضرب ریشه های معادله برابر 1 میشه
* جمع ریشه های معادله برابر -1 میشه
* هر چهار ریشه هم مختلط هستند
* چون ضرائب حقیقی اند، ریشه ها دوبدو با هم مزدوج اند
:46:

elahehamini
01-05-2012, 09:21
سلام خسته نباشید.مبحث توابع چند متغیره ریاضی عمومی 2 ،سوال داشتم.
در خصوص پیوستگی تابع زیر بحث کنید.

elahehamini
01-05-2012, 09:31
سلام خسته نباشید.مبحث توابع چند متغیره ریاضی عمومی 2 ،سوال داشتم.
سوال های 3 و 4 رو لطفا بهم جوابشو بگید.اگه می تونید ممنون ی شم همین امروز جواب بدید اخه فردا امتحان دارمممم:31:
[ برای مشاهده لینک ، لطفا با نام کاربری خود وارد شوید یا ثبت نام کنید ]

davy jones
01-05-2012, 13:08
سلام
یکسری نمونه سوال هست در رابطه با درس ریاضی مهندسی خواستم ببینم اینجا میشه مطرح کرد که بتونم جواب کاملش رو بگیرم
من خودم سر در نمیارم از این درس و میخوام روی این چندین سوال وقت بزارم و کار کنم اگر کسی از دوستان هست که میتونه در این زمینه کمک بده ممنون میشم
بازم ممنون

سلام حامد جان! از این ورا؟ :31:
طبق معمول همیشه شما سوالاتت رو بذار. منتها کیه که جواب بده؟ :31:
====================




با سلام .

ببخشید اساتید می خواستم بدون این تصاویر چه ربطی به گرادیان داره ؟ گرادیان حالا همون تعریف ریاضی اش رو فقط در حد همون فرمولش میدونم . ولی این تصاویر چه ربطی بهش داره ؟ اون فلش ها چی هست ؟

[ برای مشاهده لینک ، لطفا با نام کاربری خود وارد شوید یا ثبت نام کنید ] ([ برای مشاهده لینک ، لطفا با نام کاربری خود وارد شوید یا ثبت نام کنید ] Fwiki%2FGradient)


[ برای مشاهده لینک ، لطفا با نام کاربری خود وارد شوید یا ثبت نام کنید ]


[ برای مشاهده لینک ، لطفا با نام کاربری خود وارد شوید یا ثبت نام کنید ]







سلام
والا گراديان يك ميدان برداريه ، و ميدان بر داري هم شهودا يعني اينكه ما بر نقطه از فضا يك بردار نسبت بديم ، حالا تو اين تصوير ها هم تعدادي از اين بردارها براي اينكه ما تجسمي از ميدانمون در نقاط مختلف فضا داشته باشيم رسم شده .اين بردارها بردارهاي گراديان در نقاط مختلف هستن...

سلام.

گرادیان از لحاظ تجسمی، میزان فرو رفتگی و خاصیت سیاه چاله بودن یک نقطه از فضا رو نشون میدن.
در شکل اول سمت راست نشون داده میشه که هر برداری که از یک مقطع دلخواه به اون مقطع وارد شده، با همون مقدار و شدت، ازش خارج شده. پس گرادیان در چنین فضایی صفره.

اما در شکل سمت چپ، گرادیان در اون نقطه ی مرکزی مقدار بسیار بزرگی داره. چرا که داره بردارها رو در فضای اطرافش به سمت خودش میکشه.

در شکل دوم هم منظور از شکل اینه که مقدار گرادیان از لحاظ جبری به جهت بردارها در همسایگی یک فضای کوچک در نقطه ی مورد نظر ما بستگی داره. مثلا در قسمت آبی رنگ، چون جهت بردارها به سمت بیرون هستش، گرادیان منفی و در قسمت قرمز رنگ، مقدار گرادیان مثبته.

در مقابل مفهوم گرادیان، مفهوم کِرل وجود داره. کرل از لحاظ شهودی خاصیت گردابی و چرخش بردارها حول یک نقطه و یا یک همسایگی رو نشون میده. به طور مثال در تصویر اول سمت راست پستتون، مقدار کرل در همه ی نقاط و به ازای همه ی همسایگی ها صفره چون مقدار چرخش بردارها در جهت ساعتگرد در یک حلقه ی بسته ی فرضی (که لزوما میتونه دایره هم نباشه و فقط مهمه که حلقه ای بسته باشه)، برابر با مقدار چرخش در جهت پاد ساعتگرد در همان حلقه است.

=====================

سلام خسته نباشید.مبحث توابع چند متغیره ریاضی عمومی 2 ،سوال داشتم.
سوال های 3 و 4 رو لطفا بهم جوابشو بگید.اگه می تونید ممنون ی شم همین امروز جواب بدید اخه فردا امتحان دارمممم[ برای مشاهده لینک ، لطفا با نام کاربری خود وارد شوید یا ثبت نام کنید ]
[ برای مشاهده لینک ، لطفا با نام کاربری خود وارد شوید یا ثبت نام کنید ] ([ برای مشاهده لینک ، لطفا با نام کاربری خود وارد شوید یا ثبت نام کنید ] 2F%3Fep53ioh99aivobn)


سلام. نمیدونم چرا در سوال قبلیتون هم نمیتونستم فرمول مد نظرتون رو ببینم و از توضیحات سایرین فهمیدم که باید سوالتون چی باشه.
نمیدونم چرا ولی عکسها و لینکهایی که شما میذارین برای من کار نمیکنه. لینک پی دی اف ای هم که تو مدیافایر گذاشتین الان باز هم برای من کار نمیکنه.
دوره زمونه ای شده!




موفق باشین.
91/2/12

Kesel
01-05-2012, 13:13
شاید با این بتونین حلش کنین:
* ضرب ریشه های معادله برابر 1 میشه
* جمع ریشه های معادله برابر -1 میشه
* هر چهار ریشه هم مختلط هستند
* چون ضرائب حقیقی اند، ریشه ها دوبدو با هم مزدوج اند
:46:

ممنون
چه جوری این نتیجه ها رو گرفتین ؟
از کجا فهمیدین ضرب ریشه ها یکه؟یا جمعشون -1 ؟ یا این که این معادله ریشه ی حقیقی نداره؟
جوابا رو دوستمون skyzare لطف کردن با متلب به دست آوردن:

برای مشاهده محتوا ، لطفا وارد شوید یا ثبت نام کنید

davy jones
01-05-2012, 13:47
ممنون
چه جوری این نتیجه ها رو گرفتین ؟
از کجا فهمیدین ضرب ریشه ها یکه؟یا جمعشون -1 ؟ یا این که این معادله ریشه ی حقیقی نداره؟
جوابا رو دوستمون skyzare لطف کردن با متلب به دست آوردن:

برای مشاهده محتوا ، لطفا وارد شوید یا ثبت نام کنید

سلام.

در هر عبارت چندجمله ای در حالت کلی مانند زیر:


[ برای مشاهده لینک ، لطفا با نام کاربری خود وارد شوید یا ثبت نام کنید ]



همواره حاصلضرب ریشه ها برابر با [ برای مشاهده لینک ، لطفا با نام کاربری خود وارد شوید یا ثبت نام کنید ] %7D%7D هستش و حاصلجمع ریشه ها برابر با [ برای مشاهده لینک ، لطفا با نام کاربری خود وارد شوید یا ثبت نام کنید ] خواهد بود.

======================
سوال 3 در پی دی اف:

[ برای مشاهده لینک ، لطفا با نام کاربری خود وارد شوید یا ثبت نام کنید ] %5Cfrac%7Bx%5E%7B2%7D-y%5E%7B2%7D%7D%7Bx-y%7D&space;&&space;%5Ctext%7B&space;if&space;%7D&space;x%5Cneq&space;y&space;%5C%5C&space;x-y&space;&&space;%5Ctext%7B&space;if&space;%7D&space;x=y&space;%5Cend%7Bcases%7D=%5Cbegin% 7Bcases%7D&space;%5Cfrac%7B%28x-y%29%28x&plus;y%29%7D%7Bx-y%7D&space;&&space;%5Ctext%7B&space;if&space;%7D&space;x%5Cneq&space;y&space;%5C%5C&space;x-y&space;&&space;%5Ctext%7B&space;if&space;%7D&space;x=y&space;%5Cend%7Bcases%7D=%5Cbegin% 7Bcases%7D&space;x&plus;y&space;&&space;%5Ctext%7B&space;if&space;%7D&space;x%5Cneq&space;y&space;%5C%5C&space;x-y&space;&&space;%5Ctext%7B&space;if&space;%7D&space;x=y&space;%5Cend%7Bcases%7D=%5Cbegin% 7Bcases%7D&space;x&plus;y&space;&&space;%5Ctext%7B&space;if&space;%7D&space;x%5Cneq&space;y&space;%5C%5C&space;0&space;&&space;%5Ctext%7B&space;if&space;%7D&space;x=y&space;%5Cend%7Bcases%7D


واضحه که چنین تابعی پیوسته نیست. چرا که به ازای برابر بودن x و y، مقدار تابع برابر با صفره ولی در خارج از این شرط، به وضوح تابع پرش میکنه به مقادیر دیگه. پس پیوسته نیست.


----------

در سوال 4 هم کافیه که به ازای دسته معادلات y=mx که در تابع جایگذاری میکنید، ثابت کنید که حد چپ و راست تابع با هم برابر و مساوی با مقدار تابع در نقطه ی مبدا هستش. اثبات پیوستگی شرط لازم اثبات مشتق پذیریه. بعد که این کار رو انجام دادین (که خیلی راحت و در 2 خط تموم میشه) باید نشون بدین که مشتقهای پاره ای تابع هم از چپ و راست پیوسته اند که این کار هم به جهت اینکه در حقیقت با یک متغیر دارین کار میکنین، ساده است. منتها راه حل و مقدار نوشتنتون طولانی میشه. بنده هم به همین علت دیگه راه حل رو ننوشتم و فقط دارم راهنمایی میکنم. وقتی که نشون دادین مشتق پاره ای تابع بر حسب x و y هر کدوم از چپ و راست خودشون با هم برابرن، ثابت میشه که تابع در مبدا مشتق پذیره.



موفق باشین.
91/2/12

Kesel
01-05-2012, 13:56
مرسی
بعد از کجا می شه فهمید که هر چهار تا مختلطن . مثلا دو تا حقیقی و دو تا مختلط نیستن؟

davy jones
01-05-2012, 14:34
مرسی
بعد از کجا می شه فهمید که هر چهار تا مختلطن . مثلا دو تا حقیقی و دو تا مختلط نیستن؟

از روی ظاهرش نمیشه فهمید. اما در اینجا وقتی معادله ی ما قسمت چاق اتحاد چق و لاغر هست، میتونیم بفهمیم که دیگه بیشتر از این تجزیه نمیشده چون اگه تجزیه بردار بود، این اتحاد رو ادامه میدادند. همیشه به خاطر داشته باشین که قسمت چاق اتحاد چاق و لاغر، همواره دارای ریشه های مختلط هستش.


موفق باشین.
91/2/12

skyzare
01-05-2012, 16:08
سلام
سوم: اين عبارت جمله چاق در تعميم اتحاد چاق و لاغر هست!


جواب شما کاملا درسته . همونطور که گفتین راه اول و سوم یکی بود که هر دو تا قابل ارائه هستن و قبلا دیده بودم.چون توان چهار بود من خودم به ذهنم تعمیم چاق و لاغر نرسیده بود. بازم ممنون


با سلام .

این تعمیم اتحاد چاق و لاغر چیه ؟

این چاق ولاغر :


[ برای مشاهده لینک ، لطفا با نام کاربری خود وارد شوید یا ثبت نام کنید ]

*M!L4D*
01-05-2012, 16:36
با سلام .

این تعمیم اتحاد چاق و لاغر چیه ؟

این چاق ولاغر :


[ برای مشاهده لینک ، لطفا با نام کاربری خود وارد شوید یا ثبت نام کنید ]




فک کنم منظور دوستان این باشه :

[ برای مشاهده لینک ، لطفا با نام کاربری خود وارد شوید یا ثبت نام کنید ]

واسه ب علاوه هم فقط اون منهای جمله لاغر میشه + و علامت های جمله ی چاق هم یکی درمیون عوض میشن .. !

ضمن اینکه اون اتحادی هم ک نوشتین جمله ی چاقش مشکل داره . ab باید باشه نه 2ab !

*M!L4D*
01-05-2012, 16:50
سلام.

در هر عبارت چندجمله ای در حالت کلی مانند زیر:

[CENTER][ برای مشاهده لینک ، لطفا با نام کاربری خود وارد شوید یا ثبت نام کنید ]

[RIGHT]
همواره حاصلضرب ریشه ها برابر با [ برای مشاهده لینک ، لطفا با نام کاربری خود وارد شوید یا ثبت نام کنید ] %7D%7D هستش و حاصلجمع ریشه ها برابر با [ برای مشاهده لینک ، لطفا با نام کاربری خود وارد شوید یا ثبت نام کنید ] خواهد بود.




سلام
حمید جان فک کنم در مورد حاصل ضرب ریشه ها ب زوج یا فرد بودن n بستگی داشته باشه . اگه n فرد باشه میشه قرینه ی چیزی ک شما گفتین و اگه n زوج باشه همونه . اینطور نیست ؟


چه جوری این نتیجه ها رو گرفتین ؟
از کجا فهمیدین ضرب ریشه ها یکه؟یا جمعشون -1 ؟ یا این که این معادله ریشه ی حقیقی نداره؟

در واقع این معادله ریشه های مختلطی داره ک حاصلضربشون حقیقیه !!

davy jones
01-05-2012, 17:11
سلام
حمید جان فک کنم در مورد حاصل ضرب ریشه ها ب زوج یا فرد بودن n بستگی داشته باشه . اگه n فرد باشه میشه قرینه ی چیزی ک شما گفتین و اگه n زوج باشه همونه . اینطور نیست ؟



در واقع این معادله ریشه های مختلطی داره ک حاصلضربشون حقیقیه !!
علیک سلام آقا میلاد!

نه. فرقی نداره و بستگی هم به زوج و فرد بودن درجه ی چند جمله ای نداره. چرا که اگه مثلا n فرد باشه منظورتون اینه که ممکنه یه ریشه ی مختلط ظاهر بشه که کار رو خراب کنه. ولی باید دقت کنین که اگرچه مثلا درجه ی چندجمله ای فرد هستش ولی به خاطر اینکه ضرایب چند جمله ای همگی حقیقی هستند، ریشه های مختلط، همگی دو به دو مزدوج خواهند بود.

همانطور هم که واضح هستش و میدونین، حاصل جمع دو ریشه ی مختلط ِ مزدوج، همانند حاصل ضربشون، حقیقی خواهد بود. بنابراین در حاصل جمع ریشه ها، ریشه های مختلط، 2 به 2 خودشون قسمت موهومی خودشون رو حذف میکنن. همینطور در ضرب ریشه ها، ریشه های مختلط، 2 به 2 تشکیل اتحاد مزدوج میدن و حاصل همواره حقیقی خواهد بود.



[ برای مشاهده لینک ، لطفا با نام کاربری خود وارد شوید یا ثبت نام کنید ]

[ برای مشاهده لینک ، لطفا با نام کاربری خود وارد شوید یا ثبت نام کنید ]


البته شرط اصلیش هم همونیه که اول اشاره کردم و اون اینکه ضرایب چند جمله ای همگی حقیقی باشند. وگرنه ریشه های موهومی لزومی ندارد که دو به دو مزدوج باشند.



موفق باشین.
91/2/12

elahehamini
01-05-2012, 18:05
سلام.منم نمی دونم.سوال اینه:در رابطه با پیوستگی تابع زیر بحث کنید.
(x^2-Y^2) تقسیم بر x-y برای وقتی که x مخالف y است.و x-y وقتی x=y.دو ضابطه ای هست ببخشید که اینطوری نوشتم.

Mohammad Yek110
01-05-2012, 18:18
سلام
این فرمول به دست آوردن حجم کره ست

[ برای مشاهده لینک ، لطفا با نام کاربری خود وارد شوید یا ثبت نام کنید ]

دلیل این عملیات هایی که با رنگ قرمز مشخص شده چیه؟

چرا شعاع به توان 3 میشه و بعد همه عملیات تقسیم بر سه میشه؟
ممنون:11:

siyanor
01-05-2012, 18:33
--------------------------------------------------------------------------------

سلام
من يه سوال دارم اگه كسى از دوستان لطف كنه يه راهنمايى كنه ممنون ميشم
يه دايره داريم كه از سه نقطه A(2,1) , B(7.6), C(5,10) , عبور ميكنه اگه مركز دايره رو S بگيريم , مختصات S و شعاع رو بايد پيدا كنيم , يه راهش كه توسط وصل كردن دو نقطه به همديگه به هم و پيدا كردن نقطه وسط روى هر خط و معادله خط عمود بر اون و بقيه ماجراست, ولى من ميخواهم ببينم به صورت جبرى ميشه اين رو حل كرد يعنى با سه تا مجهول ؟



(x-2)^2+(y-1)^2=r^2
(x-7)^2+(y-6)^2=r2
(x-5)^2+(y-10)^2=r^2


الان چجورى ميشه معادله رو حل كرد ( فكر كنم بايد اول رو از دوم و اول رو از سوم كم كنيم تا r حذف بشه ولى خوب مطمئن نيستم )
__________________
من اينو يه جاى ديگه فورم هم تايپ كردم , ولى اينجا به نظر بهتر مياد , ممنون ميشم كمك كنيد

skyzare
01-05-2012, 18:43
--------------------------------------------------------------------------------

سلام
من يه سوال دارم اگه كسى از دوستان لطف كنه يه راهنمايى كنه ممنون ميشم
يه دايره داريم كه از سه نقطه A(2,1) , B(7.6), C(5,10) , عبور ميكنه اگه مركز دايره رو S بگيريم , مختصات S و شعاع رو بايد پيدا كنيم , يه راهش كه توسط وصل كردن دو نقطه به همديگه به هم و پيدا كردن نقطه وسط روى هر خط و معادله خط عمود بر اون و بقيه ماجراست, ولى من ميخواهم ببينم به صورت جبرى ميشه اين رو حل كرد يعنى با سه تا مجهول ؟



(x-2)^2+(y-1)^2=r^2
(x-7)^2+(y-6)^2=r2
(x-5)^2+(y-10)^2=r^2


الان چجورى ميشه معادله رو حل كرد ( فكر كنم بايد اول رو از دوم و اول رو از سوم كم كنيم تا r حذف بشه ولى خوب مطمئن نيستم )
__________________
من اينو يه جاى ديگه فورم هم تايپ كردم , ولى اينجا به نظر بهتر مياد , ممنون ميشم كمك كنيد

با سلام .

حل کردنی که از این روش هم میشه ولی خوب مسئله سر این هست که دستگاهی که به وجود میاد یه دستگاه غیر خطی هست با سه مجهول به نظرم زمان حل کردنش نسبت به روش اولی بیشتر باشه .:31: یعنی یه چیزی مثل شکل زیر :


[ برای مشاهده لینک ، لطفا با نام کاربری خود وارد شوید یا ثبت نام کنید ]



به نظرم همون روش اولی رو که خودتون رو حل کنید بهتر هست :31: البته من خودم هم نمیدونم این دستگاه ها رو چه جوری باید حل کرد ولی از همون روش های تکراری فکر کنم باید حل کرد . تازه کافی هست یه ذره اشتباه بشه دیگه کلا همه اش غلط میشه :31: این تجریه ام از محاسبات عددی هست ! :13:

Kesel
01-05-2012, 19:07
سلام
این فرمول به دست آوردن حجم کره ست

[ برای مشاهده لینک ، لطفا با نام کاربری خود وارد شوید یا ثبت نام کنید ]

دلیل این عملیات هایی که با رنگ قرمز مشخص شده چیه؟

چرا شعاع به توان 3 میشه و بعد همه عملیات تقسیم بر سه میشه؟
ممنون:11:

من دقیقا منظورتونو نمی فهمم. مگه اشکالی داره اینجوری باشه؟عملیات منظورتون چیه ؟
به دست آوردن حجم کره به طریق انتگرال گیری به دست می یاد و یکی از کاربرد های انتگراله.به طور کلی:
[ برای مشاهده لینک ، لطفا با نام کاربری خود وارد شوید یا ثبت نام کنید ]
بنابراین اگر از این عبارت مشتق بگیرید فرمول مساحت کره به دست می یاد.که A مساحت سطح مقطعی از x هست.
می خواستم براتون تایپ کنم دیدم تو اینترنت هست دیگه دوباره ننوشتم:

برای مشاهده محتوا ، لطفا وارد شوید یا ثبت نام کنید

lebesgue
01-05-2012, 19:17
سلام بر دوستان گرامی؛

کسی اثبات استقرائی برای:
اگر[ برای مشاهده لینک ، لطفا با نام کاربری خود وارد شوید یا ثبت نام کنید ]{i} ها اعداد حقیقی مثبتی از 1 تا n و [ برای مشاهده لینک ، لطفا با نام کاربری خود وارد شوید یا ثبت نام کنید ]{1}\leqslant%20P_{2} باشد، آنگاه :

[ برای مشاهده لینک ، لطفا با نام کاربری خود وارد شوید یا ثبت نام کنید ]{(\sum_{i=1}^{n}x_{i}^{P_ {2}})^{\frac{1}{P_{2}}}\leqslant%20(\sum_{i=1}^{n} x_{i}^{P_{1}})^{\frac{1}{P_{1}}}}
سراغ داره ؟
من خودم از یه روش برای اثبات k+1 امین جمله‌ش پیش رفتم ولی به نتیجه‌ی مورد نظر نرسیدم ... یعنی نتونستم جواب رو ازش بکشم بیرون ...
اگه دوست دارید روشم اینه :
به ترتیب به طرف چپ [ برای مشاهده لینک ، لطفا با نام کاربری خود وارد شوید یا ثبت نام کنید ]{k+1}^{P_{2}} و به طرف راست [ برای مشاهده لینک ، لطفا با نام کاربری خود وارد شوید یا ثبت نام کنید ]{k+1}^{P_{1}} رو کم و زیاد میکنیم.

اثبات غیر استقرائیش آسونه ...

میشه بفرمایید اثبات با استقراء ریاضی رو برای چی میخواین؟ بخصوص با وجود اثبات های ساده دیگه ای که هست.

lebesgue
01-05-2012, 19:27
.
.
.
[ برای مشاهده لینک ، لطفا با نام کاربری خود وارد شوید یا ثبت نام کنید ]
.
.
.
بنابراین تصویر نقطه ی M روی محور x ها برابر میشه با:

[ برای مشاهده لینک ، لطفا با نام کاربری خود وارد شوید یا ثبت نام کنید ] 8R-r%29%5Ccos&space;%28%5Ctheta&space;%29&plus;d%5Ccos&space;%28%5Cfrac%7BR-r%7D%7Br%7D%5Ctheta&space;%29%7D

با استدلالی مشابه، تصویر نقطه ی M روی محور y ها هم برابر میشه با:

[ برای مشاهده لینک ، لطفا با نام کاربری خود وارد شوید یا ثبت نام کنید ] 8R-r%29%5Csin&space;%28%5Ctheta&space;%29-d%5Csin&space;%28%5Cfrac%7BR-r%7D%7Br%7D%5Ctheta&space;%29%7D
.
.
.

یک مسئله جالب که میشه در شکل بالا هم دید، اینه که شعاع (و در نتیجه محیط) دایره بزرگ سه برابر دایره کوچک هست، اما دایره کوچک به ازای هر یک دور کامل که به دور دایره بزرگ میزنه، دو بار دور خودش میچرخه، نه سه بار. همچنین اگر دایره کوچک، به جای درون، بیرون دایره بزرگ میچرخید، میشد چهار دور.


نکته جالب دیگه، شکل منحنی در حالتی هست که نسبت R/r گنگ باشه:

منحنی به ازای [ برای مشاهده لینک ، لطفا با نام کاربری خود وارد شوید یا ثبت نام کنید ]


[ برای مشاهده لینک ، لطفا با نام کاربری خود وارد شوید یا ثبت نام کنید ]

منحنی به ازای [ برای مشاهده لینک ، لطفا با نام کاربری خود وارد شوید یا ثبت نام کنید ]{2}/100:


[ برای مشاهده لینک ، لطفا با نام کاربری خود وارد شوید یا ثبت نام کنید ]

تازه منحنی بالا کامل نیست و تنها برای θ از 0 تا 300π رسم شده. در منحنی کامل، تمام (یا تقریباً تمام) اون فضاهای خالی وسط، پر هستند.

lebesgue
01-05-2012, 19:36
گرادیان از لحاظ تجسمی، میزان فرو رفتگی و خاصیت سیاه چاله بودن یک نقطه از فضا رو نشون میدن.
در شکل اول سمت راست نشون داده میشه که هر برداری که از یک مقطع دلخواه به اون مقطع وارد شده، با همون مقدار و شدت، ازش خارج شده. پس گرادیان در چنین فضایی صفره.

اما در شکل سمت چپ، گرادیان در اون نقطه ی مرکزی مقدار بسیار بزرگی داره. چرا که داره بردارها رو در فضای اطرافش به سمت خودش میکشه.

در شکل دوم هم منظور از شکل اینه که مقدار گرادیان از لحاظ جبری به جهت بردارها در همسایگی یک فضای کوچک در نقطه ی مورد نظر ما بستگی داره. مثلا در قسمت آبی رنگ، چون جهت بردارها به سمت بیرون هستش، گرادیان منفی و در قسمت قرمز رنگ، مقدار گرادیان مثبته.

در مقابل مفهوم گرادیان، مفهوم کِرل وجود داره. کرل از لحاظ شهودی خاصیت گردابی و چرخش بردارها حول یک نقطه و یا یک همسایگی رو نشون میده. به طور مثال در تصویر اول سمت راست پستتون، مقدار کرل در همه ی نقاط و به ازای همه ی همسایگی ها صفره چون مقدار چرخش بردارها در جهت ساعتگرد در یک حلقه ی بسته ی فرضی (که لزوما میتونه دایره هم نباشه و فقط مهمه که حلقه ای بسته باشه)، برابر با مقدار چرخش در جهت پاد ساعتگرد در همان حلقه است.

من مظور شما رو درست متوجه نشدم، ولی فکر کنم گرادیان رو با دیورژانس قاطی کردید. بهرحال توضیحات ali_hp گرامی درست هست. در شکل های بالا، ما اول یک میدان اسکالر داشتیم (در دو شکل اول، نقاط سیاه تر متناظر با مقدار بیشتر) و بعد با گرفتن گرادیان از این میدان اسکالر، یک میدان برداری بدست اومده. میدانهای برداری رو هم معمولاً همونطوری نشون میدن، چه گرادیان باشن چه نباشن. در ضمن، کرل صفره، چون کرلِ گرادیانِ یک میدان اسکالر همواره صفره. (یک اتحاد صفر پرکاربرد!)

Kesel
01-05-2012, 19:55
--------------------------------------------------------------------------------

سلام
من يه سوال دارم اگه كسى از دوستان لطف كنه يه راهنمايى كنه ممنون ميشم
يه دايره داريم كه از سه نقطه A(2,1) , B(7.6), C(5,10) , عبور ميكنه اگه مركز دايره رو S بگيريم , مختصات S و شعاع رو بايد پيدا كنيم , يه راهش كه توسط وصل كردن دو نقطه به همديگه به هم و پيدا كردن نقطه وسط روى هر خط و معادله خط عمود بر اون و بقيه ماجراست, ولى من ميخواهم ببينم به صورت جبرى ميشه اين رو حل كرد يعنى با سه تا مجهول ؟



(x-2)^2+(y-1)^2=r^2
(x-7)^2+(y-6)^2=r2
(x-5)^2+(y-10)^2=r^2


الان چجورى ميشه معادله رو حل كرد ( فكر كنم بايد اول رو از دوم و اول رو از سوم كم كنيم تا r حذف بشه ولى خوب مطمئن نيستم )
__________________
من اينو يه جاى ديگه فورم هم تايپ كردم , ولى اينجا به نظر بهتر مياد , ممنون ميشم كمك كنيد

سلام

من دو راه حل فعلا به ذهنم می رسه :
1.
بهتره معادله ی دایره ی مطلوب رو به فرم زیر بنویسیم.
[ برای مشاهده لینک ، لطفا با نام کاربری خود وارد شوید یا ثبت نام کنید ]
چون این سه نقطه بر دایره واقع هستن بنابراین مختصات این نقاط در دایره صدق می کنه.بنابراین داریم :
[ برای مشاهده لینک ، لطفا با نام کاربری خود وارد شوید یا ثبت نام کنید ]
حالا با به دست آوردن a و b و c و جایگزین کردنشون توی معادله ی دایره ، می تونیم معادله ی دایره رو داشته باشیم و راحت می شه از روش زیر مختصات مرکز و همچنین شعاع دایره رو پیدا کرد.
[ برای مشاهده لینک ، لطفا با نام کاربری خود وارد شوید یا ثبت نام کنید ]
تو این مرحله پارسال پیش دانشگاهی یه روش تستی خوب هم داشت که دو تا فرمول بالا رو حفظ نکنیم که کلا توی مقاطع مخروطی به درد می خورد.اونم این بود که واسه پیدا کردن مختصات مرکز مقطع مخروطی مورد نظر ، از معادلش مشتق می گیریم(نسبت به x) برابر صفر قرار می دیم این می شه طول مرکز بعد یا می تونیم همینو بزاریم تو معادله تا عرض اون نقطه هم به دست بیاد یا این دفعه نسبت به y مشتق بگیریم و برابر صفر قرار بدیم.روش به درد بخوریه لازمم نیست فرمول اضافی حفظ بشه !
2.
می دونیم که مرکز دایره از هر سه نقطه به یک فاصله یه.بنابراین با فرض این که مرکز دایره مختصاتش (x,y) باشه ، داریم :
[ برای مشاهده لینک ، لطفا با نام کاربری خود وارد شوید یا ثبت نام کنید ]
که اینجا با پیدا کردن ایکس و وای می تونین یه ضرب مختصات مرکزو به دست بیارین بعد توی معادله ی دایره بزارین تا شعاع هم به دست بیاد :
[ برای مشاهده لینک ، لطفا با نام کاربری خود وارد شوید یا ثبت نام کنید ]

lebesgue
01-05-2012, 20:20
لطفاً حذف شود.

Ship Storm
01-05-2012, 20:21
سلام حامد جان! از این ورا؟ :31:
طبق معمول همیشه شما سوالاتت رو بذار. منتها کیه که جواب بده؟ :31:
====================
91/2/12

مخلصم حمید جان
چیزه :31: من دیفرانسیل رو موفق شدم با پروفسور بهمن مهری پاس کنم با همون اوضاعی که داشتم اگه یادت باشه :31: الان رسیدم به درس ریاضی مهندسی و میخوام این ترم این رو بپاسونم اما چون سرکار هستم نمیرسم اصلا سر کلاس ها برم بخاطر همین نمونه سوالات ترم قبل رو گرفتم تقریبا طی چند امتحان گذشته سواللات مثل هم بوده گفتم منم چندتا از این نمونه سوالا بلد باشم میشه تو این درس نمره آورد
با اجازه من اسکن میکنم اولین نمونه سوال ها رو
اجرکم عندالله ...
و من الله التوفیق
1391/02/13
---
[ برای مشاهده لینک ، لطفا با نام کاربری خود وارد شوید یا ثبت نام کنید ]

lebesgue
01-05-2012, 20:22
لطفاً حذف شود.

lebesgue
01-05-2012, 20:29
در هر عبارت چندجمله ای در حالت کلی مانند زیر:

[ برای مشاهده لینک ، لطفا با نام کاربری خود وارد شوید یا ثبت نام کنید ]

همواره حاصلضرب ریشه ها برابر با [ برای مشاهده لینک ، لطفا با نام کاربری خود وارد شوید یا ثبت نام کنید ] %7D%7D هستش و حاصلجمع ریشه ها برابر با [ برای مشاهده لینک ، لطفا با نام کاربری خود وارد شوید یا ثبت نام کنید ] خواهد بود.




یک چند جلمه ای درجه n با ریشه های [ برای مشاهده لینک ، لطفا با نام کاربری خود وارد شوید یا ثبت نام کنید ] به صورت زیر معین میشه:


[ برای مشاهده لینک ، لطفا با نام کاربری خود وارد شوید یا ثبت نام کنید ](z)=(z-z_1)(z-z_2)%5Ccdots%20(z-z_n)

ضریب [ برای مشاهده لینک ، لطفا با نام کاربری خود وارد شوید یا ثبت نام کنید ]^n برای سادگی برابر 1 فرض شده. با انجام ضرب میشه دید که ضریب [ برای مشاهده لینک ، لطفا با نام کاربری خود وارد شوید یا ثبت نام کنید ]^{n-1} برابر منفی مجموع ریشه ها هست و ضریب [ برای مشاهده لینک ، لطفا با نام کاربری خود وارد شوید یا ثبت نام کنید ]^{n-2} برابر مثبت مجموع حاصلضربهای دوتایی ریشه ها و ضریب [ برای مشاهده لینک ، لطفا با نام کاربری خود وارد شوید یا ثبت نام کنید ]^{n-3} برابر منفی مجموع حاصلضربهای سه تایی ریشه ها و... به همین ترتیب یک درمیون مثبت و منفی. همچنین میشه دید که علامت ضریب [ برای مشاهده لینک ، لطفا با نام کاربری خود وارد شوید یا ثبت نام کنید ]^0 به زوج یا فرد بودن n بستگی داره.


---------- Post added at 09:29 PM ---------- Previous post was at 09:28 PM ----------



--------------------------------------------------------------------------------

سلام
من يه سوال دارم اگه كسى از دوستان لطف كنه يه راهنمايى كنه ممنون ميشم
يه دايره داريم كه از سه نقطه A(2,1) , B(7.6), C(5,10) , عبور ميكنه اگه مركز دايره رو S بگيريم , مختصات S و شعاع رو بايد پيدا كنيم , يه راهش كه توسط وصل كردن دو نقطه به همديگه به هم و پيدا كردن نقطه وسط روى هر خط و معادله خط عمود بر اون و بقيه ماجراست, ولى من ميخواهم ببينم به صورت جبرى ميشه اين رو حل كرد يعنى با سه تا مجهول ؟



(x-2)^2+(y-1)^2=r^2
(x-7)^2+(y-6)^2=r2
(x-5)^2+(y-10)^2=r^2


الان چجورى ميشه معادله رو حل كرد ( فكر كنم بايد اول رو از دوم و اول رو از سوم كم كنيم تا r حذف بشه ولى خوب مطمئن نيستم )
__________________
من اينو يه جاى ديگه فورم هم تايپ كردم , ولى اينجا به نظر بهتر مياد , ممنون ميشم كمك كنيد
بله راحت ترین کار همین هست. با اینکار دو معادله خطی (بدون x^2 یا y^2) بدست میاد.

Mohammad Yek110
01-05-2012, 20:31
من دقیقا منظورتونو نمی فهمم. مگه اشکالی داره اینجوری باشه؟عملیات منظورتون چیه ؟
به دست آوردن حجم کره به طریق انتگرال گیری به دست می یاد و یکی از کاربرد های انتگراله.به طور کلی:
[ برای مشاهده لینک ، لطفا با نام کاربری خود وارد شوید یا ثبت نام کنید ]
بنابراین اگر از این عبارت مشتق بگیرید فرمول مساحت کره به دست می یاد.که A مساحت سطح مقطعی از x هست.
می خواستم براتون تایپ کنم دیدم تو اینترنت هست دیگه دوباره ننوشتم:

برای مشاهده محتوا ، لطفا وارد شوید یا ثبت نام کنید

سلام
ممنون دوست عزیز:11:
این یه سوال تحقیقی سوم راهنمایی هست و معلم ما میگه دلیلشو پیدا کنید که چرا این کارو می کنن
ما سوم راهنمایی هستیم اکه میشه فارسی صحبت کنید:دی:دی

Kesel
01-05-2012, 21:58
سلام
ممنون دوست عزیز:11:
این یه سوال تحقیقی سوم راهنمایی هست و معلم ما میگه دلیلشو پیدا کنید که چرا این کارو می کنن
ما سوم راهنمایی هستیم اکه میشه فارسی صحبت کنید:دی:دی

خواهش می کنم !
این سوالا یه کم زوده واسه سوم راهنمایی فک کنم . مخصوصا نسل شما که تقریبا هیچی تو کتاباتون جز نقاشی نیست (توهین نشه !)
این سوال خیلی بخوایم ارفاق کنیم مال پیش دانشگاهیه که هم تابع می دونن چیه هم حد چیه هم مشتق و هم انتگرال.ولی در کل مال ترم اول دانشگاهه.(ریاضی عمومی 1 فصل کاربرد انتگرال)
من فقط سرچ کردم دیدم مهم ترین روشی که در گذشته استفاده می شده روشی بوده که ارشمیدس ابداع کرده بوده . به طوری که این رو بزرگترین دستاورد ریاضی خودش می دونسته ! بعضی جاها اسم استوانه و دایره هم روش گذاشتن که ثابت کرده حجم کره دو سوم حجم سیلندر یا استوانه ی محیط شدشه لینک زیر قسمتی از کتاب دو جلدی ارشمیدس در همین حوزه ها هست که 225 سال قبل از میلاد مسیح تالیف کرده

برای مشاهده محتوا ، لطفا وارد شوید یا ثبت نام کنید
[ برای مشاهده لینک ، لطفا با نام کاربری خود وارد شوید یا ثبت نام کنید ]
آرامگاه ارشمیدس هم به همین شکل ساخته شده.
اما روش انتگرال:
همونی که تو ویکی پدیا بود به زبون ساده تر:
وقتی ما یه صفحه رو از توی یک کره رد کنیم قسمتی که توی کره برش می خوره به شکل دایره هست مث این:
[ برای مشاهده لینک ، لطفا با نام کاربری خود وارد شوید یا ثبت نام کنید ]پرونده:نیم_کره.png
با توجه به عکس:

برای مشاهده محتوا ، لطفا وارد شوید یا ثبت نام کنید
طبق رابطه ی فیثاغورس داریم :
[ برای مشاهده لینک ، لطفا با نام کاربری خود وارد شوید یا ثبت نام کنید ]
حالا مساحت اون دایره ای که انگار وقتی کره رو بریدیم به وجود اومده می شه این زیری و می دونیم که شعاع اون دایره حالا y هست دیگه و مساحت دایره رو هم که می دونین
اسم مساحتشو می زارم A
[ برای مشاهده لینک ، لطفا با نام کاربری خود وارد شوید یا ثبت نام کنید ]
متاسفانه فقط تا همینجاش در سطح راهنمائیه.بعدش از فرمولی که گفتم اینجوری به دست می یاد:
[ برای مشاهده لینک ، لطفا با نام کاربری خود وارد شوید یا ثبت نام کنید ]
دیگه بیشتر از این بلد نیستم !

MasterGeek
02-05-2012, 01:17
سلام
این فرمول به دست آوردن حجم کره ست

[ برای مشاهده لینک ، لطفا با نام کاربری خود وارد شوید یا ثبت نام کنید ]

دلیل این عملیات هایی که با رنگ قرمز مشخص شده چیه؟

چرا شعاع به توان 3 میشه و بعد همه عملیات تقسیم بر سه میشه؟
ممنون:11:

اینجا همون اثبات ارشمیدسی رو توضیح داده:
[ برای مشاهده لینک ، لطفا با نام کاربری خود وارد شوید یا ثبت نام کنید ]

zr1
02-05-2012, 10:06
درود به همه کسایی که تو این قسمت فعالیت میکنن.دوستان قبل از اینکه سوالام رو مطرح کنم یه چیزی رو میخواستم بپرسم که ممکنه راهنمایی های شما به درد من و دوستام بخوره.این ترم ما درس دیفرانسیل داریم و درسای استاد رو سر کلاس خوب متوجه میشیم و میتونیم مثالای استاد رو حل کنیم اما وقتی میاییم مثالایی که استاد از کتاب سیمونز مشخص کرده رو میخاییم حل کنیم یا همون اول نمیتونیم حل کنیم یا تا نصفه میریم(معلوم هم نیست که درست حل کرده باشیم).من خودم کتاب دکتر نیکوکار رو هم گرفتم که از اون هم به درسام میرسم ولی بازم خود من هم نمیتونم بعضی از مثالای این کتاب رو حل کنم.ممکنه که اول کار باشیم ولی اگه راهنمایی با تجربه ای چیزی دارین بگین ممنون میشم.
نشان دهید که معادلات زیر همگن هستند و انها را حل کنید
[ برای مشاهده لینک ، لطفا با نام کاربری خود وارد شوید یا ثبت نام کنید ]

[ برای مشاهده لینک ، لطفا با نام کاربری خود وارد شوید یا ثبت نام کنید ]

[ برای مشاهده لینک ، لطفا با نام کاربری خود وارد شوید یا ثبت نام کنید ] %7Bdx%7D=ysin%5Cfrac%7By%7D%7Bx%7D&plus;x
بازم سوال دارم و ممنون میشم که کمکم کنین:11:

davy jones
02-05-2012, 20:13
من مظور شما رو درست متوجه نشدم، ولی فکر کنم گرادیان رو با دیورژانس قاطی کردید. بهرحال توضیحات ali_hp گرامی درست هست. در شکل های بالا، ما اول یک میدان اسکالر داشتیم (در دو شکل اول، نقاط سیاه تر متناظر با مقدار بیشتر) و بعد با گرفتن گرادیان از این میدان اسکالر، یک میدان برداری بدست اومده. میدانهای برداری رو هم معمولاً همونطوری نشون میدن، چه گرادیان باشن چه نباشن. در ضمن، کرل صفره، چون کرلِ گرادیانِ یک میدان اسکالر همواره صفره. (یک اتحاد صفر پرکاربرد!)
درسته. با دیورژانس اشتباه شد :31:

ممنون از تذکرتون

skyzare
02-05-2012, 20:20
عدد حالت در دستگاه معادلات غیر خطی ( condition number in nonlinear equation )

با سلام .

اساتید ببخشید می خواستم بدونم فرمول عدد حالت در دستگاه معادلات غیر خطی چیه ؟ توی خطی رو میدونم .

با اون یکی هست ؟؟ یا فرق داره ؟

این برای خطی هست :


[ برای مشاهده لینک ، لطفا با نام کاربری خود وارد شوید یا ثبت نام کنید ]



بعد یه چیز دیگه فرق این که از کدام نرم به دست بیاریم چیه ؟









condition’ of the equations: l

a technical term used to describe how sensitive the solution ([ برای مشاهده لینک ، لطفا با نام کاربری خود وارد شوید یا ثبت نام کنید ] forum%2Fvbglossar.php%3Fdo%3Dshowentry%26item%3Dso lution) is to changes in the righthand side or coefficient matrix



================================


What do you mean by ill-conditioned and well-conditioned system of equations ? l

well-conditioned system : l
A system of equations is considered to be well-conditioned if a small change in the
coefficient matrix or a small change in the right hand side results in a small change in the
solution vector

ill-conditioned : l

A system of equations is considered to be ill-conditioned if a small change in the
coefficient matrix or a small change in the right hand side results in a large change in the
solution vector

hts1369
02-05-2012, 21:32
درود به همه کسایی که تو این قسمت فعالیت میکنن.دوستان قبل از اینکه سوالام رو مطرح کنم یه چیزی رو میخواستم بپرسم که ممکنه راهنمایی های شما به درد من و دوستام بخوره.این ترم ما درس دیفرانسیل داریم و درسای استاد رو سر کلاس خوب متوجه میشیم و میتونیم مثالای استاد رو حل کنیم اما وقتی میاییم مثالایی که استاد از کتاب سیمونز مشخص کرده رو میخاییم حل کنیم یا همون اول نمیتونیم حل کنیم یا تا نصفه میریم(معلوم هم نیست که درست حل کرده باشیم).من خودم کتاب دکتر نیکوکار رو هم گرفتم که از اون هم به درسام میرسم ولی بازم خود من هم نمیتونم بعضی از مثالای این کتاب رو حل کنم.ممکنه که اول کار باشیم ولی اگه راهنمایی با تجربه ای چیزی دارین بگین ممنون میشم.
نشان دهید که معادلات زیر همگن هستند و انها را حل کنید
[ برای مشاهده لینک ، لطفا با نام کاربری خود وارد شوید یا ثبت نام کنید ]

[ برای مشاهده لینک ، لطفا با نام کاربری خود وارد شوید یا ثبت نام کنید ]

[ برای مشاهده لینک ، لطفا با نام کاربری خود وارد شوید یا ثبت نام کنید ] %7Bdx%7D=ysin%5Cfrac%7By%7D%7Bx%7D&plus;x
بازم سوال دارم و ممنون میشم که کمکم کنین:11:
من دومی و سومی رو برات حل میکنم.
خب این نشون دهنده ی همگن بودن تابع. نمیدونم به شما چی گفتن ولی با این چیزی که من دارم میگم هیچ فرقی نداره


[ برای مشاهده لینک ، لطفا با نام کاربری خود وارد شوید یا ثبت نام کنید ] ac%7By%7D%7Bx%7D%5Cfrac%7Bdy%7D%7Bdx%7D=y%5Csin&space;%5 Cfrac%7By%7D%7Bx%7D&plus;x%5CRightarrow&space;tx%5Csin&space;%5Cfra c%7Bty%7D%7Btx%7D%5Cfrac%7Bdy%7D%7Bdx%7D=ty%5Csin&space; %5Cfrac%7Bty%7D%7Btx%7D&plus;tx&space;%5C%5C&space;tx%5Csin&space;%5Cfrac %7By%7D%7Bx%7D%5Cfrac%7Bdy%7D%7Bdx%7D=ty%5Csin&space;%5C frac%7By%7D%7Bx%7D&plus;tx%5CRightarrow&space;tx%5Csin&space;%5Cfra c%7By%7D%7Bx%7D%5Cfrac%7Bdy%7D%7Bdx%7D=t%5Cleft%28 y%5Csin&space;%5Cfrac%7By%7D%7Bx%7D&plus;x%5Cright%29%5CRight arrow&space;x%5Csin&space;%5Cfrac%7By%7D%7Bx%7D%5Cfrac%7Bdy%7D %7Bdx%7D=y%5Csin&space;%5Cfrac%7By%7D%7Bx%7D&plus;x&space;%5Cend%7B array%7D

شما باید با یه تغییر متغیر این تیپ سوالها رو حل کنی

[ برای مشاهده لینک ، لطفا با نام کاربری خود وارد شوید یا ثبت نام کنید ] ac%7By%7D%7Bx%7D%5Cfrac%7Bdy%7D%7Bdx%7D=y%5Csin&space;%5 Cfrac%7By%7D%7Bx%7D&plus;x%5CRightarrow&space;%5Cfrac%7Bdy%7D %7Bdx%7D=%5Cfrac%7By%5Csin&space;%5Cfrac%7By%7D%7Bx%7D&plus;x %7D%7Bx%5Csin&space;%5Cfrac%7By%7D%7Bx%7D%7D=%5Cfrac%7By %7D%7Bx%7D&plus;%5Cfrac%7B1%7D%7B%5Csin&space;%5Cfrac%7By%7D% 7Bx%7D%7D=z&plus;%5Cfrac%7B1%7D%7B%5Csin&space;z%7D&space;%5C%5C&space;%5 Cfrac%7By%7D%7Bx%7D=z%5CRightarrow&space;y=xz%5CRightarr ow&space;%5Cfrac%7Bdy%7D%7Bdx%7D=z&plus;x%5Cfrac%7Bdz%7D%7Bdx %7D&space;%5C%5C&space;%5Cfrac%7Bdy%7D%7Bdx%7D=z&plus;%5Cfrac%7B1%7 D%7B%5Csin&space;z%7D=z&plus;x%5Cfrac%7Bdz%7D%7Bdx%7D%5CRight arrow&space;x%5Cfrac%7Bdz%7D%7Bdx%7D=%5Cfrac%7B1%7D%7B%5 Csin&space;z%7D%5CRightarrow&space;%5Csin&space;zdz=%5Cfrac%7Bdx%7D% 7Bx%7D%5CRightarrow&space;-%5Ccos&space;z=%5Cln&space;x&plus;c&space;%5C%5C&space;-%5Ccos&space;%5Cfrac%7By%7D%7Bx%7D=%5Cln&space;x&plus;c%5CRightarro w&space;%5Carccos&space;%5Cleft%28%5Ccos&space;%5Cfrac%7By%7D%7Bx%7D %5Cright%29=%5Carccos&space;%28-%5Cln&space;x-c%29%5CRightarrow&space;y=x%5Carccos&space;%28-%5Cln&space;x-c%29&space;%5Cend%7Barray%7D
ایشالله متوجه که شدی؟
بریم سراغ سوال بعدی همگن بودن این تابع هم مثله قبلی خیلی راحت نشون داده میشه که بعنوان تمرین به خودتون واگذار میکنم.(از بچه های اینجا چه چیزهایی یاد گرفتم:31:)

[ برای مشاهده لینک ، لطفا با نام کاربری خود وارد شوید یا ثبت نام کنید ] Cleft%28x%5E2&plus;y%5E2%5Cright%29%5Ctan&space;%5E%7B-1%7D%5Cfrac%7By%7D%7Bx%7D&plus;xy%5CRightarrow&space;y%27=%5C frac%7B3%5Cleft%28x%5E2&plus;y%5E2%5Cright%29%5Ctan&space;%5E %7B-1%7D%5Cfrac%7By%7D%7Bx%7D&plus;xy%7D%7Bx%5E2%7D&space;%5C%5C&space; %5Cfrac%7Bdy%7D%7Bdx%7D=3%5Cleft%281&plus;%5Cfrac%7By%5 E2%7D%7Bx%5E2%7D%5Cright%29%5Ctan&space;%5E%7B-1%7D%5Cfrac%7By%7D%7Bx%7D&plus;%5Cfrac%7By%7D%7Bx%7D&space;%5 C%5C&space;%5Cfrac%7By%7D%7Bx%7D=z%5CRightarrow&space;y=xz%5CR ightarrow&space;%5Cfrac%7Bdy%7D%7Bdx%7D=z&plus;x%5Cfrac%7Bdz% 7D%7Bdx%7D&space;%5C%5C&space;%5Cfrac%7Bdy%7D%7Bdx%7D=3%5Cleft %281&plus;z%5E2%5Cright%29%5Ctan&space;%5E%7B-1%7Dz&plus;z%5CRightarrow&space;z&plus;x%5Cfrac%7Bdz%7D%7Bdx%7D=3% 5Cleft%281&plus;z%5E2%5Cright%29%5Ctan&space;%5E%7B-1%7Dz&plus;z%5CRightarrow&space;%5Cfrac%7Bdz%7D%7Bdx%7D=%5Cfr ac%7B3%5Cleft%281&plus;z%5E2%5Cright%29%5Ctan&space;%5E%7B-1%7Dz%7D%7Bx%7D&space;%5C%5C&space;%5Cfrac%7Bdz%7D%7Bdx%7D=%5C frac%7B3%5Cleft%281&plus;z%5E2%5Cright%29%5Ctan&space;%5E%7B-1%7Dz%7D%7Bx%7D%5CRightarrow&space;%5Cfrac%7Bdz%7D%7B3%5 Cleft%281&plus;z%5E2%5Cright%29%5Ctan&space;%5E%7B-1%7Dz%7D=%5Cfrac%7Bdx%7D%7Bx%7D%5CRightarrow&space;%5Cfr ac%7B1%7D%7B3%7D%5Cln&space;%5Ctan&space;%5E%7B-1%7Dz=%5Cln&space;x&plus;c&space;%5C%5C&space;%5Cfrac%7B1%7D%7B3%7D%5Cln&space; %5Ctan&space;%5E%7B-1%7D%5Cfrac%7By%7D%7Bx%7D=%5Cln&space;x&plus;c&space;%5Cend%7Barray %7D

تا همین جا هم بسه و نیازی به محاسبه ی بیشتری نیست.
من خودم این ترم معادلات دارم. واقعا چیزه خیلی باحالیه
در مورد سیمونز من دو تا ترجمه از این کتاب دیدم یکی ماله نشر دانشگاهی هست (که من دارمش) آخر این کتاب جواب یکسری از سوالات رو داره (سوالات فرد)
ولی یه ترجمه دیگه هست که نمیدونم چرا جواب سوالات رو آخر کتاب ننوشته ولی مزیت این کتاب نسبت که کتاب نشر دانشگاهی اینه که چون از ویرایش جدیدتری هست بیشتر تمرین داره
دو کتاب رو با هم مقایسه کردم و دیدم.

siyanor
03-05-2012, 10:17
دوستان يه سوال درباره توابع فرض كنيم كه f(x)=x-2/x+3 وm(x)=1/sqrtx^(2)-5x+6 الان اگه از ما بخواند كه دومين f(x) و g(x) و (fog) رو پيدا كنيم بايد چيكار بكنيم ؟من خودم كارى كه كردم براى اولى نوشتم كه

X+2≠0⟶x≠-2 از اشتراكش با دومين صورت تقسيم , دومين ميشه R-{-2}
واسه معادله دوم هم داريم (چون جذر هست و در پايين كسر هم قرار داره از علامت بزرگتر استفاده ميكنيم < )
x^2-5x+6>0 →(X-3)(X-2)>0 →
كه ميشه
x-3>0 →x>3
x-2>0 →x>2
يعنى دومين ميشه R -<2,3
براى آخرى هم ميشه
F(g(x))=1/sqrt(x^2-5x+6) -2 / 1/ sqrt(x^2-5x+6) +3→1-2sqrt(x^2-5x+6) / 1+3 sqrt (x^2-5x+6) so
دومين اين مساله رو كسى ميتونه به من كمك كنه پيداش كنم ؟

hts1369
03-05-2012, 19:46
دوستان يه سوال درباره توابع فرض كنيم كه f(x)=x-2/x+3 وm(x)=1/sqrtx^(2)-5x+6 الان اگه از ما بخواند كه دومين f(x) و g(x) و (fog) رو پيدا كنيم بايد چيكار بكنيم ؟من خودم كارى كه كردم براى اولى نوشتم كه

X+2≠0⟶x≠-2 از اشتراكش با دومين صورت تقسيم , دومين ميشه R-{-2}
واسه معادله دوم هم داريم (چون جذر هست و در پايين كسر هم قرار داره از علامت بزرگتر استفاده ميكنيم < )
x^2-5x+6>0 →(X-3)(X-2)>0 →
كه ميشه
x-3>0 →x>3
x-2>0 →x>2
يعنى دومين ميشه R -<2,3
براى آخرى هم ميشه
F(g(x))=1/sqrt(x^2-5x+6) -2 / 1/ sqrt(x^2-5x+6) +3→1-2sqrt(x^2-5x+6) / 1+3 sqrt (x^2-5x+6) so
دومين اين مساله رو كسى ميتونه به من كمك كنه پيداش كنم ؟
لطفا سوالتون رو تو یک تایپیک بپرسید. ترجیحا همین جا چون برای اینکار ایجاد شده.
ممنون که توجه میکنید.
منظورتون رو از "دومین" نمیفهم منظورتون ( f(f(x هست؟
اگه اره شما دامنه ی این توابع رو میخواهید یا خود ظابطه ی سوالاتتون رو؟

hts1369
03-05-2012, 20:09
این انتگرال چطور حل میشه؟
[ برای مشاهده لینک ، لطفا با نام کاربری خود وارد شوید یا ثبت نام کنید ] 5E3&plus;8%5Cright%29dx
جوابش اینه (با متمتیکا زدم و اینو گرفتم) ولی موندم چطور میشه حلش کرد؟شاید هم ساده باشه ولی نمیدونم چطور حلش کنم.
[ برای مشاهده لینک ، لطفا با نام کاربری خود وارد شوید یا ثبت نام کنید ] 7D&plus;%5Cfrac%7B3&space;%5Ctext%7BLnx%7D%5E8%7D%7B8%7D

siyanor
03-05-2012, 22:23
سلام دوست عزيز ,ممنون واسه توصيه تون :10:
دو تا تابع داريم كه دامنه هر كدام ازمون خواسته شده و دامنه (foG)(x) من خودم يه چيزاى رو حل كردم ولى مطمئن نيستم
[ برای مشاهده لینک ، لطفا با نام کاربری خود وارد شوید یا ثبت نام کنید ]



واسه f(x) داريم X+3≠0→x≠-3→D=R-{-3}
واسه D(x) هم داريم
[ برای مشاهده لینک ، لطفا با نام کاربری خود وارد شوید یا ثبت نام کنید ]

كسى از دوستان ميتونه تو حل F(g(x)) كمكم كنه , هم تابع و هم دامنه اونو


كسى از دوستان ميتونه تو حل F(g(x)) كمكم كنه , هم تابع و هم دامنه اونو چجورى ميشه پيدا كرد ؟
با تشكر

hts1369
04-05-2012, 09:20
سلام دوست عزيز ,ممنون واسه توصيه تون :10:
دو تا تابع داريم كه دامنه هر كدام ازمون خواسته شده و دامنه (foG)(x) من خودم يه چيزاى رو حل كردم ولى مطمئن نيستم
[ برای مشاهده لینک ، لطفا با نام کاربری خود وارد شوید یا ثبت نام کنید ]



واسه f(x) داريم X+3≠0→x≠-3→D=R-{-3}
واسه D(x) هم داريم
[ برای مشاهده لینک ، لطفا با نام کاربری خود وارد شوید یا ثبت نام کنید ]

كسى از دوستان ميتونه تو حل F(g(x)) كمكم كنه , هم تابع و هم دامنه اونو


كسى از دوستان ميتونه تو حل F(g(x)) كمكم كنه , هم تابع و هم دامنه اونو چجورى ميشه پيدا كرد ؟
با تشكر
سلام
خب برای و f که g راحته

[ برای مشاهده لینک ، لطفا با نام کاربری خود وارد شوید یا ثبت نام کنید ] rac%7Bx-2%7D%7Bx&plus;3%7D%5CRightarrow&space;Df=R-%5C%7B-3%5C%7D&space;%5C%5C&space;g%28x%29=%5Cfrac%7B1%7D%7B%5Csqrt%7 Bx%5E2-5x&plus;6%7D%7D%5CRightarrow&space;Dg=R-[2,3]&space;%5Cend%7Barray%7D

و برای fog

[ برای مشاهده لینک ، لطفا با نام کاربری خود وارد شوید یا ثبت نام کنید ] 28g%28x%29%29=%5Cfrac%7B%5Cfrac%7B1%7D%7B%5Csqrt%7 Bx%5E2-5x&plus;6%7D%7D-2%7D%7B%5Cfrac%7B1%7D%7B%5Csqrt%7Bx%5E2-5x&plus;6%7D%7D&plus;3%7D&space;%5C%5C&space;Dfog%5Cleft%5C%7B&space;%5Cbegin% 7Barray%7D%7Bc%7D&space;%5Csqrt%7Bx%5E2-5x&plus;6%7D%3E0&space;%5C%5C&space;%5Cfrac%7B1%7D%7B%5Csqrt%7Bx%5E 2-5x&plus;6%7D%7D&plus;3%3E0&space;%5Cend%7Barray%7D&space;%5CRightarrow&space;D fog=R-[2,3]%5Cright.&space;%5C%5C&space;fog%28x%29=%5Cfrac%7B%5Cfrac%7B1-2%5Csqrt%7Bx%5E2-5x&plus;6%7D%7D%7B%5Csqrt%7Bx%5E2-5x&plus;6%7D%7D%7D%7B%5Cfrac%7B1&plus;3%5Csqrt%7Bx%5E2-5x&plus;6%7D%7D%7B%5Csqrt%7Bx%5E2-5x&plus;6%7D%7D%7D=%5Cfrac%7B1-2%5Csqrt%7Bx%5E2-5x&plus;6%7D%7D%7B1&plus;3%5Csqrt%7Bx%5E2-5x&plus;6%7D%7D&space;%5Cend%7Barray%7D

میبینی که میتونیم fog رو ساده کنیم ولی من برای محاسبه ی دامنه ی تابع همون اول بسم الله اقدام کردم ولی بعدا میتونیم ساده تر بکنیمش .از این هم ساده تر میشه (با گویا کردن مخرج کسر و اینجور کارها) ولی برای محاسبه ی دامنه تابع حق نداریم از این کارها بکنیم چون مثلا رادیکال از بین میره و این کار باعث میشه دامنه ی تابع اشتباه بدست بیاد.

zr1
04-05-2012, 13:34
درود دوباره به همه عزیزان.ممنون hts 1369 که کمکم کردی.اشکالامو تو این هفته مطرح میکنم
دوستان دیروز استادمون یه عالمه تمرین داد بریم حل کنیم و هفته بعد 5شنبه می خواد نمره میان ترم بده و اوضاع جالب نیست!(استادای دیگه پروژه هم از ما میخوان اصلا نمیشه به همشون رسید)
من سوالامو تو چند قسمت تقسیم میکنم و هر روز اینجا قرار میدم. ممنون میشم کمکم کنین:43:شرمنده همه.
هر یک از معادلات زیر را با به دست اوردن یک عامل انتگرال ساز حل کنید
[ برای مشاهده لینک ، لطفا با نام کاربری خود وارد شوید یا ثبت نام کنید ]

[ برای مشاهده لینک ، لطفا با نام کاربری خود وارد شوید یا ثبت نام کنید ]
معادلات برنولی
[ برای مشاهده لینک ، لطفا با نام کاربری خود وارد شوید یا ثبت نام کنید ]

[ برای مشاهده لینک ، لطفا با نام کاربری خود وارد شوید یا ثبت نام کنید ]
تمرین های گوناگون
[ برای مشاهده لینک ، لطفا با نام کاربری خود وارد شوید یا ثبت نام کنید ]

davy jones
04-05-2012, 15:35
این انتگرال چطور حل میشه؟
[ برای مشاهده لینک ، لطفا با نام کاربری خود وارد شوید یا ثبت نام کنید ] 5E3&plus;8%5Cright%29dx
جوابش اینه (با متمتیکا زدم و اینو گرفتم) ولی موندم چطور میشه حلش کرد؟شاید هم ساده باشه ولی نمیدونم چطور حلش کنم.
[ برای مشاهده لینک ، لطفا با نام کاربری خود وارد شوید یا ثبت نام کنید ] 7D&plus;%5Cfrac%7B3&space;%5Ctext%7BLnx%7D%5E8%7D%7B8%7D

سلام

جوابی که نوشتین درست نیست و کافیه که خودتون برای اطمینان از جواب آخر مشتق بگیرین تا متوجه بشین.

جوابش یکمی طولانیه و بنده تا اونجایی که دیگه خودتون به راحتی بتونین تا آخر ادامه بدین، نوشتم:


[ برای مشاهده لینک ، لطفا با نام کاربری خود وارد شوید یا ثبت نام کنید ] %5E%7B3%7D&plus;8%29dx%5Crightarrow&space;u=%5Cln&space;%283x%5E%7B 3%7D&plus;8%29%5C;&space;%5C;&space;,%5C;&space;%5C;&space;dv=x%5E%7B4%7Ddx%5C: &space;%5Crightarrow&space;du=%5Cfrac%7B9x%5E%7B2%7D%7D%7B3x%5 E%7B3%7D&plus;8%7Ddx%5C;&space;%5C;&space;,%5C;&space;%5C;&space;v=%5Cfrac%7Bx% 5E%7B5%7D%7D%7B5%7D%5CRightarrow&space;I=%5Cint&space;u%5C,.&space;d v=u.v-%5Cint&space;v%5C,&space;.du=%5Cfrac%7Bx%5E%7B5%7D%7D%7B5%7D%5 Cln&space;%283x%5E%7B3%7D&plus;8%29-%5Cfrac%7B9%7D%7B5%7D%5Cint&space;%5Cfrac%7Bx%5E%7B7%7D% 7D%7B3x%5E%7B3%7D&plus;8%7Ddx

[ برای مشاهده لینک ، لطفا با نام کاربری خود وارد شوید یا ثبت نام کنید ] D%5Cln&space;%283x%5E%7B3%7D&plus;8%29-%5Cfrac%7B3%7D%7B5%7D%5Cint&space;%5Cfrac%7Bx%5E%7B7%7D% 7D%7Bx%5E%7B3%7D&plus;%5Cfrac%7B8%7D%7B3%7D%7Ddx=%5Cfra c%7Bx%5E%7B5%7D%7D%7B5%7D%5Cln&space;%283x%5E%7B3%7D&plus;8%2 9-%5Cfrac%7B3%7D%7B5%7D%5Cint%5Cfrac%7Bx%5E%7B7%7D%7 D%7B%28x&plus;%5Cfrac%7B2%7D%7B%5Csqrt[3]%7B3%7D%7D%29%28x%5E%7B2%7D-%5Cfrac%7B2%7D%7B%5Csqrt[3]%7B3%7D%7Dx&plus;%5Cfrac%7B4%7D%7B%5Csqrt[3]%7B9%7D%7D%29%7Ddx


که در انتگرال دوم وقتی کسر آخر رو به تجزیه کنیم و به صورت مجموع دو کسر در بیاریم، قابل حل خواهد بود و احتمالا به فرم یک چندجمله ای به انضمام یک آرک تانژانت در خواهد آمد.


موفق باشین.
91/2/15

skyzare
04-05-2012, 16:08
با سلام .

می دونم جاش این جا نیست ولی جواب اخر اون انتگرالتون این میشه :


برای مشاهده محتوا ، لطفا وارد شوید یا ثبت نام کنید

hts1369
04-05-2012, 16:08
سلام

جوابی که نوشتین درست نیست و کافیه که خودتون برای اطمینان از جواب آخر مشتق بگیرین تا متوجه بشین.

جوابش یکمی طولانیه و بنده تا اونجایی که دیگه خودتون به راحتی بتونین تا آخر ادامه بدین، نوشتم:


[ برای مشاهده لینک ، لطفا با نام کاربری خود وارد شوید یا ثبت نام کنید ] %5E%7B3%7D&plus;8%29dx%5Crightarrow&space;u=%5Cln&space;%283x%5E%7B 3%7D&plus;8%29%5C;&space;%5C;&space;,%5C;&space;%5C;&space;dv=x%5E%7B4%7Ddx%5C: &space;%5Crightarrow&space;du=%5Cfrac%7B9x%5E%7B2%7D%7D%7B3x%5 E%7B3%7D&plus;8%7Ddx%5C;&space;%5C;&space;,%5C;&space;%5C;&space;v=%5Cfrac%7Bx% 5E%7B5%7D%7D%7B5%7D%5CRightarrow&space;I=%5Cint&space;u%5C,.&space;d v=u.v-%5Cint&space;v%5C,&space;.du=%5Cfrac%7Bx%5E%7B5%7D%7D%7B5%7D%5 Cln&space;%283x%5E%7B3%7D&plus;8%29-%5Cfrac%7B9%7D%7B5%7D%5Cint&space;%5Cfrac%7Bx%5E%7B7%7D% 7D%7B3x%5E%7B3%7D&plus;8%7Ddx

[ برای مشاهده لینک ، لطفا با نام کاربری خود وارد شوید یا ثبت نام کنید ] D%5Cln&space;%283x%5E%7B3%7D&plus;8%29-%5Cfrac%7B3%7D%7B5%7D%5Cint&space;%5Cfrac%7Bx%5E%7B7%7D% 7D%7Bx%5E%7B3%7D&plus;%5Cfrac%7B8%7D%7B3%7D%7Ddx=%5Cfra c%7Bx%5E%7B5%7D%7D%7B5%7D%5Cln&space;%283x%5E%7B3%7D&plus;8%2 9-%5Cfrac%7B3%7D%7B5%7D%5Cint%5Cfrac%7Bx%5E%7B7%7D%7 D%7B%28x&plus;%5Cfrac%7B2%7D%7B%5Csqrt[3]%7B3%7D%7D%29%28x%5E%7B2%7D-%5Cfrac%7B2%7D%7B%5Csqrt[3]%7B3%7D%7Dx&plus;%5Cfrac%7B4%7D%7B%5Csqrt[3]%7B9%7D%7D%29%7Ddx


که در انتگرال دوم وقتی کسر آخر رو به تجزیه کنیم و به صورت مجموع دو کسر در بیاریم، قابل حل خواهد بود و احتمالا به فرم یک چندجمله ای به انضمام یک آرک تانژانت در خواهد آمد.


موفق باشین.
91/2/15



داش حمید دمت گرم ولی توانها ماله x نیست برای ln هست(متمتیکا اینجوری نشون میده)

skyzare
04-05-2012, 16:18
اگه منظورتون توی جواب هست به نظرم باز هم اشتباه داده بنده خدا قاطی کرده :31:

hts1369
04-05-2012, 16:56
اگه منظورتون توی جواب هست به نظرم باز هم اشتباه داده بنده خدا قاطی کرده :31:
اره به گمونم قاطی کرده بنده خدا
با maple زدم یه چیز دیگه داد (دو خط نوشته بود)

siyanor
04-05-2012, 23:43
با تشكر از شما دوست عزيز
پس دامنه ميشه تمام اعداد حقيقى به غير از اعداد بين 2 و 3 , درسته ؟
يه سوال ديگه
فرض كنيم كه به ما اين تصوير رو دادند و از ما محيط و مساحت هشت ضلعى و محيط و مساحت نيم دايره ها رو بخواهند
[ برای مشاهده لینک ، لطفا با نام کاربری خود وارد شوید یا ثبت نام کنید ]
همينطور كه تو شكل معلومه اندازه طول و عرض ( كه يك مستطيل رو تشكيل ميدهند داده شده ) در هر زاويه هم يك مثلث متساوى الساقين با زاويه 90 درجه وجود داره من يك محاسباتى كردم اگه ميشه دوستان يك نظرى بدهند ممنون ميشم
[ برای مشاهده لینک ، لطفا با نام کاربری خود وارد شوید یا ثبت نام کنید ]
اول محيط هشت ضلعى هست بعد مساحت اون بعد محيط نيم دايره و بعد مساحت نيم دايره ها (بايد مفدار آخر رو در 4 ضرب كنيم تا واسه 4 تا نيم دايره بدست بياد)
اگه اين محاسبات درسته راهى وجود داره كه مساحت هشت ضلعى از اين كمتر هم حساب بشه ( كمترين مقدار ممكن )

lebesgue
04-05-2012, 23:56
اره به گمونم قاطی کرده بنده خدا
با maple زدم یه چیز دیگه داد (دو خط نوشته بود)

لطفاً دستوری رو که به متمتیکا دادین، عیناً اینجا کپی پیست کنین.

davy jones
05-05-2012, 00:04
با تشكر از شما دوست عزيز
پس دامنه ميشه تمام اعداد حقيقى به غير از اعداد بين 2 و 3 , درسته ؟
يه سوال ديگه
فرض كنيم كه به ما اين تصوير رو دادند و از ما محيط و مساحت هشت ضلعى و محيط و مساحت نيم دايره ها رو بخواهند
[ برای مشاهده لینک ، لطفا با نام کاربری خود وارد شوید یا ثبت نام کنید ]
همينطور كه تو شكل معلومه اندازه طول و عرض ( كه يك مستطيل رو تشكيل ميدهند داده شده ) در هر زاويه هم يك مثلث متساوى الساقين با زاويه 90 درجه وجود داره من يك محاسباتى كردم اگه ميشه دوستان يك نظرى بدهند ممنون ميشم
[ برای مشاهده لینک ، لطفا با نام کاربری خود وارد شوید یا ثبت نام کنید ]
اول محيط هشت ضلعى هست بعد مساحت اون بعد محيط نيم دايره و بعد مساحت نيم دايره ها (بايد مفدار آخر رو در 4 ضرب كنيم تا واسه 4 تا نيم دايره بدست بياد)
اگه اين محاسبات درسته راهى وجود داره كه مساحت هشت ضلعى از اين كمتر هم حساب بشه ( كمترين مقدار ممكن )

سلام.

من شکلی نمیبینم.
نمیدونم این مشکل منه یا همگانیه چون مدتیه که برخی تصاویر رو اصلا در این فروم نمیبینم :13:


موفق باشین.
91/2/15

siyanor
05-05-2012, 12:22
سلام.

من شکلی نمیبینم.
نمیدونم این مشکل منه یا همگانیه چون مدتیه که برخی تصاویر رو اصلا در این فروم نمیبینم :13:


موفق باشین.
91/2/15

سلام
من دو تا عكس رو يه جا ديگه براتون آپلود كردم
[ برای مشاهده لینک ، لطفا با نام کاربری خود وارد شوید یا ثبت نام کنید ]

[ برای مشاهده لینک ، لطفا با نام کاربری خود وارد شوید یا ثبت نام کنید ]

hts1369
05-05-2012, 13:58
لطفاً دستوری رو که به متمتیکا دادین، عیناً اینجا کپی پیست کنین.
سلام بفرمایید
این سوال [ برای مشاهده لینک ، لطفا با نام کاربری خود وارد شوید یا ثبت نام کنید ]
و اینم جوابی که میده [ برای مشاهده لینک ، لطفا با نام کاربری خود وارد شوید یا ثبت نام کنید ]
بعد میبخشید شما روشی بهتر از روشی که اقا حمید تو صفحه ی قبلی گفتن به ذهنتون نمیرسه؟

zr1
05-05-2012, 19:03
دوستان کسی نیست کمکم کنه:worried:.والا بیچاره میشم کسی کمکم نکنه.این درس برای من خیلی تازه هست و هنوز دستم نیفتاده.

lebesgue
05-05-2012, 19:04
سلام بفرمایید
این سوال [ برای مشاهده لینک ، لطفا با نام کاربری خود وارد شوید یا ثبت نام کنید ]
و اینم جوابی که میده [ برای مشاهده لینک ، لطفا با نام کاربری خود وارد شوید یا ثبت نام کنید ]


خب باید براکت بذارید، نه پرانتز:


[ برای مشاهده لینک ، لطفا با نام کاربری خود وارد شوید یا ثبت نام کنید ]



بعد میبخشید شما روشی بهتر از روشی که اقا حمید تو صفحه ی قبلی گفتن به ذهنتون نمیرسه؟
نه.

davy jones
06-05-2012, 09:54
سلام
من دو تا عكس رو يه جا ديگه براتون آپلود كردم
[ برای مشاهده لینک ، لطفا با نام کاربری خود وارد شوید یا ثبت نام کنید ]

[ برای مشاهده لینک ، لطفا با نام کاربری خود وارد شوید یا ثبت نام کنید ]

سلام

ممنونم.

مساحت هشت ضلعی برابر میشه با مساحت مستطیل منهای 4 برابر مساحت یکی از مثلثها یعنی:


[ برای مشاهده لینک ، لطفا با نام کاربری خود وارد شوید یا ثبت نام کنید ] rrow&space;%7B%5Ccolor%7BPurple%7D&space;S%28x%29=2400-2x%5E%7B2%7D%7D


محیط هشت ضلعی هم برابر میشه با محیط مستطیل منهای 4 برابر محیط یک مثلث به علاوه ی 8 برابر طول وتر یک مثلث. یعنی:


[ برای مشاهده لینک ، لطفا با نام کاربری خود وارد شوید یا ثبت نام کنید ] ghtarrow&space;%7B%5Ccolor%7BPurple%7D&space;P%28x%29=200&plus;%284 %5Csqrt%7B2%7D-8%29x%7D


اما از طرفی باید شعاع دایره رو بر حسب x پیدا کنیم. از اونجایی که مثلثهای گوشه ای قائم الزاویه ی متساوی الساقین هستند، پس مطمئنا وتر مثلثها، قطر دایره محیطی میشن (چرا؟) پس شعاع هر یک از دایره ها برابر میشه با نصف طول یکی از وترها یعنی برابر با: [ برای مشاهده لینک ، لطفا با نام کاربری خود وارد شوید یا ثبت نام کنید ]


پس مجموع مساحت 4 تا نیمدایره ی موجود برابر میشه با مساحت دو دایره با شعاع [ برای مشاهده لینک ، لطفا با نام کاربری خود وارد شوید یا ثبت نام کنید ]


[ برای مشاهده لینک ، لطفا با نام کاربری خود وارد شوید یا ثبت نام کنید ] 2%7D=2%5Cpi&space;%28%5Cfrac%7Bx%5Csqrt%7B2%7D%7D%7B2%7D %29%5E%7B2%7D%5CRightarrow&space;%7B%5Ccolor%7BDarkOrang e%7D&space;S_%7B2%7D%28x%29=%5Cpi&space;x%5E%7B2%7D%7D


محیط نیمدایره ها هم با این توضیحات برابر میشه با:


[ برای مشاهده لینک ، لطفا با نام کاربری خود وارد شوید یا ثبت نام کنید ] 2%5Cpi&space;r%7D%7B2%7D&plus;2r%29=4%5Ctimes&space;%28%5Cfrac%7B%5 Cpi&space;%5Csqrt%7B2%7D%7D%7B2%7Dx&plus;x%5Csqrt%7B2%7D%29%5 CRightarrow&space;%7B%5Ccolor%7BDarkOrange%7D&space;P_%7B2%7D% 28x%29=2%5Csqrt%7B2%7D%28%5Cpi&plus;2%29x%7D



---------------------

کمترین مقدار مساحت هشت ضلعی، طبق فرمول به دست اومده، به ازای بیشترین مقدار x ممکن به دست خواهد اومد. بیشترین مقدار x در اینجا برابر با 20 میتونه باشه. البته اگر x=20 باشه اونوقت هشت ضلعی تبدیل به شش ضلعی میشه. اما میشه فرض کرد که مساحت هشت ضلعی به طور حدی هنگامی که x از سمت چپ به 20 میل میکنه، کمترین مقدار خودش رو داره:


[ برای مشاهده لینک ، لطفا با نام کاربری خود وارد شوید یا ثبت نام کنید ] 5E%7B-%7D%7DS%28x%29=%7B%5Ccolor%7BGolden%7D&space;%5Clim_%7Bx &space;%5Cto&space;20%5E%7B-%7D%7D%282400-2x%5E%7B2%7D%29%5Crightarrow&space;1600%7D


امیدوارم توضیحاتم کامل بوده باشه


موفق باشین.
91/2/17

siyanor
06-05-2012, 10:58
خيلى ممنون كه جواب داديد دوست عزيز
من اين مورد

محیط هشت ضلعی هم برابر میشه با محیط مستطیل منهای 4 برابر محیط یک مثلث به علاوه ی 8 برابر طول وتر یک مثلث.
رو درست متوجه نشدم مگه محيط مثلث برابر مجموع 3 تا ضلع نميشه ( نميشه اونا رو به صورت مربع حساب كنيم ؟ با قرار دادن هر دو مثلث كنار هم يك مربع با ضلع x به وجود مياد ؟)
قسمت آخر مربوط به Lim رو من نخوندم ؟!! ميشه يه كم توضيح بديد ( يا يه لينك بدى كه متوجه بشم )

باز هم ازتون ممنونم

davy jones
06-05-2012, 11:42
خيلى ممنون كه جواب داديد دوست عزيز
من اين مورد
رو درست متوجه نشدم مگه محيط مثلث برابر مجموع 3 تا ضلع نميشه ( نميشه اونا رو به صورت مربع حساب كنيم ؟ با قرار دادن هر دو مثلث كنار هم يك مربع با ضلع x به وجود مياد ؟)
قسمت آخر مربوط به Lim رو من نخوندم ؟!! ميشه يه كم توضيح بديد ( يا يه لينك بدى كه متوجه بشم )

باز هم ازتون ممنونم

سلام

نه نمیشه. چون با کنار هم قرار دادن دو مثلث، در واقع داریم دو وتر رو از بین میبریم و حذف میکنیم که این کار شاید در محاسبات مربوط به مساحت، خللی وارد نکنه ولی روی محیط مسلما تاثیر داره. چون محیط یک مربع با مجموع محیط دو مثلث که هر کدوم نصف اون مربع هستند، فرق داره. فرقش هم همون دو وتر هستش. همونطور که در محاسبه ی محیط نیمدایره ها هم نمیتونیم بگیم که مجموع محیط 2 نیم دایره برابر با محیط یک دایره ی کامل هستش. چون داریم با اینکار طول 2 تا قطر رو نادیده میگیریم.

در مورد لیمیت هم اگه نخوندین، مهم نیست. در آینده ایشالله میخونین. :31: (سال سوم دبیرستان) ولی اگه تمایل دارین مطالعه ی آزاد و جداگونه ای در این زمینه داشته باشین به لینک زیر مراجعه کنین هرچند که مباحثش یکمی الان براتون احتمالا سنگین هستش:

[ برای مشاهده لینک ، لطفا با نام کاربری خود وارد شوید یا ثبت نام کنید ] ([ برای مشاهده لینک ، لطفا با نام کاربری خود وارد شوید یا ثبت نام کنید ] Fwiki%2F%25D8%25AD%25D8%25AF_%2528%25D8%25B1%25DB% 258C%25D8%25A7%25D8%25B6%25DB%258C%2529)



موفق باشین.
91/2/17

siyanor
06-05-2012, 12:52
راستش من الان بعد تقريبا 8 سال برگشتم سر درس و كتاب :31: سال سوم ما از اينا داشتيم :18:
يه سوال ديگه فرض كنيم ميخواهيم بدونيم كه چه موقع محيط و مساحت هشت ضلعى مون برابر با اون نيم دايره ها ميشه (بدون حساب كردن قطر ), اين رو چيكار بايد كر د( كلا من تو رياضى با شكلها و مثلثات مشكل دارم :2:)

hts1369
06-05-2012, 20:20
درود

معادلات برنولی
[ برای مشاهده لینک ، لطفا با نام کاربری خود وارد شوید یا ثبت نام کنید ]

[ برای مشاهده لینک ، لطفا با نام کاربری خود وارد شوید یا ثبت نام کنید ]
تمرین های گوناگون
[ برای مشاهده لینک ، لطفا با نام کاربری خود وارد شوید یا ثبت نام کنید ]

من عامل انتگرال ساز رو خوب بلد نیستم. اگه بقیه صلاح دونستن جواب میدن

[ برای مشاهده لینک ، لطفا با نام کاربری خود وارد شوید یا ثبت نام کنید ] y%5E3&space;%5C%5C&space;n=3%5CRightarrow&space;z=y%5E%7B-2%7D%5CRightarrow&space;z%27=-2y%5E%7B-3%7Dy%27=-2%5Cfrac%7By%27%7D%7By%5E3%7D%5CRightarrow&space;%5Cfrac %7By%27%7D%7By%5E3%7D=%5Cfrac%7B-z%27%7D%7B2%7D&space;%5C%5C&space;xy%27&plus;y=x%5E4y%5E3%5CRightar row&space;x%5Cfrac%7By%27%7D%7By%5E3%7D&plus;%5Cfrac%7B1%7D%7 By%5E2%7D=x%5E4%5CRightarrow&space;x%5Cleft%28%5Cfrac%7B-z%27%7D%7B2%7D%5Cright%29&plus;z=x%5E4%5CRightarrow&space;z%2 7-%5Cfrac%7B2z%7D%7Bx%7D=-2x%5E3&space;%5C%5C&space;z%27-%5Cfrac%7B2z%7D%7Bx%7D=-2x%5E3&space;%5Cend%7Barray%7D

حالا این یه معادله ی خطی مرتبه ی اول هست که با همون رابطه ی بزرگ حل میشه. اینجوری که من حلش میکنم فکر کنم راحت تر هست باز خود دانید اول یه عامل انتگرال ساز پیدا میکنیم و بعد حلش میکنیم.

[ برای مشاهده لینک ، لطفا با نام کاربری خود وارد شوید یا ثبت نام کنید ] c%7B1%7D%7By%5E2%7D=-x%5E4&plus;cx%5E2%5CRightarrow&space;y=%5Cpm&space;%5Cfrac%7B1%7D%7 B%5Csqrt%7B-x%5E4&plus;c&space;x%5E2%7D%7D&space;%5Cend%7Barray%7D

این هم برنولی دومت دیگه حل خطی مرتبه ی اول با خودت کمی انتگرالش طولانی میشه.

[ برای مشاهده لینک ، لطفا با نام کاربری خود وارد شوید یا ثبت نام کنید ] 5E3=x%5Ccos&space;x&space;%5C%5C&space;y%27&plus;%5Cfrac%7By%7D%7Bx%7D=%5 Cfrac%7B%5Ccos&space;x%7D%7By%5E2%7D%5CRightarrow&space;n=-2%5CRightarrow&space;z=y%5E3%5CRightarrow&space;z%27=3y%5E2y%2 7%5CRightarrow&space;y%5E2y%27=%5Cfrac%7Bz%27%7D%7B3%7D&space; %5C%5C&space;xy%5E2y%27&plus;y%5E3=x%5Ccos&space;x%5CRightarrow&space;x%5 Cfrac%7Bz%27%7D%7B3%7D&plus;z=x%5Ccos&space;x%5CRightarrow&space;z% 27&plus;%5Cfrac%7B3z%7D%7Bx%7D=3%5Ccos&space;x&space;%5Cend%7Barray %7D

معادله ی پنجمت هم خطی مرتبه ی اول هست ولی باید بر حسب x حلش کنی

[ برای مشاهده لینک ، لطفا با نام کاربری خود وارد شوید یا ثبت نام کنید ] %7By%5E2%7D%7B1-xy%7D%5CRightarrow&space;%5Cfrac%7Bdx%7D%7Bdy%7D=%5Cfrac %7B1-xy%7D%7By%5E2%7D=-%5Cfrac%7Bx%7D%7By%7D&plus;%5Cfrac%7B1%7D%7By%5E2%7D%5C Rightarrow&space;x%27=-%5Cfrac%7Bx%7D%7By%7D&plus;%5Cfrac%7B1%7D%7By%5E2%7D&space;%5 C%5C&space;x%27&plus;%5Cfrac%7Bx%7D%7By%7D=%5Cfrac%7B1%7D%7By %5E2%7D&space;%5C%5C&space;e%5E%7B%5Cint&space;%5Cfrac%7B1%7D%7By%7D dy%7D=y&space;%5C%5C&space;y%5Cleft%28x%27&plus;%5Cfrac%7Bx%7D%7By% 7D%5Cright%29=y%5Cleft%28%5Cfrac%7B1%7D%7By%5E2%7D %5Cright%29%5CRightarrow&space;yx%27&plus;x=%5Cfrac%7B1%7D%7B y%7D%5CRightarrow&space;yx=%5Cln&space;y&plus;c%5CRightarrow&space;x=%5Cf rac%7B%5Cln&space;y%7D%7By%7D&plus;%5Cfrac%7Bc%7D%7By%7D&space;%5Ce nd%7Barray%7D

davy jones
06-05-2012, 23:38
يه سوال ديگه فرض كنيم ميخواهيم بدونيم كه چه موقع محيط و مساحت هشت ضلعى مون برابر با اون نيم دايره ها ميشه (بدون حساب كردن قطر ), اين رو چيكار بايد كر د( كلا من تو رياضى با شكلها و مثلثات مشكل دارم :2:)
سلام

فرمول پارامتری مساحت هشت ضلعی و فرمول مجموع مساحت نیمدایره ها رو که حساب کردیم و میدونیم. کافیه که این دو فرمول رو مساوی هم بذاریم تا ببینیم به ازای چه مقداری از x تساوی برقرار میشه. خیلی راحته. خودتون امتحان کنین. کافیه که یه بلد باشین ریشه های یه معادله ی درجه ی 2 ساده رو به دست بیارین. اگه نتونستین در خدمتتونم.



موفق باشین.
91/2/18

davy jones
07-05-2012, 11:37
من عامل انتگرال ساز رو خوب بلد نیستم. اگه بقیه صلاح دونستن جواب میدن



هر یک از معادلات زیر را با به دست اوردن یک عامل انتگرال ساز حل کنید
1- [ برای مشاهده لینک ، لطفا با نام کاربری خود وارد شوید یا ثبت نام کنید ]
--------------------
2- [ برای مشاهده لینک ، لطفا با نام کاربری خود وارد شوید یا ثبت نام کنید ]


سلام.

این شاید آخرین بار باشه که تمرینهای دانشگاهی رو برای کسی حل میکنم در اینجا. چون احساس کردم که ممکنه واقعا توش مشکل داشته باشین. ولی بعد از این تمارین و تکالیف دانشگاهی برای کسی حل نخواهم کرد.

در اینگونه معادلات دیفرانسیل، ابتکار و خلاقیت و دید ریاضی داشتن خیلی مهمه. باید حدس بزنین که از چه تغییر متغیر یا تابع کمکی انتگرال ساز باید استفاده کرد و این حدس هر چه خلاقیت و دیدتون بیشتر باشه، به واقعیت نزدیک تر میشه. چون حل این معادلات در حالت کلی خیلی وقت گیر و کمر شکنه. شاید عامل انتگرال سازی که باید استفاده کرد، تابعی از x باشه و شایدم تابعی از y باشه و یا هر دو. اگر از هر دو متغیر x و y درش وجود داره، شاید جداشدنی به صورت جمع یا ضرب باشه و شاید نباشه و ... پس هر چه دید بهتری داشته باشیم و خلاقیت رو چاشنیش کنیم، حل کردن اینگونه معادلات برامون راحت تر و سریعتر امکانپذیره.


در معادله ی اول کافیه که به یه نحوی از شر x به توان 2 خلاص شیم و کاری کنیم که بشه عامل [ برای مشاهده لینک ، لطفا با نام کاربری خود وارد شوید یا ثبت نام کنید ] رو فاکتور گرفت و ساده کرد. در اینجا با کمی دقت میبینیم که اگه از تغییر متغیر [ برای مشاهده لینک ، لطفا با نام کاربری خود وارد شوید یا ثبت نام کنید ] استفاده کنیم، به هدفمون میرسیم. چرا که در هر دو پرانتز، عامل [ برای مشاهده لینک ، لطفا با نام کاربری خود وارد شوید یا ثبت نام کنید ] تولید میشه که میشه فاکتور گرفت و حذف کرد:


[ برای مشاهده لینک ، لطفا با نام کاربری خود وارد شوید یا ثبت نام کنید ]
[ برای مشاهده لینک ، لطفا با نام کاربری خود وارد شوید یا ثبت نام کنید ] Cmathrm%7Bd%7D&space;x%7D-2xy=0
[ برای مشاهده لینک ، لطفا با نام کاربری خود وارد شوید یا ثبت نام کنید ] tarrow&space;%7By%7D%27%28x%29=%28f%28x%29&plus;x.%5Cfrac%7B% 5Cmathrm%7Bd%7D&space;f%28x%29%7D%7B%5Cmathrm%7Bd%7D&space;x%7 D%29
[ برای مشاهده لینک ، لطفا با نام کاربری خود وارد شوید یا ثبت نام کنید ] &space;f%7D%7B%5Cmathrm%7Bd%7D&space;x%7D%29-2x%5E%7B2%7Df=0

[ برای مشاهده لینک ، لطفا با نام کاربری خود وارد شوید یا ثبت نام کنید ] 7D%7B%5Cmathrm%7Bd%7D&space;x%7D%29-2f=0

[ برای مشاهده لینک ، لطفا با نام کاربری خود وارد شوید یا ثبت نام کنید ]

[ برای مشاهده لینک ، لطفا با نام کاربری خود وارد شوید یا ثبت نام کنید ] arrow&space;%5Cfrac%7B3-f%5E%7B2%7D%7D%7Bf%5E%7B3%7D-f%7D%5C;&space;df=%5Cfrac%7Bdx%7D%7Bx%7D%5CRightarrow&space;%5 Cint&space;%5Cfrac%7B3-f%5E%7B2%7D%7D%7Bf%5E%7B3%7D-f%7D%5C;&space;df=%5Cint&space;%5Cfrac%7Bdx%7D%7Bx%7D

[ برای مشاهده لینک ، لطفا با نام کاربری خود وارد شوید یا ثبت نام کنید ] &plus;%5Cfrac%7B1%7D%7B3%7D-f%5E%7B2%7D%7D%7Bf%5E%7B3%7D-f%7D%5C;&space;df=%5Cint&space;%28%5Cfrac%7B%5Cfrac%7B1%7D%7B3 %7D-f%5E%7B2%7D%7D%7Bf%5E%7B3%7D-f%7D&plus;%5Cfrac%7B%5Cfrac%7B8%7D%7B3%7D%7D%7Bf%5E%7B3 %7D-f%7D%29%5C;&space;df=%5Cint&space;%5Cfrac%7B%5Cfrac%7B1%7D%7B3 %7D-f%5E%7B2%7D%7D%7Bf%5E%7B3%7D-f%7D%5C;&space;df&plus;%5Cint&space;%5Cfrac%7B%5Cfrac%7B8%7D%7B3%7D %7D%7Bf%5E%7B3%7D-f%7D%5C;&space;df=%5Cfrac%7B1%7D%7B3%7D%5Cln&space;%28f%5E%7B3 %7D-f%29&plus;%5Cint&space;%5Cfrac%7B%5Cfrac%7B8%7D%7B3%7D%5C;&space;df %7D%7Bf%28f&plus;1%29%28f-1%29%7D

[ برای مشاهده لینک ، لطفا با نام کاربری خود وارد شوید یا ثبت نام کنید ] 7D-f%29&plus;%5Cint%28%5Cfrac%7B%5Cfrac%7B-8%7D%7B3%7D%7D%7Bf%7D&plus;%5Cfrac%7B%5Cfrac%7B4%7D%7B3 %7D%7D%7Bf-1%7D&plus;%5Cfrac%7B%5Cfrac%7B4%7D%7B3%7D%7D%7Bf&plus;1%7D%2 9df=%5Cfrac%7B1%7D%7B3%7D%5Cln&space;%28f%5E%7B3%7D-f%29-%5Cfrac%7B8%7D%7B3%7D%5Cln&space;%28f%29&plus;%5Cfrac%7B4%7D% 7B3%7D%5Cln&space;%28f-1%29&plus;%5Cfrac%7B4%7D%7B3%7D%5Cln&space;%28f&plus;1%29=%5Cint&space;% 5Cfrac%7Bdx%7D%7Bx%7D=%5Cln&space;%28cx%29

[ برای مشاهده لینک ، لطفا با نام کاربری خود وارد شوید یا ثبت نام کنید ][%5Cfrac%7B%28f%5E%7B3%7D-f%29%28f&plus;1%29%5E%7B4%7D%28f&plus;1%29%5E%7B4%7D%7D%7Bf% 5E%7B8%7D%7D]=%5Cln&space;%28cx%5E%7B3%7D%29

[ برای مشاهده لینک ، لطفا با نام کاربری خود وارد شوید یا ثبت نام کنید ] Rightarrow&space;f%5E%7B10%7D-5f%5E%7B8%7D-cx%5E%7B3%7Df%5E%7B7%7D&plus;10f%5E%7B6%7D-10f%5E%7B4%7D&plus;5f%5E%7B2%7D-1=0



که در اینجا باید ریشه های این معادله ی درجه 10 بر حسب f رو بدست بیاریم تا جواب نهایی بدست بیاد که دیگه از حوصله ی بحث خارجه. وقتی که f بدست اومد، مقدار y که جواب نهایی مساله است برابر میشه با [ برای مشاهده لینک ، لطفا با نام کاربری خود وارد شوید یا ثبت نام کنید ]

و اما جواب آخر این معادله با کمک نرم افزار متمتیکا:


[ برای مشاهده لینک ، لطفا با نام کاربری خود وارد شوید یا ثبت نام کنید ]


مشاهده میکنین که اگه عنصر خلاقیت و حدس بر پایه ی دید ریاضیاتی رو در دستور کار نمیذاشتیم، چقدر باید سر این مساله وقت صرف میکردیم!!!

---------------------------------------------

در معادله ی دوم هم روال کلی کار بر اساس همون معادله ی اوله. در اینجا دقت میکنیم که اگه یک x از کل معادله فاکتور بگیریم و ساده کنیم، معادله به فرم معادله ی کامل در خواهد آمد:


[ برای مشاهده لینک ، لطفا با نام کاربری خود وارد شوید یا ثبت نام کنید ]
[ برای مشاهده لینک ، لطفا با نام کاربری خود وارد شوید یا ثبت نام کنید ]
[ برای مشاهده لینک ، لطفا با نام کاربری خود وارد شوید یا ثبت نام کنید ] 28y-%5Cfrac%7B1%7D%7Bx%7D%29%7D%7B%5Cpartial&space;y%7D%5CRi ghtarrow&space;the%5C;&space;equation%5C;&space;is%5C;&space;exact


بنابراین بقیه اش دیگه از راه معادله ی کامل حل خواهد شد که به عنوان تمرین به عهده ی خودتون میذارم چرا که جناب hts1369 یه دور در اینجا ([ برای مشاهده لینک ، لطفا با نام کاربری خود وارد شوید یا ثبت نام کنید ])زحمت کشیدن و نحوه ی حل معادلات دیفرانسیل کامل رو به خوبی توضیح دادند.

دقت کنید که اگه میخواستیم به روش کلاسیک و اصولی، عامل انتگرال ساز رو پیدا کنیم، بعد از اینکه ده ها خط باید فرمول مینوشتیم و حساب میکردیم، دست آخر میرسیدیم به این که عامل انتگرال ساز تابع [ برای مشاهده لینک ، لطفا با نام کاربری خود وارد شوید یا ثبت نام کنید ] خواهد بود در حالیکه با کمی دقت به راحتی تونستیم مراحل کار رو برای خودمون بسیار ساده و راحت کنیم.



موفق باشین.
91/2/18

siyanor
07-05-2012, 13:40
منظورتون همون فرمول[ برای مشاهده لینک ، لطفا با نام کاربری خود وارد شوید یا ثبت نام کنید ] هست ديگه ؟ ولى خب تو مساله داره ميگه بدون حساب كردن قطر نيم دايره ! اگه كارى كه شما فرموديد انجام بديم عملا داريم قطر رو هم حساب ميكنيم , درسته ؟
(چه جورى ميشه اينجا فرمولهاى رياضى رو تايپ كرد , يا بايد عكس رو كپى پيست كنيم ؟)

ممنون

davy jones
07-05-2012, 14:09
منظورتون همون فرمول[ برای مشاهده لینک ، لطفا با نام کاربری خود وارد شوید یا ثبت نام کنید ] هست ديگه ؟ ولى خب تو مساله داره ميگه بدون حساب كردن قطر نيم دايره ! اگه كارى كه شما فرموديد انجام بديم عملا داريم قطر رو هم حساب ميكنيم , درسته ؟
(چه جورى ميشه اينجا فرمولهاى رياضى رو تايپ كرد , يا بايد عكس رو كپى پيست كنيم ؟)

ممنون

سلام.

بله منظورم همونه.

خب من از شما یه سوال میپرسم: چطوری مساحت یک دایره (یا نیمدایره یا هر قطاع دلخواه از یک دایره) رو بدون دونستن شعاع اون دایره میخواین به دست بیارین؟ چطوری؟ البته روشهای انتگرالی دوگاه و سه گانه برای این کار وجود داره ولی خیلی سخته و در حد دبیرستان نیست. مشخصه که در اینجا باید شعاع دایره رو حساب کنیم تا بتونیم مساحت دایره رو به دست بیاریم.

برای تایپ فرمولهای ریاضی هم از سایت زیر کمک بگیرین (تقریبا همه ی دوستان در اینجا از جمله خودم با استفاده از این سایت دارن فرمول مینویسن) :

[ برای مشاهده لینک ، لطفا با نام کاربری خود وارد شوید یا ثبت نام کنید ]


موفق باشین.
91/2/18

mojtaba2321
07-05-2012, 22:26
بچه ها این مسئله ساده رو من نمیتونم اثبات کنم: [ برای مشاهده لینک ، لطفا با نام کاربری خود وارد شوید یا ثبت نام کنید ]
میشه کمکم کنید:20:

ali_hp
07-05-2012, 22:28
سلام بر دوستان گرامی؛

کسی اثبات استقرائی برای:
اگر[ برای مشاهده لینک ، لطفا با نام کاربری خود وارد شوید یا ثبت نام کنید ] ها اعداد حقیقی مثبتی از 1 تا n و [ برای مشاهده لینک ، لطفا با نام کاربری خود وارد شوید یا ثبت نام کنید ] _%7B2%7D باشد، آنگاه :

[ برای مشاهده لینک ، لطفا با نام کاربری خود وارد شوید یا ثبت نام کنید ] D%5E%7Bn%7Dx_%7Bi%7D%5E%7BP_%7B2%7D%7D%29%5E%7B%5C frac%7B1%7D%7BP_%7B2%7D%7D%7D%5Cleqslant%20%28%5Cs um_%7Bi=1%7D%5E%7Bn%7Dx_%7Bi%7D%5E%7BP_%7B1%7D%7D% 29%5E%7B%5Cfrac%7B1%7D%7BP_%7B1%7D%7D%7D%7D
سراغ داره ؟
من خودم از یه روش برای اثبات k+1 امین جمله‌ش پیش رفتم ولی به نتیجه‌ی مورد نظر نرسیدم ... یعنی نتونستم جواب رو ازش بکشم بیرون ...
اگه دوست دارید روشم اینه :
به ترتیب به طرف چپ [ برای مشاهده لینک ، لطفا با نام کاربری خود وارد شوید یا ثبت نام کنید ] D و به طرف راست [ برای مشاهده لینک ، لطفا با نام کاربری خود وارد شوید یا ثبت نام کنید ] D رو کم و زیاد میکنیم.

اثبات غیر استقرائیش آسونه ...
سلام
من گام استقرا رو با استقرای قوی ثابت میکنم ، یعنی فرض می کنیم حکم برای همه k های کمتر اکید از n برقراره ، و بعد ثابت می کنیم برای n هم بر قراره.البته برای n>2 اینکارو می کنیم.
فقط مشکل اینه که اثبات حکم برای n=2 به نظر من فرق خاصی با اثبات حکم برای n دلخواه نداره،یا حداقل من راه حل ساده تری بلد نیستم،اینه که استفاده از استقرا اینجا بی مزه میشه!کلا اینجوری به نظر میاد که بار اصلی این مساله روی n=2 هست.و همونطور که خودتون و جناب 1233445566 گفتین راه حل غیر استقرایی ساده ای داره این مساله.
[ برای مشاهده لینک ، لطفا با نام کاربری خود وارد شوید یا ثبت نام کنید ] 9%5E%7B%5Cfrac%7B1%7D%7Bp_1%7D%7D=%28%28%28%5Csum_ %7Bi=1%7D%5E%7Bn-1%7Dx_i%5E%7Bp_1%7D%29%5E%7B%5Cfrac%7B1%7D%7Bp_1%7 D%7D%29%5E%7Bp_1%7D&plus;x_n%5E%7Bp_1%7D%29%5E%5Cfrac%7 B1%7D%7Bp_1%7D%5Cge%5C%5C&space;%28%28%28%5Csum_%7Bi=1%7 D%5E%7Bn-1%7Dx_i%5E%7Bp_1%7D%29%5E%7B%5Cfrac%7B1%7D%7Bp_1%7 D%7D%29%5E%7Bp_2%7D&plus;x_n%5E%7Bp_2%7D%29%5E%5Cfrac%7 B1%7D%7Bp_2%7D%5C%5C&space;%5Cge%28%28%28%5Csum_%7Bi=1%7 D%5E%7Bn-1%7Dx_i%5E%7Bp_2%7D%29%5E%7B%5Cfrac%7B1%7D%7Bp_2%7 D%7D%29%5E%7Bp_2%7D&plus;x_n%5E%7Bp_2%7D%29%5E%5Cfrac%7 B1%7D%7Bp_2%7D=%28%5Csum_%7Bi=1%7D%5Enx_i%5E%7Bp_2 %7D%29%5E%7B%5Cfrac%7B1%7D%7Bp_2%7D%7D

که نامساوی اول از درستی حکم برای 2 نتیجه میشه ، و نامساوی دوم از درستی حکم برای n-1 .